Download as pdf or txt
Download as pdf or txt
You are on page 1of 478

A Level Preliminary Examination Paper

H2MATHEMTICS


Hwa Chong Junior College


Anderson Junior College

Nttional Junior College


St.Andrew's Junior College


Serangoon Junior College
Pioneer Junior College

Yishull Junior College


Rames Junior College


Victoria JuniOr College
Tampines Junior Collcge
Nanya7ng Junior College


Mc dian Jllnior College






Ca olic JuniOr College






Anglo Chinese Junior Colicge





lnnova Junior College


Millennia lnstimte
_















HWA CHONG:NST:TUT:ON
JC2 PRELiMiNARY EXAMiNAT:ON 2007

MATHEMAT:CS 9740 01
Higher 2

Paper l

Wednesday 12 September 2007 3 hours

Addiliona:mateda:s: Answer paper


Cover Page
List of Fomula(MF15)

READ THESE INSTRUCTIONS FIRST

w_1'!eyoqr 1a.me and CTdass on all the wod( you hand in, induding the Cover page.
Write in dark blue or black pen on both siles of the paper.
You may use a soft pencil for any diagrams or graphs.
Do not use staples, paper clips, highlighters, glue or coneciion fluid.

Do not write anythlng on the List of Formula (MFlS).

Answer all the questions.


Begin each answer on a fresh page of paper.
Give non-exacl numerkxl answers conect to 3 significant figures, or 1 decimal place
in the.case of angles h degrees, unless a differeni level of aco.rracy is specified in the
question.
You are oxpeded to use a graphic calculator. .
Unsupported alstErs from a graphic catcutator are allowed unless a question
specifrcally states otherube.
where unsupported an$v6rs from a graphic calculator are not allowed in a question,
you are requlrcd tg pesent thefiatheinali, steps using mathemathat notations and
not calculator cornmands.
You are reguired of the need for dear presentation in your answers.

The number marrks given in brad<eb I t at the end of each question or part
queslion. At fle erd of-bthe examhation, phce the completed correr'page on top of
_of
your ansrer scripts and fasten all your work seanrely together wtttr ttre itri-ng provrued.

This queston paper conslsts of 5 printed pages and 1 blank page.

0 MTH 2007 97
ffurn over

"'

1. A sequence is defined by sr = I and u*, = for r = l, 2, 3,

,L*lu.

(" - Dl
Prove that ,. = by mathematical induction. 14]
n

2 (r) A graph with cquarirm y=f(:) undcrgoes tsansformaion I followed by


trarsformation B wkG A utd B arp describcd as follows:

l: e trardairn of 2 unib in ttc negativc direction of r-o<b '


B : a scaling paraltcl to tho 'r-oxis bY a factor
i '

The resutring cquatfrn is g(x)= 8.r+8+ith


Find thc equarion y=q.r), showing your workings ctearly' 121

(b) The graph of v= h(.r) is shown below'

[31

3. show thar s-qJi + x is alwal's positive for 'r > 0 '


5 -4 JI+ b.15]
O h terlns ofa and ,WhereO
X b) .
Hcn SOIVC the inequJ

4. A cylinder of radius r and hcight inSCrb h a nght cOnc of lXCd radiuS R and

h COntact wi hncr
height 8 u The circdar ends Of dlC tOp Of ttc cylhdcr

sCacc of the oOne and the basc of thc COnc lics in the sallte planc as the baSC Of thc

16 r2
Hen ind
C hdCr Show ultt me cu ed surface aca Of dlc Cylhder

the maximum curved surfacc area of the cylinder, in terms


of R' [6

MATH 2007 97 7

Indicale clearly on an Argand diagam, the region R represented by the locts


fz - ll <lz + rl. l2l

(D Find the.largst valuc of r such that all thc complex numbers w sadsrying
ln-\sr lieentirclyinside R. 121

10 eCOlllJ mm h R Arch =3 md =1 ]d

the cxrct value of 4 .
21

A trio-package consisting of a Notcbok (NB), a Home Thcdre System GmS) aod a


Television set (fV) cost a totsl of$7545. Two companfus offcr thc following discounts

ifa customer buys the rio-package:

Discounts glvcn for erch itcm in thc peckegc


Comprny Totrl prlcc
NB Irrs TV
A 10% 50/. 50/0 S7068.35

B 200/ 25% 100/ S6341.50


During the Grcar Singaporc Sale, ABS Bank crcdit card holdcrs are entitled lo a 6V/o,
50o/o and 307o discount for the NB, the HTS and the TV respectively for the trio-
package. How much would an ABS Ba* credit card holder have to pay in total for the
trio packagc?
6]

Expresstthpartialtt bn,

Hen , brmo
-1 ,W
5,JVingyouranswer h
nstant.

Dcdu extt v Ofi


6]

pe. t--
L"t u=
"*.Showrhat ,\dr,
I {1\-@\' -(r*t),.
= :t-.
) \ar.l

Hence, find 0re Maclaurin's exparsion for y, up to and including the term in

Deduce the Maclaurin's exparsion fo, y = up to and including the term in 'r2 '
"*

O HO MATH 2007 974001 N07 flurn over


4

9. An arithmetic progrcssion fu") is g;ouped into ses containing 1,3,9, ... integers as

shown below, so that thc number of integers in the zi set is 3'-r.

tr), {s, q l3}, [z 21 2s 2s 3i 37 41 4s 4s1,...

(l) In either ordcr, find exprcssions in tenns of a, for


(a) sct,
thc first intcgcr in fire ao 13]

(b) thc last imeger in the n 6 set 121

(lt) Show drat tlre sum of ttrc tcrms in thc an set ii grven by f [,](f')+ a],
wlrcre I and I are integers to bc dctrmined. l4l


10.


Tlrc parametric equations ofa curve C


are



y=e'5i11,


t=etcrolst and -tstsr.


wherc


(D Sketch the curve C, showing clearly the axial intercpts.
2

(ii) Find tlre equation ofthc tangent to the curve d the point f wherc t = {4 .
41

000n 0 c od n md O the"inton thc ttrve C su


"0=:
Widl making an obtuse angle with the positive .r-axis, Iind the area of the
Lianglc q2
41

11. The diagram shows the shaded region R bounded tt lines y=2,y=:,thC axls

and dle curvey=J FI


=2

1 3
71
2


) ShOW that dle area oF R is given by dr where a is a constant
I
(

to be detenlemed By umng me substtuuOn =: Sec 'lnd uC f


tte exacI I
me area Of R 17]
(b)Find dle volumc ofthe solid generated when R is rotatcd ulrough 2 about the

x ax ,giving your allswcr rFeCtt0 3 d imd places 13


O a MATH2007 974 N
nbn ctpN( I}EIINTFN 2odl htlrn frq,r (H2. e6tls Fn" r)

The functiom f and g are defined by


I'-4"-
I x J' ------.-:..:-- xelR, x+-3, x*l
(x+3)(x- t)'
Jr-tl
E:X-+e'. xeR
(i) Sketch on separate cleady labelled diagrams, the graphs of y=f(.r) *6
y = gQ) .Hcncc, or otherwise, show that 0re inversc frrnction of g does not cxisL
t3I
(it) Thc fimcrion h is deftrcd by
h :.r -r g(x), xe A,
wherc/ c IIlax3md Subsct of ulc interval(Q )Such ftat ttc inversc
:cxists.
mctiOn h
State thc sct z{ and givc tlrc rutc and domain of h-r . t4I
.
(iii) Dctermine which of trc following is a function:
(r) fg
o) fh-t t

In each casc, if the function exists, give is doman and range. [3]

y=l *f*4

The curvc has equation


x+b
It is given that C has a vertical asympotc x = -l and a stationary point at : = 2.

(i) Determine the values of a and 6 . t3I

(ii) Find the equation ofthe other asymptote of C . tll


(iii) Pmve, using an algebraic method, that C corurot lie betwecn two values (o be
detcrmined). t4l
(lv) Draw a sketch of C, showing clearly any axial intercpts, Gynptotes and
stationary points. t3I

(v) Deduoc the numbcr ofhal roots of the cquation


(c -.r')(:+ t)' = (x' - nx + +)' . 121

0 MATH 2007 0740r01m7 ffum over



HWA CHONG:NST:TUT10N
JC2 PREttM:NARY EXAMiNAT:ON 2007

MATHEMATICS 9740 02
Higher 2

Paper2

Frlday 14 September 2007 3 hours

Additiooal materials: Answer paper


Cover Page
List of Fomula MF15)

READ THESE INSTRUCTIONS FIRST

.Write your narn6 and CT class on all the work you hand ln, including the Cover Page.
Wdte in dark blue or black pen on both sUes of the paper.
You may use a sofr pencil for any diagrams or graphs.
Do not use staples, paper clips, h[hlighters, glue or conedion flukJ.
Do not write anything on the List of Formula (MF15).

Answer all the questions.


Begin each ans,wer on a fresh page of paper.
Give non-exact numerical answeG conecl to 3 signiftcant frgures, or 1 decimal place
in the case of angles in degrees, unless a different level of accuracy is specified in the
question.
You are expecied to use a graphic calculator.
Unsupported answors from a graphic calculator are allowed unless a question
spedlically states o$emise.
Where unsupported answers from a graphic calculator.are not allowed in a question,
you ar-e required to present the mathematical steps using mathematical notatlons and
not cal(illator corunands.
You are required of the need for clear presentation in your answers.

Ttie flumber of rmd<s is given ln brackets I I at the end of each question or part
question. At the end of the examlnation, placq the cornpleted cover page on top of
your answer scripb and fasten all your wolk searrely logether with the string provided.

This question paper consists of 6 printed pages.

O 0740n o7 ffum over


`MATH 2007
2

Sectlon A: Pure Mathema0cs [40 marksl

Booboo bird is an endangered species of birds on cook's Island, The omithologist


found that the populaion of the Booboo birds is decreasing at a rate inversely
proportional to the number of birds and decidcs to bring in a fxed number of lhe
bo6Uoo Uirat yearly. fu time , years aftcr Ycar'Z0m, x is the number of Booboo birds
on lhe island. It is also krown that the number of bir& remain constant *hen x = 20 .

Show thar Id,= f l.,\ - A]


x/
whcre * is a positivc constanl l2t

Thc omiErologist found l(X and l2l Booboo birds on Cook's Island in the Year 2005
and 2(X)6 rcspcctivcty. Usc thc above modcl to predict the number of Booboo birds on
thc island in Year 200?.
Statc an assrtrnption madc in your celculation. tsl

The equation E is giveo by za +az2 +b=.O where a and , are constants.

(a) Solve E when a=0 and 6=l-,J3, giving your answers exactly, in the form
fe.
The points A, B, C urd D rpresent the roots of the equation E on an Argand
diagram. By corsidcring tfte modutus and argument of each of the roots found,
. identiS the shapc ofthe
quadrilaterd ABCD . Justify your answer.
t5l

(b) Find the rangc ofvalues of 6 such that all dre roots to E are rcal when a = -2 . [3]

(a) Find
4

TdL
=;::

(b) Given that o>0, find answer in termsof a. t31


J" lr-"'l*,givingyour

(rln.r)2 dx, giving your answer in an exact form.


[4

OHO MATH a"7 7


3

4. The points'l andB have position vectoo 3l + j and 3i + 3J respcctively. Thc line I, and
the planes fll and [I2 have equations as follows:

(2\ tt': x+22=3' r'l r0)


/r: r=
".1-',,l ' "tt=^[l,l--[l,,|'
where tr, l. and f 1R.

It is given that the planes fI, ard n2 ht {


rscct in thc linc .

(l) Find a vecor cquation of thc line /, and show that tlrc linc I, is parallel to the line /, .

Hence, find thc shortGt distancs betwecn drc lin6 4r ad ,r. [5]

(1)The pOints C and D arc on c lines t and pective suCh OC=90 and
BDg a pardLIogram Find thc pOsmiOn v tors ofdle poins C and D. 141

(1)1 e plane n3 parallei to the plane 2 and equidtantto both point and ule
plane n2 Sh W thtt thc cqm otl ofthe plane 3 given by ro(1-j+k)=1
Find the posidon vector ofthe foot ofthe perpendicular from the point/to dlc planc
3 [5

Sectlon B: Statistics [60 martst

Tlree single men, two single women and two families take their places at a round table.
Each ofthe two families consists of nro parents and one child.

Find the number ofpossible se*ing arrangements if


(i) members of the same family are seated together and the two single women are
separated, t31

(iD the seats are numbered and each child sis between thcir parents. l2l

A bag nns lo orange avollret i4 strawbq aav4Dured and 16 chc


navoured sw ts which ate ofidentical shapcs and sizes.


Ben y sciects a sweet at random ttam E bag.Ifit is not cheny navoured he


and selects another sweet at mndom.Hc repcats thc process unti:hc o ns

y

navOured sw t Caiculate hc probability that

(i) the first swet selected is strawberry-flavoured and the fourth sweet is orange-
flavoured, tzl
(ii) he selects an even number ofswee6. t3I

OHO MATH 2007 97 7 ffum over


.4
7. A random vaiable ,Y is rrcrmatly dffibuted with meot p and variance 2.5. tt is given

that P(x>13)=0.2s.

(i) Find thc value of p, giving pur arswer to 3 signilicant figures. I3I

Another random variable- I/ is.also normally disributed widr known mean and vriance'

60 A stdc[lt ftid thc fbability thc 3 indcpcndent obscrvlioos of X


was rsked to
difFer ftbm 3 timcs m obscrvairn of I by at lcast l. State an nsrmptim dnt the
strdcnt hr to make to solvc ltc $estion. .

Part of ttlc studcnt's solutiqr to thh quction is givan r follous:


3x -3r - rv(3E(x) -3E (r), evar(x) -evr(r))
Pl -3y 1)=
Point out 3 mistakes that the student has made. 131

The owner restaurant colmts th nurnber of banquets received by the reStaurant at the
ofa
ind ofeach week. State a condition under which d Poisson distibution model would be
suitable. lU
Assuming a PoiSson disfibution, thc pmbabitity that tlre rcstaurant rcceives at least two
'in
L*quit a randonrly chosen week is 0.3. Find tlre mean number of banq,as received- -
perweek. t31

A week is considered busy if tlre restauranl receives at leat two wedding banquets in the
week.

(D Calculate the probability that in a period of 4 comecutive weeks, thcrc is at most


one busy weeli. l2l

(ii) Hence calcutate the probability that in a pcriod of 6 consecutive week5, the 66 week
is the second busy week, given that thcrc is at most one busy wcek in thc firs
four^-
wecks. 131

O HC: MATH 2007 97-7


n clbtt ulluT 2007h 2 hlls
(

5

The deparhrcnt managers of a company are to study thc number of days of sick tearre
taken by ernployees under droir charge. The oompany coosists of 2 deparhents namely
thc Sales Departsnent and ttrc Finarx* Dcpaftnent The nunber of employec in the 2
dpartrnents and tho rcspectivc probabilities that thc employecs would take sick leave in a
montlr are givcn in thc table below:

Department Numbcr Probab ity


Sales 55 0.2
FinalB 45 0.3

(a) Using a suitable approximuioq furd the probdility thd therc are at lcast 15 pcople
in ltc company taking sic& leave in a pcriod ofolrc non0r. 13]

ne nulnber ofdays ofsict icave per mondl taken by an accomts clerk fo:lows a Poisson
d tributioo with nlean 12.

( USing a suitabic approximation, nd the maximuln nulnber oF days ,h apc od


of oncthc aCCOunts dcrk can take so that the probabi:ity matthe number of
days oFsick lcave is icss than wi::not excecd O.7. 14]

10. Bcn,a fmit sc:lcr,oniy sells tnangoes from o ormrds,A and B.In cach oFChard ule
weigh of dle mangocs are norlnJ:yd tributed with a standard deviatlon 39g in A and
an unknowi standard dcviatlon in B.The mcan weigll are 200g:n A and160g m B,

( )OnC L And a business parmcr of BcL brought h a largc batch of II angocs


mm Orchard 10 mangocs were randolnly sddcd m mis batch and the
wcigh (n9ofulc ntangOcs we as fo ows:

175 182 167 160 169 189 137 164 207

1f a at 2 5%sittd6cance icvel indicates thcrc a dccreasc in dle mean wcight


of dle mangocs from orchard flnd ule g valuc of x, rect to l dccimal
place 141

(b)On mOther day,Bcn exolnlned 20 mangoes lom orchd B which have nun=
wcight 140g and a standard devi 453 He was worried dlat tt mean wei t of
manpes in orchard B might havc changed.Andy helped him by ng out a
stat cal testtO flnd out mdl

(1) State the nui:and altemat c hypoulescs in ca_ing out such a test l

OD Andy has to rcicctthe nd hypoth at l%icvel oFsipincance.0 ctll


the minirllm valuc of l,dcnoted by ,00mctto 2 dccimJ p:" in
order to arrive at this dccision, [2J

(ili) Ifa mango m orchard B which wcitts 140g added to the samplc of20.
Would this afFect Andy's nclusion for thc test at t%ieVe1 0f
gnilcance? 12]

O HC: MATH 2007 974 07


7
ffum over

Considq thc de set and its omesponding scatrer diagram as output from a GC:

x: v L3 2

e
s-EE'
-lrtl
5-21,,
,lr.0z' .
.

E',

,E I

(r) Give a rcron why it might bc unwise to use cither of the regression lines
y on .r
or r on y to estimatc tlr valrrc of y whcn r=1.5. tU

(b) Thc cquation of thc rcgression linc of y oo : is /=l'069x+l'0E3' Find thc


obscrvedvalrrcofywtrco,=|.S2,andhenccdeducethelinear.(productmoment)
conclatioo cdefficien( r, bctwecn r 8nd /. t31

Adding more points (a y) to Ot autu may chango the value of r ' Give an example
ofa pair ofpoinB that would not carso such a change' Ill
Find rhe equation of the rcgression line of (x-t.108)' on y. Use this equation
(c) to

estimare dre vatu{s) of x wlren y=3. I31

Can we use thc same cquation to estimate the value of y when x = 3? tll

th3 Si1eaq.o1
12. In order to find out how much ouriss spcnd during tlrcir stay in SinBSnore,
TourismBoard(STB)interviewedeverytenthtouristleavingtheairport,simmlgratlon
departure checlqoint to obtain a sample of250 responses'

(a) Give two reasors why the sample obtained may not be representative' tzt

(b) De.scribe how STB may, ar the airport's,immigration departure


checkpoint' obtain a.-
sampre of zso tourlss via a luoa ia*pling mihoa. tU

A random, samplc of 250 tourists ws intcrviewed and their expendiure' in $x


' is

surnmarised as follows:

f,(r-160)=l2so md f,(r-165f =2813?0

(i) Calculate the unbiased cstimdcs of the population mean p and variance o'' I2l

STB bcliwes ttur eaclr tourist spads $ 170 on average

(ii) Estimatc the probability .tlrat another random samPle of 250 tourists will have a
sampte mean ixpendinnc exoecding that found in (i). I2l

(iii) Find the lcct numbcr of additional touriss sTB should sample so that the sample
' ' ;; expcndiore of lhc enlargcd sarnple excceds the mean found in (i) with a
p.U$ifi''tyhi$"tthan0.99.
- t3l

O HCl&MATH 2007 07,(Xr2nvo7


2007 HCI Prgliminary Examination, H2 Mathomatics Paper I
l. Let P, be the statement "r, ={I-JI
n
YneZ"'

4: LHS= :=l RHS=9=l=LHs


I
Thereforc,Pl tuc.

Assume P, is tsuc, thal is, ,, = E# -


Jor some k e Z'
SbW 4
J

LHS= 1.= cen)
-1)
= .
+1 1
-1)
==
1+1
!

+1)
i

Pl mc,PI tne=,P.J also me,by mattcmaticalinductioL mc


cZ

2(b) The 8raph of y = t1 (.r) is shown below.

Sketch ttrc graph ot , =


3

2007 HCI H2 Mathcoratics Paper I


5- aG +x= (2-G)' +t > O for anY.r> 0
x(5-aG+x)
(x- a\(x- b) =O
Since -4rE+.r > 0, 0
)( b)
5

xS0 or a<x<b

But G exist only when x> 0.

Therefore, x=0 or acr<6 (ans)

4
L 8r=8F R" =8-lr
l=
2
=2
(8-l )=16 ::


=:6

whcn 14_ ,16 =0 r=16 = =:


2007 HCt H2 Mahcmarics PaPcr I


1

=
:

0

Thus,/is maximum when r=:.

Maxhu = Rm'
(1) (1)2=4


31 : Largest valuc ofr=

Deducc =costt oR I
3 3 4 =1 =: or 2 12

Lct P represent compicx nulnbcrz:

s =i and sin = =7F=



4 3 4 3

- 7r+10 A B C 5 3 2
r(r +l){r+2) r r+l r+2 r r*l r*2

2007 HCI H2 Mdhemarics Papcr I


5 3 2
+
1 2 3
5 3 2
+, 5 7
5 3 2
+
3 4 5

+: T6-[ :T+ : ]

5 3 7
7 2
+
5 3 2
+
7

n n+l n+2

+30 -3

-3
21
=3
?=,r(r +l)(r +2)

=3(o-1:6-3-;i]= =

8.
1

=
l
'

h =
=
;


; ( )'=

( )'= )


=Q = , = =

+ 1+
=
+ 2

0=

0(OTOtal number orin ges mml set to thc( -1 Sct

2Cll17 HCI H2 Mathcm ics Paper i


= l+3+3, +...+3.-2 =
3- -l
.,

H Fh
SCt +{ Jttr
(DG)2h"")*r, z(r*')-r+a, z(r*')-r+4cy,...
sct = z(r'" )- f + ft"" -
Last integcr in the z6 fltl
OR: Last intego in the rdr set = first intcger in (z + l)t - 4 = 20')- 5

= 120H l+
(ilD

)
:(3 :+3
=3 -3)
2+3" :-1)
=3 0
=3
1413 ) ll

x= et cos l Y=e'sinl


= COS sh
= Sh +a r

d/ _ sinr+cos,

dx cost -sin I
Wtren r=f, cos, -sin, =0 = tangent at t=L 1*or,

Whcn = ,,=i

Thus,le tangent to thc curvc at = iS X=i

10(lii)OP= :+: := :
:

2007 HCI H2 Mthcmatics Papcr I


Given P iics on dle lncy= and OP tt 09,
dlen O lies On dlc lincofy=`
= =COS
`:sin
tan =-1

=2

4

=` cos _

=sh =


+ = =a
:


of ZV 0(2=:( )(P)=:( 1)=:`r units2


)(


=
:`

+: =`

:F)( :)=: f units2


Area of AP` 2=:(C 2)( P)=:(

Area of R =

2 7 HCI H2 Mathemtics P per l



=+S

S tan

When
= , =O md whcn, , =:

= =Itan f=
Sec2 _1
=ftan' :
Arca ofR=N5- ( -1)=:
i
(b) VOi OfSOlid=vo1 0fcone+voi oFcy:inder voi ofsoiid rotatcd under dlc curvc

=:

(:)2:+ (:)2(1:_:) i111
=:907(3dP)

Since Chas a vertical asymptote at r =-1, thus D = l.


ay _(2x + a)(r + t) -(x' + a* +)
_ l +2r +a_4
dr (:+l)'1 (: + l)'!
Since Chas a stationary point at
' x=2,9=0
-dx *ten r=Z
4+4+a-4
----- =0=a=-4
(2+t|

(iD -n-12 -4x+4 - r-5+ 9


r+l r+l
The obliquc asymptote is y = r-5.
x2 -4x+4
-r+ I
t(x+l)= 72 -4'*4
+ y = x1 -4x+ 4

"x +(4-y)x+4-y=0
xt
The equation above hm no real roots when discriminant < 0.

2007 HCI H2 Malhcmatics Paper I


(+-y)'-40X4-r,)<o
y2 +8y+16-16+4y <O
y2 +l2y <0
y(y+12)<o
-12<y<0
So C cannot lie bctrvecn .12 and 0.
(iV)

1" Y-'-s
(o,4)
{t'



(v) (r-,')(,+r)'=(,'-r.r+r)'-(-*"; =4-x2=v2 =4-x2

Need to sketch y'+x'=4

Since there are 2 intersections between the graphs,

(a-:')(r+l)' =(r'-lx+4)'has 2 real roos.

2007 HCI I{2 Mnrhernrti6 PaP.r I


2007 HCII12 MadIPAPER 2 SDg 10N A(PURE NIATD

1.F m quc on, =ratc of po


rate Of" =1-: ,ulc"

When =20, = ._ =0 =201

=1-22 = (1_21)
Thus, (ShOWn)

Solving DE: L=
I: =

(Itf:)
+201nl,-2q=

1+ =
`
When =104, =5,104+201n84=51+c (1)
Whcn =121, =6,121+20in 101=61+` (2)
(2) (1)E =20.6861 20.7 and flnally 89.2
When =7,solve
+20inl -2q=71+c=233 988 for 139(Ushg Go
Possibic Assumptions:
Numbcr of B B birds found in thc isiand in 2 5 and 21106 is ciosc or
cqual to thc actual number oF B Boo birds on dlc isiand
Therc arc no cxtcmal factors like human intcrferen on dlc is:and h 2007
that may afFect ttc popultton in a very dittrcnt way utat ls not within thc

2( z`+1-j =o im i Z `=-1+` =


Thus z4=2`( )and z=z =2:`(: :) , =0, 1,2,3

Suppose/=z , =zt, =z2and D=z, /




Obscrvc that Lcntt =OB=OC=OD



_

Aiso,the argumcnts ofule r00ts difFer by 90


thc diagonals intcmt at 90


And dedu dlat/aCD fon ls a square: `

D
J22_
2(b) Sub =-2 hOcq on d get 22=2 Jl b.


:
T
J I =

F ,we sOIVC for 4 z= lttJl .


=l
For rcal z,1 VI mustbe 0
Ol l
Since l+Jl b is dways>O for b 1,we need
worry about solving for D when I - JiJ > 0 (see UJng graphs,Vl 1

diagram). Thus 0 s b sl- Q) 0 l


Combine (l) AND (2)
= 0<6<l (ans)
b) ALIER}IAIT'G SOUMOil

Sketch the graph of y -- zn -22 .

Tuming points as intersection points or rools are


in the form ofa repeated rool

If all roos are rcal of z' -2i + b = 0 sre terll,


the gnph y-- z' -22 and thc linc .y = -A must
have 4 intcrscctioo points (taking into account
the repeared root).
+ -15-&50 0<D<l (ans)
=



f =:

3(
T : ITT
=:hprtX ll
= =7

=:h +1_: tmJ



2+ +

=:h + +1- tan +



2_

3(b) l
2_ 2) + 2_ 2)
= :_(
(


= a2
11: I+[; ]l


=
3

30 h 2
i '(n
2

=[
;(in

)2]_ 1 2(in ):`

= i
2(in

= :I[
(in,)] il }=;(5`'




To show the two lines arc parallel:
Method l: Point out 0lat tlrc dir vector of I, is scatar multiple of
Method 2: Compute 4 x4 which is // /, and is scalar multiplc


4(ii) Let( ]= +




lilfOrsome


cR
e get =2and O =111(ans)



= , =111 (
'hce CD
111FIII
4(iii) observe rhar o ries on tlr. Midpoint of oA whichk(irz, l/2,0) &en rieson Hr.

c'mpu'le
"
=[j].[i]=[;'] *'' =ff] = "1,'; =',"'-",
[,'] [,')
L.et Fb foot of perpendiodar from ,4 !o fl, and pick a point E ( l, 0, 0) on fl, .

*=[-+[,,]]+[lJ= +[j -- oF =oi+,,e =,[_i] *,,


]
ALTERNATTVE SOLUTIOI{

Note equation of line AF is given by r= + -t n, DF.


l. suu into to get

1
2 7 HCIH2 Mam PAPER 2 SEC110N B
S1) fating eactr family as a unit, we find that there are (5 - l)! ways to arrange 2
families and 3 siirgle men at a round table.
Nex! the tkcc membets of each funily ccn artange among themselves in 3l ways.
Since the two singtc womcn are sc?aratcd, they have 'P, ways o take tlrcir places.

Total number ofways =(s-t)txllx3lxtPr= 17280 (ans)

s(ir) Similarly thc ll persons can takc thcir phces in = (7 - l)tx 2l x 2lx I t = 31680(ars)

Rcqu"d p babil = 02 =
Q 6

l)Requ d probttn =4X:)+4)' +4)' +


= l

tt 9

70 Given O,25),P( >1 =0_25-(1)


Th implies P( 13)=0.75.

Standardizing:P( =0 75 and fr N
"(075)=067449
1) :

Thc b
=0679 md =119( 3suncanti (allS)

ALTERNATIVE SOLUT:ON to solving(1)


UJngGC cCraphiCal Approac :SketCh Yl=Nonndcdf(13,E99, , .5)-0.25
and flnd the x inter ptto get x=119.

7 ) Assumplom and r are independent random variables.

Mi es ItShould be :+ 2+ inStead OF 3
/ar( l+ ,+3 37)=3/ar( )+9/ar(y)
) We should bc finding P(X' + X, + X, -34 > l)

2007 CI 2 N
2 Sol 6

8. Some possiblc conditbns(any Onc ofbelow for answering qucstioo:


Mcan number ofbanquets propottonatc to thc numbcr ofwceks
Banqucts at ulc rcstaurant occur at random
Each banquct is indcpcndcnt oFcach other
o Receiving banquc"iS a rarc event
P(X 2)=0.3 l,P(X") P(X=1)=03 P(X )+P(X=1)=07

(1+ )=07 - )

=1.10(anS)
By GC:Graph (mustShOW graphsD Equalion So:verlgiVe kcystrokcs

8(i) Lct B( 0.3)L ule humbcr ofbusy w bo of"wec ,

P(L 1)=binOmcdf ,0.3,1)=0.652(a )

8(i D P (6th weck is 2nd brsy week I at most I brsy wcek in tirst 4 weeks)
P(t weck busy in lst 5 week & 6th wcek busy
P nlpSt l WCek busy h 4 wce

=P(Ys=ll(Q31=5(0.3)(07)4(03)=a166.cans)
06517 0

ALIER}IATTVE SOLUTION
P(6th week is 2nd busy week I at most I busy week in first 4 weeks)

_ P(wk l- 4 rct busy, wk 5 & 6 busy)+P(t wk busy on first 4 wks, busy on wk6)
P(at most t wk tusy in first 4 wk) |
P(Y. = 0x0.3)'z+.1(Y. = lxo.7xo 3)
- 0.65 r7
= 0. 166 . (ans)

9(a) Lct (55,Oa and B(45,0.3)


Thcn (11,8.D apprOXimatcly and r (135,9.45)apprOXimatcly
Thus +r (24.5,1825)apprOXimatciy.
Required probab ity=R + l =0987(to 3 sFw/o ccp(ans)
P( + 1 P( + >14 =0.990(t03 sfw )( )
ALTERNATIVE SOLUT10N
La r
(55+45, 1)Then ttproximatc r (245,18.4975)

Requircd probabnity=P(r 15) Pc>14.5)=0:"O Ct0 3 so(a )(MUST CC

b)bt (14.4 dcIIOtC dle no.ofdays ofsick:cavc per month the derk w i takc.
Sincc >10,7- (14.4,14.4)apprOXimately
Then P( ) 07 P( +05) 07(do cc fOr using nollllal disttibution)
From CC,P( 16.38996)=07
Thercfore +0.5 1638996 =o :588996
largcst vdue of Is 15(alls)
( tsso+x-r*) I

l0(a) To indicate decrease in mean weight, ,l , '-*1;5- '


o'ozs '
|
[.r
1550+ x
- r'o
And we have l0 . -1.95 [GC: invNorm('025) = -l'96]
39/Jl0
Solving for x: t<208.28 and thus greatest x = 208.2' (l dp) (ans)

100)
(i) Ho: [r = i60 vs Hr: ]r * 160

(ir) Use GC, pvalue = 0.06772 [using two-tliled rtest wift ,." f =
20-](lS' )t
To reject Hq k%>O.06772 and thus (o = 6'78 ' (2 dp) (ans)

(iii) Addition of the special mango with weight l40g means


(D New sample mean remains unchanged '
(lD Newu.e sr* =#E(x--J =452 <s!'
GC to find
This mears new p-value is smalkr than previous p-value [QB input into
out the new p'Yalue = 0.0551 < 0.067721
This means Andy's conclusion: "still rejed f'lo" (ans)

ll(a)Thedatapointsonthe'scaserdiagramclearlydonotshowalinerform(theygivea
pott".l. Thus using either regrcssion line would not be suitable'
"orrifin".
ll(b) ComPute r=-9.62 and Y=-6
19.78+ Y
.-_,
Sub them into v=1.069.x+l'083 since [,i) must tie on the regression line'
Thus, / = -3.00 (3 s0 (ans)
will only Eet us the Predicted y value'
tsub r=1.52 directly into /=l'069'r+l'083
lie on this
i r.rOf m obcerved y value. Note also 0.Sz' - l'Oo) may or may not
I regrcssion line.l
| ii&the;rtu irio CC and using LinReg(ax+b) Ll, L2: r =0'392 (3 sf) (ans)
I

in r ' (ans)
I . poiilE, r)= (1.603 ,2.797\ willnot
eauing tt cause a change

poins on rhe regrcssion linc other oan [' y) wil still carse r io change]
I f*"n iat
I
I t t(c) Ll = x,12 = Y,L3 = (x -
Using CC, with 1. 108)^2, and LinReg(ax + b) L2' LJ:

| 1r-l.ro8)2 =o.66zy+t.lrt -_(r;(ars)


I SuU r=3 into (I), -r=3'03 or 0.817 (3 s0 (two ars)

I
this line to
I Sinr" r = 0.985 is close to I and x = 3 is within data range, we can use

I estimate y .
l2(a) Some possible reasons (choose any two blow for answering question)
.
Method leaves out tourists who learrc the counuy via other foms of transporr like
cars or fenies,
.
PossibiliU of cyclic pattern if thcrc is a clustcring of passengers from a porticular
country (e& tour groupd confercnccr' seasooJ holi&ys)
o
Other Cyclic Pancrns: orrer-reprcseotatira ofone geada/ nationslity/ sgc groups
.
Tourisls may continue spending at airyort rermiml rftr being sun cycd.
.
Some tourists may be oo holiday with otbcr funily membcrs so the ryooding they
repon may be higher than individrul tavclcrs
.
SingaporeanV childrcn may be sampled

l2(b) Quota should be presented in categorical form under the catcgodes


Nationality, Gender, Age Gtoup, Duration ofstay etc.
Arswcr shoutd include crcsing at least two categories such as [nationality vs gendcr]
or [gender vs age group vs durarion ofstay].
Answer should also includc mentioning total sample size:250.

r2(D tr.E of p,, ; = I(::teo) + roo =#+ 16o = 165. (ans)


281,3]0
o,
24gu' -,
", -1-y1r-165f
u.E or 1, = = = I130. (ars)
249


l2(ii)

Sincc n=25O larBe, so by CLT, =452)apprOX



r(x > tos) = normatcdf(16s, rEs9, tlo, Jlsz)= o.s9r. (ans)

l2(iii) Let the sample size needed be n . Since 0.999 > 0.991, so z > 250, and by CLT,
x - N(ruo, !-!19) unp,o*.

set P(X > 155) > 0.99e


-----(.)
=
f ) = ff.,n*orm(o.oot)=-r.oe62
{,.iii-'21..-
= Ji rZO.'tB + n>431.6*432
We should increase tfte sample size by at least 432-2SO =lBZ (arc)

Note: GC method to solve (r)


(l) Graphical: Plot a glaph such as Yr = normalcd(165, lE99, tlO, X- ana Jitly l
Yz= 0.999 and find intersection point bctween thc two graphs, thcn deduce the
inequality. OR
(ll) Use Equation Solver: 0 = normatcdf 65, I 899, I 7 O, \tt :^]lOI X-) - O.gg9 .
0

Anderson Junior College


Preliminary Exam inati,on 2007
H2 illathem atics Paper 1

Expand
^1* in ascending powers ofx up to and including the term in l, stating the
Q-,f
range ofx for which the expansion is valid. Find the coefticient of/ in dre expansion,
simplifuing your answer. 14]

G nthat y=cCOS
2,show dlat

2)(
)2=4
_
O By Cr difFe ndttbn of c above"suL nd the Madaudn'ssel,9fyup tO
and including the tcnln in x2. [31
(D Dcducc dle equation ofttc tangent to thc curvc ofy=( s lX)2 atthC point where
X=0 [11

3( nd ttCCtttVducof[Jn nJ 13]

(b)Find j:
7:

dX by using the substitution
=

[3



itF
)
t'
0 The points A and B have coordinates (2,-l,3) and (4,13,_3) respectively. Thc linc
/, passes through the point I and is parallel to the line /, with cartesian oquatiOns

?=T''=-' '{'11u, "2:Zi" tl)'(i")+i1g;


tlI
the vector equation of
t41

ftO r\
. cos lx-

5 The functions fand g are defined by Y . cos\r-\


f :xr+cosl-r-ll, xeil , l-zsxsl, cos''
X --
1= 1- 1

lqE-l1tr
I +.r
gtrr+ :e!, -l<_x<l_ !'(r)'rts-lx{r
1_r,

Define, in a similar form, the inverse function f -r


Explain why fg does not exist. Find the maximal domain of g for fg to exist.

Page 2
6 (1)Findthecxactvaiucof l [3]
T

The da m below shows dlc graph of y= f rO L Rectangles ofcqual


2-
width are drawn as shown in the interval be v nx=O and =1.
Show that the total ofa the recangles is givcn by

[ +




Deduce thc valuc of lim s.

lnx
'lr- (x -r)ln2
(a) The shaded region.,{ is bounded by the curve y= and the line 4y -- ,

of/.

as shown in the diagram. Find the exact area

/ =
h2

Page 3
(b) The diagram shows the finite region R bounded by the curves /'1 = 16 - I 3.r and
y = (x + 2)2 . Find the volume generated when R is rotated completely about the
y-axis, giving your arswer correct to 3 significant figures.
41

2
16-13

y=(x+2)2

A cheetah starts its chase of a deer with a leap of 4 m. Each subsequent leap of the

cheetah is shorter than the preceding leap by l0 cm. The deer is 2l]m away from the
5-
cheetah at the start ofthe chase and runs away with uniform leaps of2 m each, taking 5
leaps to every 6 leaps by the cheetah. Both the cheetah and the der start running at the
same moment.
(i) Show that after the cheetah has taken z leaps, the deer will be at a distance
(rr1. 1"\ from the cheetah's starting point. lll
I s 3)
(ii) Find the least number of leaps the cheetah needs to take to catch the deer. t4I
(iii) Find the shortest dislance (to the nearest tenth ofa metre) that the deer must be from
the cheetah initially, in order to survive the chase. [4]

ftf (z+i)' =2b+i.


@ Solve the equation

(b) (i) Write down the 5 roots of the equation ,t - I :0 in the form rei'a, where r> 0
and -r<O<tr. t2I
(ii) Show that the roots ofthe equation (5+z)5 -(5-z)5 =0 can be written in the
ktr
form 5i tan J,5 where /r = 0, + l, + 2 . t41

Page 4
l0 The curve C has equation, = p + 6, q + -g .
" :r:,-,,
(i) Determine the equations of the asymptotes of C. [3]

(ii) Show that if C has 2 turning points, then q <9 -3p .


[31

(iii) Skerch C for the case p S2whenq= l, showing clearly where the asymptotes
ofthe graph cuttheaxes. t2l
By considering the graph in (iii) and the graph of y=! for suitable values of the

constants t and rr, show that the equation .r4 + 2x3 -"2 -2*-6=O has exactly 2 real
roots when p:2
and q=1. I2l

ll (a) The Fibonacci seqrrcnc {4} satisfies the recurrence relation


u,*2 = u,+1+ u, with zo = [ s1d s, = | .

Another sequence {v.} is defined by v, =L:! ,r>I.


./,

Show that the recurrence relation for the sequence {v,} is v,., =t+]. tll

vr
Given that the sequence {v, } tends to a finite limit t as r tends to to, find the
exact value oft t31
(b) The sequence u1,u2,u3,..-..,nr*.... is defined ty u,=fr,, n>3 urd u,=1, 1tr=1.
i=l
(i) Write down the terms u,z.,z, and uu. tll
(ii) Make a conjecture for a formula for z, in terms of z for n22. tl]
(iii) Prove, by induction, your formula for 2,. I4l

12 )By uSing the sub"itution =x ,sO!Ve the difFcrcnthi equation

+ +
= 2exp gttemsJ
21

(b) A certain drug is being administered to a patient in a hospital at a constant rate R mg


per minute. The rare at which the drug is lost from the patient's body is proportional
to the amount .t (mg) of the drug present in his body at time, ( mins ). Given that
the amount of drug in the patient remains constant at the instant when it is l.5R mg,

show that mc dfFe ndd equatbn ch ng,and


ven by =R
:

l]

( IfhiS bOdy is inidally ofthc drug,lnd in te Hs Of .


[3]
(ii) When , ---+ co, r...+ a .Find a and explain the signilicance of this result. t2l ^

Page 5

13(o ShoW thatthe dittmcc Z bcmocn any point( 2and thc flxed
=
l(18, 0) I "on dle curve
point satisfies the equation
= ./xa + (18 - x)'? . tll
Hence, or otherwise, find the point on the curve that is closest to the point.,4. t4I
(b) Water is poured at a constant rate of 20 cm3 per second into a cup which is shaped
like a truncated cone as shown in the figure. The upper and lower radii of the cup
are 4 cm and 2 cm respectively. The height ofthe cup is 6 cm.

(i) Show that the volume ofwater inside the cup,


i
,, is related to the height of the water level,
through the equation

v -!-(h+6)3
27' -Bn. t3I

(ii) How fast will the water level be rising


when ft is 3cm? Express your answer
in exact form.

END OF PAPER

Page 6

2007H2Mathematics Preiim I solutions


Solution
1,
1

=4(2 3)(1_:)3


= :(1-3(



:)+ [ ( :)2

:+:
+:




+21)



(-3)(-4
Coefficient ofr"

(:)[ :)"]

(:)(-1'hT (:)
+2(F+1)
+2)
1:)
2.
,: lcos
-rr),

4:2*.'''f-
dx ' l
I Jr-,'J
(^[l-,')4-=-t*;'.
ft-*\*)' =+("o.-' ,Y =4v (p,ou"a)

Differentiating wrt x ,

t rh(*)(#)-*(*)'=^*
t_.\(*)_.(*)=,
whenr:0,y=(cos-r012= -2 dv: -" d2,
;, ; ; =z;
l

i!
By Maclaurin's Theorem , y : ( -nr**'*....

At x = 0, equation of langent to the cuwe is


o2
'4
3a) 0

a) hJ = Sin ) +1 Sin (Sin,)`


I[JnX ISin
4
4
Solution

+
J
4



b) l l
=
2


=

=

0+C=
n (


+
4.(i)

DircctiOn vcCtOr of l iS I::


`+

EquatiOn of l iS[=:ll+ cD

l::

=lil & =
i#

)28+4 +30+25 =0
=)29 =-58
=-2

Equation of line BN :


Solution
5ti) y = cosl.r-ll
cos-ry=lx-tl
x=llcos-l/
Since l-z<r ( l, 'r=l-cos ty

f-t:xt-+ l-cos-l -r, xeD, -l<-r<l

=[0, )
CD/=[1- 11

/g dOCS nOt exist.

Forj& to exist,
Rr =[0,1]- maximal D" =[-1,0]
6(i)

: 1

7
=

2- ):+2(2- )J]
ll
0

lim
)
Solution
Altermat e :ution for o by pal

f &=[[-2
f2V'
]1 2-
`

=
= [-2x 2-x+2[
:(2-x ]]

= [-2 J2-x ):]]


[:(2-

= :(4 -5)

7a)

Area =



=[
II
= _ in 2
2 8
7b) Points ofintersection ofcurvcs arc(-5,9)and(0,4)
Volume

=
_ _
!(-2+j
)2 )2
I lF/1)2

=46652653-83775593-107.66306
=3504859107 350
8i)
Ancr icapS Ofthe chcctah, thc decr would havc leapcd: 2=:

Thcrcforc thc dcer is at a distance(21:+: tah's starting point


)from thC Ch
i
Dimnce leapcd by chcctah: =4, = 10

Ancr leapS,the dis&mcc lcapcd by thc chcctah= =:[8-


}(
lll

To catch thc d r,Sc 21:+:

-lll 21:+:
[8-

4 -1)
+:"

2+3
240 -3 1284+100
2_143 +1284 0
3

13
-107X -12) 0=o12 35:
Lcast numbcr ofleaps=12
4
Solution


For tte d o su i the chasc, r tt n values,
+

:[8- }
lll +:

240 -3
2+3
6111+100
2_143
3 + >0
Discriminant 0
1432_72 0

=o >28.401m
icast distan =28.5m
9a) iet z= +yl
(X+ +f) =2((X+J )+f
= +J(2x+1)
r ( +1)
,=-2y (1)

+1)=2x+l
Sub(I)intO(2)
l=2(-2y)+l

=:: and =_:

9b. -2L'
zt-l=o=r=i '
2,

.7. .ar

5, 5, 5

)z=e

ii)(z+5)S (z-5)S=0

=)(:
6)5=1

: = ,k=QJ, 2 (frOm(1))

(1-
(51z)=(5-2)

)
l)

,_51`(
Z=

:I

)( ( )
(11))
_5`(


Solution




(p Vd)



'tan


101)
y= 111:F a= + -3+2F::llE Asymptotes:y=x+ p-3, x = -3

=1_lFl:

For =0,

(
+3)2=9_`_3
x= -3tJ9-q-3p
For 2 tuming points, 9 -q -3p >0
+g<9-3p (shown)

y=x+P-3
//


3-

Whcn =2,4=1,y=
x2 +2r-l
4+2 3_ 2_2 -6=0

,2(
2+2x-1)=2( +3)

x2
1= (1)

2h rsection poh bemccn c& = 2 real roo oWn)


1la) 2=
+:+

+2=1+

+l
+:

J=1+

As ,4 and v,*, -+ t
=1+

Solution

2= +l

2_ _1=0

=
2
Since >O for all l

4=
>O for all l
r

= (ansp

1lb) (i) z, =1
uz=l
ur=ur*ur=2
ua=ur+u2+ur=4
us=q+u2+uj+u4=8
U6 = ut + U2+u3+Un *u, =l$

l
(rl) u"=2*2, n>2

(111) Let n=2, LHS= z, =1


RHS= 20=l
Therefore the result is true for n 2. :
Assume that the result is true for n = 19 k >z
t_l
i.e. u, =\u, =2'1, k>2
, k_t
For n=,t+1, ur*r=Zu, =lu, + u,
i=l ,=l
_)L-2 L1e_2 _ 1 aL_2

- "r-l -.r(r.lF2
Therefore the result is true for n = k +l .

Hence by induction, the result is true for dl ne Z, n> 2 .

12a)
a) :[+ 1+( ] s2x=sin2 x
Using = ,

=1- =l )

+[1+ 2 1cos =Sh2x


= [1+ 2][cos' ]+1_sin2 x

21
II cos2.[2+
Solutaon


d' = :
ll+CoS2 )a"

n2
+tan +=:(X+ )+C
tan 12 =:( +:Sin 2r) C
7
y= V2tan(13(X+ :Sin 2x)+V5C)

=R
12b)
ap ve conmnt
,





At,=15R, =0 =)R


:RI=0
Thus, =R
:X(ShOWn)



inlR +
: l=

inl R
: l= : : C
R = ]

:
:
X= /
(R )

At = , = , 0=(R /)iC/=R
=
(1-` )

)As ,` :` 0
x R = R

ie regardless oftime, the amount of drug in the patient's body will n"u". 1R'
"*"."d 2
13a) =
2+(18-x)2

o
+(lE-r)2

4+(18-
= )2

3+2(18-


=4 x-1)
Solution
At nlin pt =0
_4x3=36-2

2x'+ -18=0
From GC, =2 is the only solution
Thcrcforc the poht (2,4)

=2+, >0

x=2 ,# 0 Mh pOhL

13b) i)

2= x=6
4 6+
2==2+
6+ 3

4= +6)2
(

180
=
(20) ( +6)2

=3 ,
`

= = /S

Anderson Junior College
Preliminary Examanation 2007
H2 Mathematics Paper 2

Section A: Pure Mathematics (40 marks)

la) Differentiarc tan-(ln r') r.


with respct to tzl
b) The curve Chas parametric equations ,=?+l', /=12-r+l,whererisanon-zero
t
parameter.

O Show that the gradient of the curve at any point (-ry) satisfies the equation
(2r-1)r2

=2'-2 E21

(iD The linex:p is a tangent to the curve C. By using the result in (i), find the exact
valueofp. I2-l

2 Express(2r+3)h thc fOrln 2( +1)+/ +3(r-1),WhCrc/and B are nstan

Ushg thc method ofdifFen , nd (2 +3)2 h nns of


r [
[5] _
2"
Hcncc,or othepise, nd an cxP ssbn br
(2 +3)2 h rlns of [2]

3. [n an Argand diagram, the point I represnts the fixed complex number a, where

0 arg(a) l and
, ThC COmplcx numbcrs z and w arc such that lz-2ial=l
frvf = fw+
ial. Sketch, in a single diagram, the loci of the points representing z aod w
[31
Find
a) the minimum value of lz-wl in terms of lal, [1]

b) the range orvatues of are(j) in t"r., or-g1o1. [31

Page 2
4a) By completing the square, or otherwise, describe the geomtrical transformation by
which the curve 12 -y2 -4y-5=0 can be obtained from the curve 12-y2 =1. Pl
b) The diagram below shows the graph of y = f(r) with asymptotes y = 2 and x = 0. The
curve has turning points at (-2, 2) and (3, -2).

On scparatc diagrallns,sketch the graphs Of


2=f(_
(i) )
3]

(li) y=f'(X) [3]

Show all interceptq asymptotes and turning poins clearly on your diagrams ifthey can
be found.
i
_io, j
/6)
l"l
5. Relative to an origin O, the point,{ has position vector
- t*J
|2 lin" /,
| , th" has equation
,i'

'=[l].^[joJ',.0 andthepranerl hasCartesianequation 'ir-.,trr#' o


,1
^ ,

(i) Find the shortest distance from z{ to ihe plane fI, and determine whether I and //' \\
the origin O are on the same side oron opposite sides of fI,. t3l
(ii) tI, is the plane that passes through point I and contains rhe line /,. Find the acute
angle between the planes fI, and fI, . t4I
(iii) The plane fI, has Cartesian equation r+8y+az =6 . Find the values of a and D
ifthe planes fl,, fI, and flr intersect along a common line. t5I

Page 3


Section B:Statttics c60 marks)

F m me c fthe word LEGIMSFl nd


,XttII l

are nextto caCh OthcL :


MS
[ 1 :111[l::l llI[111: ll:
c(T'1
,
-
,r Z/1n a in un ,16%ofthe total popuhtbn are aged 60 years or more.and 18%of
t( :

klY
f r

l :l

thc total population have myoph.Furthermore,65%ofthosc aged 60 years or more havc


b h
,lTttp
n at ran
3
lT
3c2

'4is /LSS than 60 years o
, II ` and has myopit 12]
C 31=.(
fi aged 60 yearsor more, given that the person does not have myopiq [3]
" (i.iif either aged 60 years or more, or has myopiq or both.


[2]


A manufacturer produces pens in two colours, red and blue- On average, 35% ofthe pens
i l

produced are red pens.

14 fi"a the probability that in a sample of l0 pns, more than half of them are .* *tr,

(b) The manufacturer offers a special price on red pens by selling them in packs ofsix.
Pens are selected at random to fill each pack, Given that five red pens have been
selected, find the least value ofn such that the probability that at most n more pens
need to be selected to complete a pack of 6 red pens is greater than 0.98. t4]

9. An IT services company has two help-lines which receive calls independently and at
random times. The helpline manned by Alice receives calls at a rate of I call per hour
and the helpJine manned by Brenda receives calls at a rate of 3.6 calls per hour.

(a) Find the probability-that both help-lines receive a total of less than 5 calls in each of
3 successive 30-minute periods.
2

(b) I
Find the probability that Alice receives at most call in a randomly chosen 30-
minute period, given that both helplines receive a total ofless than 5 calls.
t31

(c) The helpJines are open for 8 hours a day. By using a suitable approximation, find the
probability that, in a random sample of60 days, Alice receives less than 6 calls on at
least ten but less than twenty days. 14]

Pagc 4
10. A manufacturer claims that his slimming diet helps people lose weight. A random
sample of 20 people whose original weights were 80kg took the diet for a month and their
new weights, r kg were summarized as follows:
)t,-aoy =-r 4.7, I(r-so)'? = t02.5

(a) Assume that the population variance is not known.


(i)Test whether or not the manufacturer's claim is justified ar the 57o level of
significance. You should state any assumptions that you need to make. t6I
(ii) Explain what is meant by the expression..at the 5% level of significance. in the
context of this question.
llI
(b) Suppose the population variance is now known to be l0 kg. Determine the set of
i
values of the mean weight, kg of a random sample of 20 people which will lead to
the rejection ofthe null hypothesis 5yo level ofsignificance. t}l
^tthe
If the test is now conducted at the 87o level of significance, explain briefly what can
be concluded about the manufacturer's claim if a random sample of 20 people
yielded a sample mean weight of 78 kg. tl I

I l. Large beer cans contain a volume of beer which is normally distributed with mean 500
ml and standard deviation 3.3 ml, while small beer cans contain a volume of beer which is
also normally distributed with mean 340 ml and standard deviation 2.4. The volume of
beer in any can is independent ofthe volume of boer in any other can.

(a) lf 5% of the small cans produced contain more than /c ml of beer each, find the value
oft lzl
(b) Find the probability that the volume of beer in two large cans differ by not more than
l0 ml. ti1
(c) Beer is also sold in bottles, each of which contains four times the volume of a large
can. Find the probability that a crate of six bottles contain in total more than t2M0 ml
of beer. 131

A sample of n large cans and a sample of zsm all cans are selected. If drere is a
probability ofat most 0.01 that the average volume ofbeer in a small can exceeds halfthe
average volume of beer in a large can by more than 92 ml, find the least value ofn t4]

Page 5
12(a)The mb Of"vo short qulzzes in Mathematics,,and y.of 10 students are shown in
dlc tablc:

Studetrt 1 2 3 4 5 6 7 8 9 10
Mttb for ist quiz(1) 6 5 8 8 9 6 10 4 9 6
MJb for2ndqu 8 7 8 10 l 8 10 5 8 7

(D Find the product-moment correlation coefticient between x and y and comment


on dre relationship between r and y. I21

(iD Plot the data on a scatter diagram ofy against r. 121

(iiD By using (ii), state with a reason whether or not your interpretation in (i) should
beamended. Justi$ your answer. l2'1

(b) Drums of hair shampoo are kept in storage for a number of week before being
rebottled for retail sale.
To investigate the relationship between the number of weeks(.r), 0 <r Sl5, that each
drum is kept in storage and the amount of water content loss (), ml) due to evaporation,
6 drums arc examined and the following results are obtained:

):=aE, lx'--ntz, Zy=ao& lt'1=2e5Ea, Lry=xaq


(D Find the least squares line of regression ofy on .x t3l
(ii) Esilimate to the nearest integer, the water content loss due to evaporation for a
drum kept in storage for eight weeks. tll
(iii) Explain why you would not expect to get good estimates for evaporation loss
from the line ofregression when the storage time is more than a year.
tl1

END OF PAPER

Page 6
2OOl H2 Mathematics Prelim Paper 2 solutions

No. Solutioos



la
3


1+ x3)2 ,(t + s1l" r;')
lb
4=-]*r,. 4=a-t
dtfdt
4 =,r-rr( !' ')-(z-D"
dx '\2t'-2 2t'-2 )
At intersection,
2t1 -2=0=t =L

y=?af =3
I

P=3
2 2r +3 = 2(r +l) + r -(r -t)
f1z, * tp.' = [z' 121' * j.
i r + (, - r)]
.=l .=l

= Ik' *,p*' +,2' - (, -t\z,l


,=l

= ift, *,P*, + r2, -z(r -tp,-,1


t-l

=fyg
/=l
*rS* 1(r) - 2/(r - t)l where /(r)= 2

= (0+/(l)-2/(0)
+'S
( 2/(1)
+/(4)+
c)
+.................,......:\.

n-3)-zf(n-4)

+/( )+Ak2/(
+f (n+t)+ f(nl-2 1)

= f (n + t) + 2 f (n) - f (t) - f (o) 2

=(n+t)2'*t +2n2" -2
=2'(2n+2+2n)-2
-2'(an+2)-2
=(2n+t)2"'-2

Page I
No. Solutions









+21

3a
AOB= AOD=90
OB=20A,OD=OA

IJ=IWttral

Minimum lz Wi=EC=
:
arg(:)= arg Z

Sin = =
:

arg +(: arg Z arg a+(:+:)


)

arga+: arg Z arg +i = arg -1 arg(:) arga_:

4a x2_ 2_4 5=0


x2_[
2+4y+5]=0

2_[( +2)2_4+5]=0

x2_( +2)2_1=

,2_(y (-2))2=1
A transiation of-2 units in the y dirgPtion_
4bi)

Page 2

N . Sol tio s



(2, )


[
J5

4bli)

=/'( )

5i

Distancc Of/from : =

=1
=
5il
> /md o m onttcsalne d of L

Anglc betwccn l and 2


Page 3
Solutions



",,. '.[i]=B-[iJ



Equa,.n.f
"


flf : 5x-4Y+32=15
fI2 : 2x-6y-z =6 -O
[r: x+8Y+oz=b -@
For line of intersection of fI, and fI,-O

ls -+ 3 r5l
A=t I
L24-1 6J
l-r o r 3l
rref A =l
L0 t t/2 0J
I

x+z=3 -x=3-z
y+:Z=0 =
:z

=lil+

l:
Lct =0,1 ,"O points on dle common line arc (3,0,0),and (1,-1,2)

Substitute into x+8 + =b:


3+8(0)+O =b b=3
& 1-8+2 =b= 2 =7
=10 =5
Aiternativelv

Sincc the common:ine lles in ,

i-2F-8p+2Fa=b
Page 4
N . Solutions
3-(10-2a)=b
b=3,a 5
6i


[_]I ]
Number Ofarrangcmen =5!x6cs x =7200

6ii
A letters different 7 3=210

A pa of dcndcal letters: 6q xttx2=36

Threc I's: 1

Totai numbcr ofways=210+36+1=247

7a

M(myopia)

S (60 or over)

S' (less than 60) I111


Requ cd P ba l =P(Sh )

=P(M) P(S^ )
=018-0.16 X O.65=0076
7b dh
lk [
P( ')
0.16xO.35
1-0.18
=006829_. =0.0683
7c P(S )
=P(S)+ (1 ) P(S r)
=0.16+0.18-016x065
=0236
8a lct X=numbcr ofrod Pcns in sarnpie of 10
B(10,0 35)

Rcquircd Probability=P( >5)=1-P( 5)


=0.0949
Pagc 5
N . Solutions
8b P( at nost n rnore pens) > 0.9E

(0.3s) + (0.65X0.3s) +(0.6s)'?(0.3s)+....+ (0.6s)" (0.3s) > 0.e8


o.3s(r -(0.6s)')
> o.e8
- 0.65I

(0.6s)'<l-0.e8
ln (0.02)
n>+
ln (0.6s)
z > 9.08
: l0
least z
9a :
Let A number of calls Alice receives in 30 minutes
Let B : number of calls Brenda receives in 30 minutes
AD Po(o.s) and B0 Po(l.E)
:. A+ BD Po(2.3)
required probabitity = [r(;+a< s)]'
: lr(e* o <+))'
:0.916251 = 0.769

9b required probability
: P(A<tl A+ B <5)
_ r(z = o).p(o < a <+)+r(,e = t).p(o < a < l)
P(A+ B <s)

060653x096359+030327x089129 0933
091625
9c Lct W: number of calls Alice received in one day
wt Po(8)
P(rr <6)= P(w <s)
= 0.191

Let C : number dfdays out of60 days, with less than 6 calls per day
c - 8(60,0.19124)
since np > 5 and n(l -p) >5, C0 N(11.4744,9,2t) apprcximately

P(lo<C<20)= P(9.s<C <tg.s) :0.737


10ai - _te
x= '""7 + 80 = 79.065
20

lJtoz.s (-1!:7)'
5.' = - = a.4t45 = (2. l 1530)'z
' 19' 20
1

H.:P=$Q, Ht:P<80
lfHo is true, rhe test statistics is r={#-<rel.
Jn
Page 6

N . Solutinrs
We perform a one tailed t-test at 5oZ level ofsignificence and rcject [I"
if p < 0.05.
Use cC with F=80, n=20, i =lg.Ctr,S, S, =(2.11530)

we have p:0.03138

Asp = 0.03 138 < 0.05, we rcject tL ar 57o level ofsignificance.

And conclude that therc is significant evidence that the manufaclurer's claim is justified at 5Yo level
of significance.

Assume dut dte weighs (X) follow a normal dlstribution.


10a probabilty of nduding that ale manuFamrer juslfled in his cldm when amJly he L nOt
jusdred k o o5
10b c'z is known to be lOkg,

7-.iv1ao,o.sy o, 7 :-7o -
=x.,/0.5 N1o,ty

At 5oZ level ofsignificance, reject tL ifP <0.05


i.e. r1V <i1<o.os or Pe <I!1<o.os
'
Jo.5
_n
<ll.s4 or x -8o<-1.645=i<zg.g4
UsinsGc,,
Jo's
ns i= 73.67*.78.84tg, we rejectH"* syo lcvel of significance.
Therefore H" will also be rcjeded at 8olo level ofsignificance (bigger rcjection region). Hence the
manufacturer's claim is iustified at 87o level ofsignificance-
lla LetX= volume of beer in large can
- .Y0 N(500,3.3,)
Let f = volume of beer in small can + I/ 0 tt (llO,Z.n')
P(r > r)= o.os
= P(Y .k) -o.es
From GC, k:343. 94.. : 343. 9 (or 344)
1lb x | - x z D N(500 -500,3.3, + 3.3,) x | - x
= zo,?V(0, 21,7s)
Required probabiliry = p(lX, -.Yrl< l0)
=P(-to<x,-x, <lo)
= 0.9678... = 0.968

volume ofbeer in boate = EJ il (4(500),4r(3.3, ))


Let r=4 + E, +....+ Eu s rv(6(4x500), O(+,
)tr.r, l)
= rD t/(12000,1M5.44)
P(r>r2o,+o)=o.lo8

lld

7-: + 1/ S
(340-ll,21 ) :

Pagc 7
N Solutions

P(7-: >92) 0.01

P(7-: 92) 0


P(Z
: )
0 99

2
From GC: 2.3263

ll.4765
: Ct =12

12ai Use CC, r = 0.t47 (3s.f.)

As r is small, expect r and / to be not linearly correlated

12ali

12aili As (9, l) is an ourlier (or far away from the rest ofthe data), the interpretation in (i) should be
amended.

If (9, I ) is removed, the new r : 0.823 which indicates that -r and y are linearly correlated

12bi
=8, y=68,

_

6 =
b= =!= q.sa,sa,s

)2 88
ll
)]x2_126
The estimate l Ct squares regression line ofy on is

y =b( )
=4.54545 +31.63636 =4.55 +31.6(3s.
)

12bil When =8, 68 ml


ne estimated cvaporation ioss for a drum kcptin storage For eighth weeks is 68 ml

12blil When the storage timc is more than a year,x wili be outside the rangc l 15,and hence we
would not expectto gct good estimates from the iinc oFregression For evaporation ioss.

Page 8

NATIONAL JUNIOR COLLEGE



2OO7 PRELIMINARY EXAMINATION




MATHEMATICS

974OIOI

Higher 2

Paper I
Additional Matcrials: Answer Paper 3 hours
Listof Formuh (MF15)
Cover Shect

126 Scptember 2007, Wcdnesday 0t15- lll5 hrs

INSTRUCTIONS TO CANDIDATES
Write your name, regisradon number, subject tutorial group, on all the work you hand in.
Write in dark blue or black pon on both si&s of the paper.
You may use a soft pencil for diagrams or graphs.
Do not use paper clips, highlighters, glw or conection fluid.

Answer ALL the questions.

Give non+xact numericol answers conect to 3 significant figures, or I decimal place in the case of
angles in degrees, unlqss a different level of muracy is specified in the queslion.

You are expected to use a graphic calculator.


Unsuppo(ed onswers from a graphic calculator are allowed unless a question specil-rcally states
otherwise. Whcrc ursupported answers from a graphic calculaior are not allowed in a question, you
are required to present the mat$ematical Seps using mathematical noEtions and not caloulator

commands.

You are reminded ofthe nced for cleargreseolation in your answcrs.


Up lo 3 mark ma! fu dedacledlot gancral poor pt6e atioa ln aa! pa of thlt papet.

The number of mark is given in the bractets [ ] at tlle end ofeach question or part qustion.
Al lhe end of$e examinarion, fasten al! your work securely together.

This qucstion peper coosisb of6 printed pagcs rnd ! blank page.

[Tu rn over
-2.

Veriff that -2+3i is a root of the equation z'+522+l7z+13=0 and determine the
other rools of the equdion. t4l

Solvcl +31>3-x2.

Hcnce,soivel -1>3-c". [51

3. A cubic curve is dcfined by the equation y=d + bxz +a+d' where a' b' c and d
are real comtants. The cflrve cuts the laxis at Jr=-8 and the 'r'otis at 'x =
4 The '
line y = -8-2r intersecb the curve at r = 3 and is parallel to ttre tangent of the curve
at x =2.
(D Write down the value of d [1]

(ii) Form a system of linear equations involving a, D and c, and find the equation
ofthe curvc. Show your workings clearly. t5]

4 dtth g" mh

[3]

tto terrns ofttc cxPanSb Of bttn an approximttion


(D Uing the st +x,
ttc onof Cq On = + e p W a frac o

Explain why thiS approximation is a valid Onc. [3

Byconsidering
5.
# #,showthat


+ ,

where a and D are constants to be determined.

Prove lhe above result by Mathematical Induction-

[Turn over
-3-

6
=f( )

Sketch, on separate diagrams, the fottowing graphs, indicating cteuly any asymptotes,
axial inrcrceps and tuming poins, where possible.

(1) = f( )-1:

r2l

=f( ); I2t

=f( ) t3l

7 The functions fand g are given by

f :r t-+ (x -2)2 -3,re R,r S 2


g:rHa-e-',reR
(i) Show that f -r exists and express f-t in similar form, stating the domain
clearly. t3I
(iD Determine the largest intcger value a such that fg exists. 121
(iii) For the largest value ofa obtained in part (ii), dctermine tlre domain and range
of fg. r2t

r= . i-l

8 Giventhat ,.detcrmine the cxact value ofthe modulus of zand show


(J'.r''
--Str
-12 --.
tnat argument ofz is t3]

(a) Find the fourth roots of the complex numbcr z. Give yow answer in the form
reid , where d is in radians. I2l
(b) Find the exact values ofa and.D which suisfy the equation z'=eo*iD, where
aeR,and -r <b3tt. t31

[Tu rn over
-4-

9. Given thatlf(r) = r+(zr+ r)+(3$, x * t, a > o.


Find the asymptotes of y = f(r) . t2t


l,rlt A and B be the minimum and maximum tuming points of y= f(x)
rtspectively. Show that coordinarcs of A md B ue (a+2,a(a+l)) and
(-a,0) respecively. t3l
(iii) Skach the graph of v=f(t) , indicating clearly the
wheo 0<a<2
asy{nptotes, Eming points and axial intErccpts. 121

(iv) Fin4 in tcrms of a, dte equation of the'parabola of the form y= k(x+ c)2
whose minimum hrning point is B and passes through l. [ll

10. A chemical spill from a pharmaceuticat plant causes a disturbance in the steady
population of irshes in a river. The total number of fishes (in thousands) in the river t
days after the spill isy, urd1,x is the initial steady comtant population.
y
It may be assumed that the natural population growth rate of the fishes is , ana
lyo
that the fishes are dying ar a rate of
' {t+5 au" to thc chemical spill. At the same time,

in an attempt to balance the number of fishes, environmentalists decided to release


1000 new fishes every day into the river, immediately after the spill'

(i) Show that the fsh population in the river immediatety after the chemical spill
can be modelled by the difrerential equation

dy tl+(y-y)t+5(t-n) [2
dt ,(r+5)

By using thc substi$tion I - lo = st , show that the above differential equation


can be reduced to
drI [3]
dt t+5

(iii) Given funher that thc total number of fishcs in the river retums to }t after l0

da),s, express y in terms of / and yo. Hcnce' by considering the graph ofy
against t or otherwise, deduce the minimum value of the initial steady constant
frpulation such lhat the fshcs will not become extinct because of
the
chemical spill.
t6I

[Turn over
c2o" (H Mqls PdPr l)

5


H.0 By"ing ttc tt ttbn =secr,cvduate
:7:

(b) Dduate

=

(0 Fhd c

tv of c"Sh(2 )cos(2)
6

t2. rhe plane rr, has equation , The linc I pcses rhrough two points A nd B
[+J=.
whose position vec,ors ,," *
[iJ [(J,*0".,,"",,
(a) Sto* tt ut the position rector of the point fl on fI, such that 8il is

perpcndicular to fI, is | ,l. H"n." find the perpendicular distance from I to


IrJ
l. [5]

(b) Veri$ lhat point ,'4 lies on fI,. Hence show that the reflection of / in II, is


31

(c) The planc fI, is perpendicular tLo fl, and contains I. Find the equation of fl,
inthc Fom r n= .

3]

(d) fI, is perpendicular !o both fl, and fI, and contains the line Bir'. By
corsidering the trianglc ,{B/V, or otherwisc, detcrmine

(O ule d tance of/Oom ,:

00 thC acutt anglc bctwccn and 3 [4

END OFPAPER l

[Turn over
6.

BLANK PACE

[Turn over

Nationel Junior Cotlcgc


' 2007 Year 2 Preliminaiy Eramiretions @rper 1)
H2 Mathemrtics (9740)

Suggested Solutions

Q N . Supc6acd Soluaioo! Rcmrrls

l Su tll =-2+3i lnto +522.172+:3=0.W get

(-2+ +5(-2+ +17(-2 )+13 0(By00

H =-2+31 is a root cSLown3

As l dlc cocfFtcic ts oFdle o on is"d,modler rootis

=-2-31

The last root ltlust bc B realllmbcr By trial and e

when z= :,(-1)'+5(-1 +17(-1)+:3=0

Hencc,z= :is Jso a ro t

Roots oFthe cquatioll =-2+3i, -2-31 and-1.


/
/
/


/ /

/
]
/ I
/

,

/
/

From rhc graplq .r < -0.5 or r>0.

f" sof* .rirg prcvi<ius rcsuhs, wc mrico lhrt r her


[--3|,I-s!
bco rcphc.cd wi0t -c'ircqudoo(t). Hcncc, wc hrvc

' -05
`>0
>05 0
0 A as callnot bc solvod)
>!t105
3.o y=@.'+bx2 +q+d
Wher r = 0,/ = -t, rE gc,d= -8
3(li)
y=d+br'+s-8
Whcot=4,/=0, wc gct
640 +l@+4c =8-1tu+1b+c=2 (l)
-
whn r = 3,/ = -E - 2(3) - -14, wc ecl
27 d +9b +3c = -14 + 90 +30 +c =
-t -2 -- (2)

9=td +zt +,
dr
whco x = 2,
fl=a,*oeo
l2o+4b+c=-2 Q)
-
Puning e4uarions (l) (2) and (3) into ur auBmcntcd mafix fo]It\ wc
gct

l,T:il-il """l,; l:l :l


[rz r rl+J [o o rl+J
Hcncc, o = l,b=-3o c--2.
Thcrcforc, cquaiion ofwvc is y = I -1r' -2, -8 .


T=(9+ ) :=92[l+;]

=:[:+(
:ll;)+ (;)=

=:[ i+ 1

+ r.
rtOm)
=:

4.(li)

= +

1 : l l i4 5 45
- X=
3 54 2 2 27 '=
6 28

fHdJ

Sh" v
=:: 9'

pronIIladon
,=

5.

=(
+) ( " )+(
)
+(
)+(
)
=:

Hcnce, =:,

bethe ttmentthat

,=1

Considcr P;:

LHSorPr=*Cd;rT=[t
fit T (3X5) r5

RHS orPl =
i-"h=i i=1 =1sso1p,.
Hencc, P1 is truc.

Assumc ofPl is ruc for somc t e Z' .



) 5
+ + 1+2
l

Consider Pl.:

RHS oFP :=:


:

LHS oFP 1
:)
(1+2 )(1+2

=1
[)(1+21 1)
+(l+2


: T
+ F
,FT

=: l

:
lhl=: l

+ M J

Thus,PA"uc P .lalso me

Sincc P: m,and Pl me lalSO me,hence,by


Mathematlcd illd don,P"is cZ
"V
6_() mphofy= ftr)-1

6(11) Craph of =F(


)

=F( 11)

6(ili) Graphofy=f'(r)

N3(200W ( 2 s, 1

7.(1) F: 2)2-3, eRx 2


(

8:' C `,ICR

Lct y=( 2)2_3

x=2 vy+3
=2-V 5,( =
2)

Fhus, 1:x 2- +3, e R,x -3 , No =


7(li)

___1___ 2=( J
4 =(*' ")
0

r For fg to cxist, R. g D. .

Hencq largost integer value


oto=2.

7(lii)

=2=R
USh8 =2,

R L
f r
(_2) (-s, -)
TLus t= 3, )

B(Z)=argl (i-1)-2 arg( 6+i)


=a

==
h )

8.( we m", = )_

J=Z=
(9

=
. CZ

W= j J, = Q12

8(b)

=
=


Compartng the modu:us and the argulllent

=h



9(i Lcr.,/= f(r)= r+(2,* g*$, x *t, o>o
By obsrvadon, Obliquc arymptotc; y=r+(2a+l)
Vcnic.al asy4otc: r=l

9(10

d2
dr2

For nti
"


J
1=`+l
)a=(a+1)2 ,1 = o+l
x-1= -l
or
t=o+2 ,=


Wllon
= +2,y=4 , >0( )
Q
Wnx= 0(Xn

Coordnd
+2,4
C rdmats oFB ( ,0)

9 (ill) When =0, = a2

=r+(2+1)

=1( +C)2
9( )
Mln pL 3 meansthatc=,We hav = + )' (:)

Subst +2,4(4+1))intO the eqll(1)and sOlvc For t

we geti =
+l

Thuq cquation ofparabola is y =


-L;(r +a)' .
10(0
=
)
+5( )+1 +5 -5
=2-y
+5) (

y.) +5( y )
=1+(
dr J 5)
Wnb

y. ;
10 (iD = r= = +,
" dr dr
ds
2+rr2+5"
+J=
`

=
+ +5 rr8-Ssr

= ,(, + 5)

=
= OWn)

10 0ii) htcgratirg both sidcs wrt ,,

,=J4ar=r"lr+{+c
t1"'nl'*4*'
y = y. + t\rl.|, + 5l+ Ct

Whcn I = 10, y =y6,


&:21 = 6116 a5lac + o = tnt5 +C::? C = -tn l5
,l
rhcreforc, y = y" +rtnlr + 5l -(rn rs)r = r = r" + I rnlf,l

Considet thc graph of y=r.*rfrffi for,>0. wc h,f,vc

in order ror dte 6shes notto


b o
"exttnct,


'0,V
y -20563)0

y.>20563

Thus.mlrL 20563
ie the min iiti steady nstant population k 2057
2001 ttn 2 PQF I 1


:

H( sec s =


=Scc`tan

Secrtan
=
=
m

=

F
=sin +C

= +c
X


=
IG
By ver up ml C=:

,2=( +8)( -1)+:(2 2+

WLcn =0, !)+:=0 =:


Wllcll =-1,( +:)(-2)+:(3)=l=o/=:

Thu =:(1 +7
)


aF
: +&

=:
:( 1)+
= +:
:[:h
+ m (a)+h
11+D

= h 2+ +:h
1+ (a)+D

10

(d,"*n)
d,=*(l-c)



(2,)
c"cos
(2,)
sio
J;
_(0
:3. :)
:2(a)

:r=:+
,
CR
G
a7V=:iiilForsOIIE CR
3

Thu

:li:llllll=4
-7+ -2(18-2 ) :+ =43 =8

0N=llil:i)=(ShOW

Pcrpcndicular distancc

=PI=1

8Vl+4+1=8 7

:2o)

2=4. [l;.

Thercfore point,{ liesin (Vcri6cd)


B = +

:2(c)
=:-lil=::=-311)

TWO VC Ors tO n,arelilmdill

:IX1121=liil=-21il

VeCtOr perpendiculaF tO n,, I12=lil Also,pOint :iCS ilt n,




TherefOre,riil=lil
lil=4

E4uttiOn oF n2:

{)'4
12. xi)I Dinnce mn.

1 =p

-1:=:

=Vl+4+25= 0

l2-(dxii) I Acutc angle betwccn I and II1

= =m
= 6 (01d

NATTONAL JUNIOR COLLEGE
2OO7 PRELIMINARY EXAMINATIONS

MATHEMATTCS 974OIO2
Higher 2

Paper 2

Additional Matcrials: Writing Paper 3 hours


List of Formulae (MFl5)
Cover Sheet

196 Septcmbcr 2007, Wcdncsday 1300 - l6lXl hn

INSTRUCTIONS TO CANDIDATES
Write your narne, registration number, subject tutorial group, on all the work you hand in-
Write in dark blue or black pen on both sides ofthe paper.
You may use a sofl pencil for diagrams or graphs.
Do not use paper clips, highlighters, glue or conection fluid.

Answer ALL the questions.

Give non-exact numerical answers corrert to 3 signi{icant hgures, or t decimal place in the case of
angles in degrees, unless a different level ofaccuracy-is specified in the question.

You arc expected to use a graphic calculator. I

Unsupported answers from a graphic calculator are allowe.d unless a question specifically states
ofherwise. Where unsupporled answers from a graphic colculator are not allowed in I question, you

are required to present the mathematical steps using mathemalical notations 8nd not calculstor

commands.

You are reminded of the oeed for clear presentation io your answes.
Ilp to 3 maths may be deductedlor giterol poot ptaenlatioa in atry Pa ol lhts pape?'

The number of marks is given in the brackes [.] u lhc end ofeach question or part question

At $e end of lhe examination, fasten all your wort securely togedrcr.

This question prper consists of 8 printed pagcs' including I btank pege'

[Turn over
-2-

Scction A: Purc Mrthcmatics [40 Mrrksl

The curve C is defined by the parametric equations

= = +:,
cR {0}
:,

Show that = [2

Detcrminc thc coordinates of the poins where the grdient of the tangent line is
parallel to thc.r-axiS. [2]

(iii) Show 0rat nonc of the tangent lines of this curve will pass through thc origin. [3]

Given that y=vl_ W alat


2'Sh

2) =

i) =2=O when,=0

2
0btain thc Maciaurin's expallsion dfy up to and including the term in

Hence find the Maclaurin's expansion of y = 5in-t x up to and including the term in

x'.
tzl

The sequcnce of non-zero numbers at,a2,ay... forms a convergcnt geometric

progtession. Given that apal utd -a, are three consecutive terms in an arithmetic

progression
-d tr, =(tr,-,)',
(D find thc common ratio of the gcometric progression. t3l
(iD Show that a, = b(Zt - l) whorc 6 is a real constant to be determined . t41

[Turu over
-3-

4. Sketch on an Argand diagram the set ofpoints representing all complex numbers z

satisfoing both of the following inequalities:

q s uc(z
-3tr -) <f nalz rl <lr++{
Fid the range ofvalues of lz +5 + 2il, leaving your arrswers to 3 sigrrificant figures.

[7]

50 agrm shtt part of e graph = .

=Vx'+l

(i) By considering ,, + I rectangles of equal widttr from .r = 0 to r=3,showthat

for all on ncgativc integer"


OP

(b) (D showthat
f(rrin"r*Ji-r')=r"-',.
12]

2)for
The rc on R bounded by tho curvcs y= and =Sin(

O
1:; Find the exact volume oFthe soiid obtained when R is rotated

about the y axis dlough 2 radians

[Turn over
-4-

Section B: Stetistics [50 Marksl

The Ministry of National Devclopment wishes to find out how the recent property
hike alfected Singaporeans. A list of houscholds, in ordcr of sumalncs, in a partiorlar
constitucncy is gcncratcd and evcry lOd household will be pickci to be interviewed
to get thcir rsponscs with regards to this issue.


(D Explain why the samplc is biased.

(iD What tlpe of sampling mahod is being desCribcd above?

(iii) Suggest a bettcr sampling mefiod Justiff your arswer.

1. Given the woTd "PHILIPPINES", find the number of anangements of thesc leners
such that
(D the ttree'l's are separated from cach othert I2l
(ii) the leners "PI" appear as ordered pairs in the word (e.g. PIPIPIHLNES). t?l

A bag contains ten marbles, outofwhich two are labeled " a ", two are labeled " D ",
tfuee are labeled " c " and three arc labeled " d". Four marbles are taken from the bag
at random with replacement.
.

(i) Shoul that the pmbability that all the sclecled marbles are distinctly labeled is
00864. tu
(ii) Find the probability drat each ofthe selected marble bears cither the letter " a "
or " d". I2l
(iii) The experiment abovc is repeated l0 times. Find the probability that therc are
more than 3 experiments such that all the selected marbles are distinctly labeled.

t3l

[Turn over

,tJ\, .rev | ,ls4llll FlqIn ur{ rv&flh t"r. D

g. A circutar card,. with a pointer pivoted ar lhe center, is divided into 5 uncqual sectors
numbered "l", and
*5". The pointer is spun and tre score will be the

number at which the pointcr stopped at.


The probability ofscoring a'5" is I - 4. The pointer is spun l0 times independently
and the numbcr of'5" obtained is denoted by f.

civenthat var(r)=[E(r)]',sho*trar q=ii. I3I

Suppose there are 50 pcople invited to spin the pointer ten times cach. Find the
probability that fte mcan number of times tlrey obain a'5" excceds l. t3I

10. Jet-Around travel agcncy iccives independently, at random instants oftime, an


averuge of 2 complaint and 3 compliment tetters each month

(i) Find the probability tha! in a randomly ihosen month, the agency reccives no
more than 6 lefiers. tzt
(ii) Using a suitable approximation, determine the probability tha! in a randomly
chosen year, the agency receives betwcen 45 and 55 letrcrs (inclusive). t3l
(iii) What is he probability tha! in a randomly chosen mont[ tlrc agency rece ives 3
or more complaint letters than compliment letters given that there are a total of 7
letters received?
t3I

11. Alvin and Bekky are interested in ,y, the number of sMS's a college student sends in a
day.

Alvin claims that this mcan number is higher than 25.


Bekky claims that this mean does not differ from 25.
Alvin conducted a sample 6n 50 studens and obtained the following results

Zx=t270 and ).r, =32918.


(i) Bascd on thesc results, test at 5% significancc leyel Alvin's clairn. t4l
(ii) Based on the rcutts in (i), state and explain ifBel*y's olaim isjustified at 5%
significance level.
t2t

[Turn over
-6-

[cont'd Question lll


Bekky also obtained her own data which is as shown below.
22, 21, 40, 24, 28, 15, 30, 41, u, 27.

(iii) Test at 5% sigrificance level Bekky's claim. Justiff the test you used and state
any assumption(s) you need. t4l
of60 smden
OV)BCtt Suggcs oombimng bo sc ofdata to obtain a samp

Explain why this might not bc advisable. tu

12. A PE teachcr, Miss Melon, wants to determinc if thcre is any conelation between the
weight of and 0re mean ambunt of money spent on csnteen food per month by an I 8'
year-old cotlege student. A group of eight l8-year-old mllege sttldents was
interviewed and had tlreir weights takcn. The results are tabulated as follow:

Weight x / kg

Mean amount spent on


food per mon0q y / $

Given that rhe equation of thc least squares regression line ofy on x is
y=1.0031r-4.3018, find the value of/<, leaving your answer in I decimal



placc.

for
Calcutate the value of the linear product moment correlation coeffrcient
these eight college students.

>
(iii) The following models for the above data are suggested for b 0:

2: (D)
(A'1 y= a+ bx:' (B) =a+b (C) Y=a+blnx; Y=o+!x

State, with a reason, which model is most appropriate ' {21

(iv) With tlrc choice of the modcl indicated in (iii), calcllate the least squares
estimates ofa and D and cslculat the value of the linear product moment

correlation coefficient for the lrarxformed data l2'l

(v) Suppose we have an l8'year-old college student who spends an average


of$80

per month on food, determine the weight of this student using the least squares

regression line obtained in (iv). comment on lhe suitability of


your answer. [2]

[Turn over
-7-

13. A fruits wholesaler solls durians imported from Thailand.


The most popular types of durians are the"D24" and'XO26". The masses of each
type of durians follow independent normal distributions witli the following means and
standard deviations:

The wholesale price of"D24" an d


*XO26"
durians are $30 per kg and $35 per kg
respectively.

(i) Calculate the probability that the mear mass of four..D24" duriars and five
"XO26" durians is between 1.4 kg and 1.7 kg. t3l
(ii) Find the probability that the difference between twice the mass of a.,XO26"
durian and the total mass of three "D24', durians is at least 0.5 kg. t3l
(iii) The cost of buying two "D24" durians and one ..XO26,' durian exceeds $a with
a probability of 0.1. Determine the value of a.
t3I
(iv) Three "D24" durians are selected at rardom. Determine the probability that two
of them weigh more than l.3kg.
I2l

**.** THE ENI) *.***

[Turn over

8

IBl nk Pagel

[Turn over

2007 JC2/1P4NJC Preliminary E amin3tiOnS Paper 2

10 x=
:
=1+

=
: i =1
Thus wc have= =(1-;
: ) (l+ )

.== hOWnD

1 0 WhCre tte ttientOftt tangentunc


lCl tO ule ax ,wc have
r2 0
=J
nusc
d ofsuch p are(0,2)&(Q-2).(anS)
====0
0 mJngcnt ha y=( ),+
:/+ , h
"
+:=(
J111=( +
)
=
)( :)+

=
(=)
=

="=#+*orincer*o
(r'+ l)
Hence-none of the tangent line of this curve will pass
tfuough the origin. (shown)

= 2 .2=: y20_,2)=:
_
DifFerendattng with ttrd to x,we get

2)(2'lb)=0=

'(-2x)+(1-
2)1:=0 +(1-

( ,2) = _
6hOWn)
2007 JC2 P4NJC Preliminary Ex3mi 3tiOnS Paper 2

Dircrcndathg(I)wih rcsP tto X,WC gct

(l_x2)11+(-2r):i +


2)g _3
(1_ ( )
::=
DirerentiatingcII)With resp ttO X,we gel

r) +(-2x) -3,
=
=5, +4
,

When I=0, =T =1, =0' =L


crefo" =O whcn,=0 0hOWn)

Thus y=1+lx+ ,2+l ]+ 1+:,2(a )

h graung wi w gd = + +
But when r=O,c=0. Then sin-r.r ^, x+f . f-O
3.(i) at-4=-dz-at= qr'-q=-qr-qr'
5 2r2 +r-l=O
(2r-1)( +!)=0



=:(ans) = :(NA)

)
= =( J
% =( %HI

=(

)=
al= )
(

q= (2
-0(shOWn)

2OO1 JA 1rc4 NJC Preliminary Examinations Papcr 2

4
L ofz

Greatest lz+5+2il = maximum dist bet z and (-5,-2)


=AB
=Janl +s=G8 +S=13.2

From diagram, coord of C t (l-s-rf ,-Sti.f).


Least lz+5+2il = minimum dist bet e and (-5,-2)
=AC

ll-8+: )2+(_2+i )2=4_72

Hence we have 4.72 s +5 + 2ils 13.2. (8ns)

s.(ai) J

=J 2+1
2007 JC2/1P4NJC Preiiminary am: 6ons Paper 2

Considering the diagram abovg we get

Ara ofthc rectsngles from r=0 to x=3

L


=

=

Area underthe curvc hm x=0 0o x=3


= : d,
EI

From thc diagrarn above,wc gct

: d DEJ9r2+(PI+1
(ans)

=J "

5.o)
(XSh + X'

=
(
:= )+Sin_:,+:(1-,2)(_2,)

= h0

2Ul7 JC2 I lP4 NJC Prctimiorry Ereminrtious Prpcr 2

5 oi)
,2
y=
+I

f, = sin (r')


Volume ofsolid

=
2 _
(:)(11)+ 1,12

= +
l J

= J+ + +V ]
= +
lnll_ 1 +

= Jn
+
]

=
n
+

6 (i) Only households in that particular constituency are


interviewed. Hence it is a biased sample.

6.(ii) Systematicsampling

6.(iii) A better sampling method is stratified sampling.


Using this method, the population will be divided into
non-ovedapping represntative strata like income group &
age group. Then houdrolds are selected randomly from
each stratum to be intervicrred. In this way, the mrnple is
more represntative ofthe popul6tion and hcnce gives
accurat responses to the issue.

7(:) Nurnber ofarrangements


](:)=564480( ")
7(li) Numbcr ofarrangcmcnts] =6720(allS)
2007 JC2 IP4 NJC Pre:imi ary Exominations P,per 2

8 o Requircd proba :ity

= x x x x4!=00864(shown)

8(D Required proba :

=(l)+(1
)( )1+(1 I( )2

+(1 r(:)
)(1 )1+(1 )
=Q0625(a )

8.(iii) La Xbe the number of experiments that have all lhe


selected marbles distinctly labeled.

Then x- B(10,0.0864)
Thus P(x > 4)= l-P(X s 3)=0.00764 (ans)
(t 0,0.0864,4)]
It - uinomcof

9. Lct/bc thc nu bcr of``5"obtalncd


Then/ B(101-9)

Now wchave VT(y)=109(l-9) dE( )=100-9)


Sincc Var(y)=[E( )]2,thcn

109(1-9)=[10(1-9)]2
9=10(1- )
9=10 109
1:9=10
9= Wn)
(Sh

P(mean umbcr oftimes they obtain a``5''cxcttdS l)


=P(7>1)

Nowwc know that 7 N(10(+),::(


)( ))by CLT

PF =Q20
mJ f[,
, ) )

2007 JCI t IPINJC Preliminary Ereminotions Papcr 2

10.(i) LtXand Ibethe number ofcomplaint and complimeot


leBers per month repectively.

Then X-Po(2) and r-ro(3) and x+r-Po(s).


:lhcrefore P(X + r < 6) = 0.?52 [poissoncdf (5,6) (ms)

10oo Lct r= +42+4+ +42


+.

Then I - no(6o) and hcnce I - N(60,60) approximatcly

P(4s s r < ss) = p(44.s < r < ss.s) = 0.258 (ans)

I f 44.Jl55:5160'.,6)
100iO P( / 31 +y=7)
/ 3 +y=7)
=P(
P( r+y=7)

Y=7,y=0)+P( r=6,y=1) P(jr=5,y=2)


=P(
P( r+y=7)
=(O IX1344)(90499+(10120)(9149)+(00361)(97 )
0.104

= = % m9 _

I l.(i) Ho lt=25 vs Hr:p>25


Under H ,wc have _N(25,::)by CLT

Since o2 is unknown, we set out to calcutate s,:

i9l I rrzsrs)-fl27o)'l= 660


", =
4elso'----'l.50/J 4e

We conduct a z- test at 5% significance level.

Using GC, we obtain p-vatue = O.220 ud z-value = 0.7?l

We reject Hu ifpvalue < 0-05.

Hence we do not reject H, and conctude at 5% sig. level


that there is insufficient evidence to say that p > 25 .
2007 JC). I lP4 NJC Preliminary Examinations Paper 2

I l.(ii) Ho : lt =25 vs H,: P *25


Nowpvalue = 0.22O x 2 = O.44O

We will reject Ho if;r'value < 0.05.


Hence we do not rejed fL and corrclude at 5% sig level
that Btty's claim is justified.

I l.(iii) Ho : lt =25 vs Htl. P * 25

Under Hn, we have X - N[rs,{.|


( '10l
We conduct a ,' test at 5% significance level since d2 is
unknown and samPle size is small.
We need to assumc that Xfollows a normal distribution

Using GC, we obtain pvalue = 0.M27 and t'valrc:2'159

We reject Ho ifPvalue < 0.05.

Hence we reject Hu and conclude at 5oZ sig level


that there is sufficient evidence to say that * 25 ' I
I I .(iv) This is not advisable since the 2 data seb might not be.
independent, e.g. the same interviewee is in both samples'



12.(i) Since v = I.oo3lx- 4 !018,then we have
6 = I 0031 and Y-6.r= -4'30t8'

Hence y- 1.0031(52.75) = -4.301 8= v = 48'611725

From the table we have


f,l = 338.7+t = 48.61 l?2s(8) =+ t = 50 2 (ans)

12.(D F m GC,we haVe r=0863(ans)

12.(iii) Model C is suitable since it is observed from the scatter


plot diagram that as r increases, thc rate at.which y
increases is decreasing Since b>0,MOdel I)iS not

12.(iv) L, -+.;r
Lz+ !
L, --+ ln(L1)

2007 JC2 IP4 NJC Preliminary Ex3minatiolls Paper 2

Using GC,LinReg L,,L2yiclds y=5372!n 16314


Hencc =-163 1ttb=5372, =0905(a )

12.(v) 80 = 53.72|n.x - 163.14 + x = 92.39 kg (urs)

Since y = 39 is out ofthe rangg the value ofx obtaincd


may not be accuratr.

13.(i) Lt Xand f be the uaeighs ot"D24" uld.Xo26" durians


respectively.
then x - N(I.s,0.t') and r - N(l.8p.2,).

y.I6yy ge1 1,7 X, +"'+ X.j ( +"'+ f, -(;,#)


= -

Therefore P(|.4 < M sl.1)=0 730 (ans)

bOmttJ 4 ,:,
)
13.o)La r=2 ( :+ + ) N(-09,019)
P(14 > 0.s) = p(r > o.s)+ p(r < -o.s) =0.s21 1ro;
[t - normatcdf (-0.5,0 s,-{.9,J0.19)]

13.(iii) Let C =30(X,+ X,)+35y - N(1s3,67)


Since we have P(C > a)= 6.13 n(C < a) = 9 9,
we get o = t63.49 [invnorm (O.e,rsl,.67)l t*rl
13(v)Req red P ba :lty=[P( >i3)]2P( i3)x]
=00652(ans)
SA:NT ANDREW'S JUN10R COLLEGE
PRELiMiNARY EXAMiNAT10N
MATHEMATICS
Higher 2 9740r01
Paper l

Thursday 30 August 2007 3 hours

Additionat materials : Answer paper


List of Formdae(MF15)
Cover Sheet

REAO THESE INSTRUCTIONS FIRST

Write your name, civics group and index number on all the work you hand in.
Write in dark blue or black pen on both skles of the paper.
You may use a soft pencil for any diagrams or graphs.

Answer all the questions.


Give non-exact numerical answers conect to 3 significant tigures, or 1 decimal place in the
case of angles in degrees, unless a different level of accuracy is specified in the question.
You are expecled to use a graphic calculator.
Unsupported ans\ /ers from a graphic cdculator are allowed unless a question speciftcally

state otherwise.
\y'Vhere unsupported ansu,ers from a graphic caldllator are not allowed in a question, you are
required to present the mathematic steps using mathematical notations and not calculator
commands.
You are reminded of the need for clear presentation in your answers.

The number of marks is given in brackets [ ] at the end of eacfi question or part question.
At the end of the examination, fasten all your work securely together.

This document consists of 6 printed pages including this page.


2

Sketch, on a single diagram, the graphs of f=2x+9 and y=l(r;12; , showing clearly the
point(s) of intersection by giving the x-coordinates in exac-t values.

Sotve the ineoualitv


2'*9
[31
lxl+2 =o.

The functions f, and g are defined by


f :xt-+2-x2, xeO, xSa
g:xr+ln(3-x), :eD, r<3

Find the latgest value of a given that the inverse f-l exists, and define f-r. [3]

Show thal gf exists. [11

3 (1) Exprcss flx)=


in partial fractions.

(ii) Given that-r is sufficiently small for x{ and higher powers of.r to be neglected,
show that f ) 2-2x. [3]

hO On re htcgcr
h

)=h( )

(z).

Hence find giving your answer in the form of ln [2

Mr Tan invested $25,000, part in a structured deposit account, part in bonds, and part in a mutual
fund. He invested $6,600 more in the bonds than the mutual frmd. After one year, he received a
total of $1,580 in simple interest from the three investments. The structured deposit account paid
an interest of6olo annually, the bonds paid 77o annually, and the mutual fund paid E% annually.
Find the amount Mr Tan invested in each category, giving your answer to the nearest dollar.


5]

[Turn over
3

3I J ! and g(r)

The diagram below shows tlre curve f(:1 = , the reflection of f(r) abolrt the line
x+c
l=a'
Graph of f(r) GphoFg )

Find the values of a,b and c. t4l


Find the equation of g(.r) .
t2l

Thecurve C has equation ,= "i' -li' where.r *I anda is a non-zero constant .


(D Show that if C has no stationary points, then -2 <a<0 .

(iD It is given that the line y:-r- I is an asymptote of C. Find the value ofa.

(iii) Sketch C, showing clearly the asymptotes and coordinates of any intersections with the
coordinate axes. [3]

Find A and B snch that 2k+l -A+ - B


2]
kz(k+l)z k2 (k+l)2
Show that the sum of the following series
3 5 7 +...+,r,(t,+tf
2n+l = t-*1y.
I
11g,+ 2.161,+ t(4f 13

Deduce that surn to infinity of this ,.rio I * 1 * 1 *... i, [ess than I . [3]
2'

[Turn over
9 (a) On a string of 3l pearls, the middle pearl is the largest and most expensive of all. Starting
from one en4 each pearl is worth $l(X) more than the one before, up to the middle pearl,
From the otlrer end, each pearl is $150 more than the one before, up to the middle pearl.
The string of pearls is worth $63,0O0. What is the value of the middle pearl? I4I

(b) An ant of rcgligible size crawls a distance of I metre to the right from the origin (0, 0) along

'2
the x-axis. It then tums baclq and crawls I metre to the left from its cunnt point to the point

(j,O). continues going back and forth, each time going half the distance it
"* -,
previously went, and repeating the Eattem, where does the ant evntually end up? Give your
answer as the coordinates of the final position ofthe ant. t31

10 consldcr the scqucncc dcflned by


.l=. _.+3,for al:Posit e integers of ,and 0=3

Z

O Find the values of u, u, and ur. [2]

(ii) Suppose z, converges to / as z -+ co. By forming and solving a suitable quadratic equation,
find the valueofl. l4l
(iii) Under what condition will z, converge to the other limit you rejected in (ii)?
You should use calculations obtained from your G.C. to substantiate your answer. tl l

ll / ' / \
(a) civen that u=2lcosI+isin4l-a
--6) ,=+[ nna the modulus and argument
\ 6 3
\ "o.1-itin4l,
3)'
lt+
of' -. t4I
w

(b) Let z be the complex number - I+ iJJ . Find the value of the real number a such that

^re(r'+*)='L- ts1

[Turn over
5

12 (i) Show that the series expansion of ln(cos 2.r) in ascending powers of , up to and including the

A 8
lrt . Hence show that
in( S2"-0
term in r' is - 2r2 - 3. 16]
J 0

(ii) The region R is bounded by the axes, the curve y =l + xtan2x ana m line ,:
i.
Using integration by parts and the result in (i), find the approximate area ofit.
E4]

13

It is required to take a rectangular frame in a horizontal position along a corridor bounded


by
veftical walls of which a horizontal cross-section is two concentric semicircles of radii
r and ..6r;
the frame is oflength 2.x and breadth y (see above figure). one side of length
2r is tangential to the
inner wall, and the two ends ofthe opposite side areln conracr with the ou-ter
wa[, as Ihown in the
diagram above.


Prove that x2 = 2rz -2ry-yr-

Prove that, if.r and y may vary, the greatest possible


area enclosed by the frame


[Please turn over for Question I4l

[Turn over
6

14 (a) By using the substitution ar=3sin0 or otherwise, find the valueofa if


I





151

(b) (i) Given that .r and y are positive numbers such that y2 + 2ry + 2x2 = I , show that

v=Jl-xt -x.
Show also thar the gradient of the curve with equation y2 +2ry+2t2 = I is negative for
all positive values of: and y. t41

(ii) The diagram shows the curve with the equation y2 +2ry+2x2 =l for.r>0, -v>0.
R is the shaded region bounded by the curve y2 +2ry+2x7 =l,the line y:r
and the y-axis.

y'+ 2nJ + 2*= 1

Find the volume of the solid formed when R is rotated through 2t radians about the
.x-axis. Give your answer corect to 3 decimal places. t31

End of Paper I

[Turn over
2007 WL lUaths Prelim Exam(Paper 1) Solutions


dinatcs of mo r Ons:,

For 4,

_ the solution set is ( :


:or :}

2(1) 0=0
f: 2-
2, xc
,x 0
y=2- 2

x= V2-y sinccx 0
f 1( )= V2- , 2
ShCe =(,21 (,3)=2,gfex .

3(1) +C
=TTT+1+ 2

2=/(1+ 2)+(+C)(1+ )
Subts x=_1,/=I
Comparing cocfflcients Of

2:B=_1
x :C=1
=1, =-1,c=1

0=
I l-r
--.1-_
l+x l+x'
= (r +,)-' +r,f'
+ (r -..{r
= (l -,+,'-r'+...)+( -r{l -r, *...)
_ I -x +x2 --r' +. .. +
| -x -x2 +r' +-..
= 2 -2x +... (Stnwn)
2

h Statcmem h
+)=h( )WhC

Z =h
)=h:,
=h( )=h(:)


rme n
TIsme Ve htegcr l 2,Lc

ih +)
n(
)
Wantto prove that 4J is true based on 4 istrue,

(
h
)

=2h J
=h()Jn )

=h( ) )

=h( )
)

=h( )( )

=h(#
)=
mQ 43m
A 4J meb6d m


by Mathcmatical lnduction
(D ilh _+)
=']in(1_)-1:in(1-)

Jn( n(
) )


n(7) )

: )
= =in(J:

_ =in(: 1:)

5 tet r be the amounr ofmoney invested in ttre structu.ed deposiiiccoiit


be the amount Ofrnoncy invcstcd in bOnds
z be dle alnount ofrnoncy invcsted in thc mutual ttnd

+ +Z=25000
006x+007 +008z=1580
a z=6600

_ x=$17,467,y=S7,067,z= 67
6(D Lety= 1)
= C iS the vcrtical asymptote So
_ =-1
l B
Jm,y= + d

_ = is thc hodzontal asymptote


Sincc thc rencc n of aboutthe lnc y= ThepOht lo, ) the
midpoint of 'and/.
=5+ 1)=2
2
2 +b
_ y= _1
Whcn =o,y=-1_

-1=
-1

b=1
( )TO getgo mo
l)Shi dOWn 2 units along y axis
2)Rencct about x axis
3)Sht up 2 ulllts along y axis

g( )=
(f )-2)+2=2,-5

Altemative soll lon l:


(1) y=f(.)=2x+1

g( )= f( +4
2x-5
1

Al mative lution 2

(li) Since F is a tangular hyperbol y=2 and =l are its asymptotes

bt g(X)=
When x=0, =5
5=
-1

=-5

7(D 2+2
=
_1

Since C has no stationary points, there are no real roots for a' -2asc - 2=0
(-2a)'-4a(-2)<o
4a(a +21 <0
-2 <a <0

ax2 +2 2+ a
(iD Y=----, =E+a*-
AsymPtoteisY=6+o-
a= -l

4
00 hf = :

2k +l
=
AB
I,[TF k' ' (k +t)'
2k +l A(k +l)2 + Bkl
Fu;g= k'1k +11'
2k+l _ A(kz +2k +l) + Bk2
2(
+1)2 k7 (k + t)2
+ =0
2=2/
=1,3=-1
2 +1 1
2(
+1)2 2 (
+l)2

+ + =
=

( )

+ -7

=1 ( Wn)
+1)2(Sh
8 Note:( +1)2( +1)2> 2( +1)2 for all posilve integers Thus,
3 5 7 2 +1
+
220)2+3202+42(02+ + 2 2

++
T :5 F::11

+ +
d
) .

tK16
+ +

t
+

Q
90)btthe st ofthc middle pcari bc x
Sum of16 PcariS(WhiCh difFer by$100)=
0
(2x+15(-lCXl))=16x-12
Suln of 16 pcaris(whiCh dif r by S150)=

1 -18000
(2x+15(-150))=16
Cost of n acc=(16x-12Cll10)+(16 -1 8011Cl) =31x-30000=63000
Solving,x=S3000

(b) Lct a=1, =-05



Ir=:
Coodh s of on tth am=(:p)_

102^
(D \=-1r=),
'' 2 )
uz=L+3=31r'r= . ti-, *l=+L5
(ii) Suppose z, converges as z-+@, we can assume that t,-un,r'
Write z, =-2-4'
u"-2
Expand and solve the equation ui - 5u, + 4 =0 .

("" - 4Xr, - t):0 ; z,=46s1


By G.C. or observation from part (i), limit is 4. (reject z, =l). l: 4
(iiD When z, = l' or u, = l-, the sequence diverges (fiom G.C-). z" will only
converge to I when zo = I .

G n =2( s:+ n S: FgnO,


)and W=(

2,arg = mdlW 4,arg
= = = =
)=arg Z arg
3

= -argz -3ar9v,

=lz\-{
to/ -[-z]r
r.l
=5o
6

tt 2r
ar$\z)=E--=-
/. \ T
atg\z' + az
)= --
ary(z(z+a))=-1

arg(z) + arg(z + a1= -L


z2z
atg\z +a) = -r-T
7r
arg\z +a) = --

.. 5r
arylz-(-a))=-
Equation of this line:

r-o =-ta",(")C-c"l)
v=-lG*ol
a/3'
Since poinr I t,.fi) in,"o"", this line,
= 1+ )

-3=-l+a
a=-2
Altemative solution :

"'
a o, = 1-l +,tii)'z +a(-t + fl;
=-<2+d+J1@-2)i
arg(z2.aZ)=
:
tan-1[
] ]= :
2+ )=0
=-2

hlcos2
n[
+ ]



=-2
2+2,4_2x4
3

Alternative solution:

Let y=in(cos2 )

=2Jn2 =_2tan 2

" cos 2x

22x
=

m2,= 6
= 2x
22
6s n2x

=-161scc2 2 (2scci 2x)+4scc2xsec2xtan2xtan2xl



=-32scc4 2x-64sec2 2x tan2 2x
When =0,y=0,

=Q =4, =Q =-32
Th cxpmJon
H +
0+ =2x2_:/6hm
3_f: 5][
)
1 lin(CoS2 [ :

-0043 (Shown)

8
Area R=,+xm2
0
By Parts

r
=
=tan 2x

=lxf in(COS2 )]i+::lin(CoS2 )d
[: =1 =_ h( s2 )
2
in 043)
: : 3+:(-4
=0.439
13 (r+ y)2 +x'=3r'
12 +2ry+ y2 + x2 =3rz
,' =2r' -2ry- y'
Area of the fram e, A = 2ry

=2J2r' -2ry- y'y


=2J2r'y' -2ry'-yn


=2 (lyr' -a,y' -+y')
2
2-34 2)
2
_2 (2
2 22 3 4
V2

When =0,

o (na)

When =:,


X=
The =2(:)(291)

14
0) Let 4r = 3sin d.
dx3-
-dea-cosa
=

=21.*ze:\,
al 2
=Ll !,,,nr, *r11
2aL2 Jo

:90
4a
Soa:2

(b) yz +2ry+2x2 --l


(t+r\'+t'--l
(t+x)'=t-x'
y=1$-i -'
we reject y = -.ll= -, since Yzo
Hencey=J1-i -r -

d'
dr= -: -l< 0 for all positive values of.r on the curve
Jl -"
When r=y, 5r' =l,so .r=1.
.6
1

2]`& r 2_.)2 _:

/= _ l

i[( )2_ [( (1:)2 F=0716


(by GC)

10
SA:NT ANDREW'S JUN:OR COLLEGE
PRELIMiNARY EXAMiNAT:ON
MATHEMATiCS
Higher 2 9740 02
Paper 2

Wednesday 12 Sep 2007 3 hours

Additiona:mateda:s: Answer paper


List of Fonmu:ae(MF15)
Cover Sheet

REAO THESE INSTRUCTIONS FIRST

Write your name, civics group and index number on all the work you hand in.
Write in dark blue or black pen on both sides of the paper.
You may use a soft pencil for any diagrams or graphs.

Answer all the questions.


Give non-exact numerical answers conect to 3 significant figures, or 1 decinral place in the
case of angles in degrees, unless a different level of accurary is specified in the question.
You are expected to use a graphic calculator.
Unsupported ansvvers from a graphic calcudator are allorcd unless a question specifcally
state otherwise.
where ursupported answers from a graphic calculator are not allowed in a question, you are
required to present the mathematic steps using mathernatical notations and not calcuiator
commands.
You are reminded of the need for clear presentation in your answers.

The number of marks is given in brackets [ ] at the end of eacfi question or part question.
At the end of the examination, fasten all your work securely together.

This document consists of 6 printed pages including this page.


um over
2
Section A (40 marks)

Find the three complex numbers, zo, zp z2 that satis! the equation ,' - +Ji + *lZi = O .

Give your answers in the form red , where 0 is in terms of zr. E4]

Hence show tlut w : zl + zl + zj is an imaginary number. Find Im(rry). [3]

Wtren Mrs Wong retired in 2006, sh put a sum of $50fl) into a firnd that has a corstant rate of
retum of 5 % per annum. Starting in 2006, she withdraws $4O0 each year and gives the money
to her grandaughter as a birtMay gift. Denote the smount of money Mrs Wong has at time
r years by $x.

(D The differential equation relating.r and , is in the form 9I = /a+c . State the values of t
dt
and c. [ll

(ii) Solve the differential equation and find the amount of money Mrs Wong has after
l5 years. Cive your answer to the nearest integer. [41

(iii) In which year will the granddaughter receive her last $400? [2]

Comment on whether the model can be regarded as a good model of the situation in the real
world. tII

The paramehic equations of a curve are given by .r=asecO, y=atan0 where a is a real
constant.

) ShOW that =cos [2]


OD Rnd h rate of chang of 0=:when hcrcasing tt a constatt ratc of

2 per second [2]


00 nd dle quation oftk tangem th curve a the pi R(as o,a tan )and ShOW
that dlc eqtlation 9f tt nonnd to the curvc is y=2 tan sin . [2]

(iV)The tangent and nol11.alat R m tthe raxis at r and respectively


By indhgthe

c dinates of rand
h the dm e Hn,showthat16 lpl=242
[5]

[Turn over
3

The points l, B ard C have position vectors, relative to the origin, i - 2l\ 3i+ j- k and
-5j-7k respectively.

(D Find, in the form , - n = p, an equation of the plane fll which contains ,,{, B ard C. [3]

The line I1 passes through the points I and B. The point ^R on /1 is srrch that CR is
perpendicular to Ir. The line 12 passes tkough C ard .R and the point Q on 12 is such that
],CR=2c8.

Find the position vector of-R. t4l


Deduce an equation, in the form r. n:p, for ttre plane tI2 which contains the line /: and
is perpa rdicular to II1. t3I
(iv) Find the area of the tnangle BCQ,leaving your answer in surd form. t4l

SectionB(60marks)

There are l0 members in the Project work committee who sit around a table. There is t seat
designated for the Chairman only- There are 4 members wtro constitute a written Report
subcommittee- Find the number of ways the l0 members can be seared around the table if
(a)


the members'of the Written Report subcommittee have to sit together;
(b) no two members of the Written Report subcommittee sit next to each other.

The helghs ofa new species of sunflower are normally distributed with a nrean of tgo
and a variance of90 cm. "rn

(i) Find the least value ofn such that the probability that the average height ofa random
sample of n sunflowers exceeds 181.5 cm is less than I%.
t3l
(ii) A random sample of 20 sunflowers is taken and the average height is found. Find the
probability that th sample mean is greater than l7E cm.
l2l
A hundred such samples, each of20 sunflowerq are laken. In how many of these would
you expect the sample mean to be greater than 178 cm?
tll

[Turn over
4
Of the bicycles parked in the void deck ofa HDB flat,607o belongs to males and the rest to
females. 90% of the bicycles belonging to the males are racrs as arc 7V/o of the bicycles
belonging to females. Dynamos are frtted to 5o/o of the non-racing and l% of the racing
bicycles irrespective of whether the birycle is owned by a male or a female.

Ifa bicycle is chosen at random find the probability that it is


(D a racer with a dynamo belonging to a male,


(ii) a racer without dynamo

A bicycle is chosen at randorn If it is not a racer, find the probability that it belongs to a
female. t3l

Measurements of the relative humidity and moisturc content of samples of a certain type of
raw material on l0 days yielded the following results :

Relative
46 53 37 42 34 29 60 44 41 48
Humidity)
Moisture
12 14 13 10 8 17 12 10 15
Content(//o)

Draw a scatter diagram for the data Calculate th prodrct-moment correlation


coefTicient. Explain what they reveal about the relationship between relative humidity
and moisture content. [3]

It is required to estimate the moisture content given that the relative humidity is 58%.
Find the equation of a suitable least squares regression line. Use your equation to
obtain the required estimate. [3]

(iii) Use your equation in (ii) to estimare the moisture contenl when rlative humidity is
l0olo. Comment on the reliability of the estimation [2

(iv) It was found out that the moisture content of the raw material was measured wrongly
due to a fault in the measuring instmrnent. The acttral moisture content is l0olo more
than the measured values. Wilt this affect the product-moment correlation coeflicient?
Explain yotn answer. ll

[Turn over
5

9 Calls to a chatline service are received at random times at an average rate of9 per hour. The
service is offered for l0 hours in one day.

(i) Show the probability that more than 9 calls are made in a randomly chosen one-hour
period is 0.413.
t}l
& one hour periods are chosen at random, where t > lfi). The number of calls received
in each period is recorded. Given the probability that more than 9 calls are made in at
least 20 out of the * periods exceeds 0.9, use a Normal approimation to show that I
satis& the(approximate) ineqtrality
k-47-2>r.$Ji t4I
(ii) By using a suitable approximatioq find also the probability that the total number of
calls made in one day is at least 90 but less than I OO. I31

10 A certain brand of butter cookies are packed in two sizes : small and large. For each size, the
mass, in grams, of a randomly chosen packet is normally distributed with mean and standard
deviation as follows :

Mean Standard deviation


Small packet 720 9
Large pecket 970 10

0 Find the probability that, of2 randomly chosen large packets, one contains morc than
960 g ofcookies and one contains less than 960 g ofcookies.
l}t
(b) The-cookies are delivered to supermarkets in boxes of20 packets of the same size.
Find the probability that a box of20 small packes contains more than 4 packets
whose contents are each less than 710 g. t3I
(c) Find the probability that 3 randomry chosen large packets contain at lest l0 g more
than 4 randomly chosen small packes. t3]
(d) The label on a small packet ofcookies reads .llet mass : 715 grams". A trading
standard offrcer insists that at least 90olo ofsuch packets should contain cookiei with
mass ofat least 715 g. Assriming that the standard deviation remains ,nchanged, find"
the least value of the new mean mass of a small packet of cmkies that is coniistent
with this requirement.
t31

[Turn over
6
lf(a) A department store rcported that over the past six months, the average amormt spent per
customer has been $215 with standard deviation of $100. The store carried out a sales
promotion for a week on a range of products. In order to test, at 5o/o level of significance,
whether or not the sales pmmotion has increased the average amount spent per customer, a
random sample bf 100 customers visiting tlre store during the sales promotion week was
taken. The amorurt spent per customer was recorded.

State, with a rcason, whether, in performing the above test, it is necessary to assume that the
amount spent pr customer follows a normal distribution. tll
Write down the null and alternative hypotheses under test. Show that the null hypothesis is
rejected if the sample mean is greater than $231 .45. t31

(b) Based on past results, the average score in a round obtained by a golfer was 88 poins. In
order to improve his skills in golf (i.e. make his score smaller), he attended a series of golf
lessons. For each of tlre 8 rourds after the lessons, he calculared his score, x. The results are
summarized by :
),t,-to) =ots, l{,-to)' = 47343.

(D Calculate unbiased estimates ofthe population mean and variarrce.

Carry out an appropriate test to establish whether or not the lessons had sigrrificantly
produced, at the l0% level, a signifrcant improvement in his skills in golf (i.e. hed
mede his scores q!!g).
State an assumption made in using the test that you have chosen. t61

(iD Suppose the data from the 8 rounds had been such that the unbiased estimate of
'popuiation'variarae as found in (i) was larger, but without changing the sample mean.
State, with a reason, whether this would have an effect on the conclusion of the test.
tzt

End of Paper 2

[Turn over

[Turn over

24X17H2Maths Prellm E m(PaDer 2)Solutions
S tion A

4 f4 =& =

z3=4V2-i4
r
z =8`(2
),.=0,1,2 r 1).

z=2
rl)
12

z=2 (1 ),1=,l,2 r-1)


. , .
z=2 ', 2 and 2 4 r2 4)
= +zl+zi
=(2 )6+(2` )6+(2 )6


:+
_ 26[ =+ ]

=26[` :+`:+ :]



Hencc lm(w)= 192

-400,
2(i)

) =005x
=005, =-400

(ii) J0.05x-400
l----l-dx= rtdt

hp ,-40q +c
hp05 -4CXll=005 +Cl
005x-400= '`
005x=/47 5:+41Xl
,= 5`+811tXl
Whcn =0,x=501Xl 5Cll10=B+801Xl
=-3000
Hence x=-3000 '+8000
WhDn =15,
x=-3CX10` 75+8QXl= 3510+8CXXl
_ :649
(iiD
Whenx:o.eoos' = !+o.o5r=0.9808
J
- t= 19.62
On the l9m year, the granddaughter will receive her last $40O.
It will be in year ( 2006+19-l) = 2024
Possible answers for last part:
Most rate of returns of fund flrrchrates from year to year, rc probably not a good model.
The retums are normally compounded over discrete time interval (uually years) raflrer
than continuously. So pmbably rlot a good model.
When x = 0 , the model will no longer valid. So may rrct be a good model.

3(D x=asere y=abne


4=os,y-otzrre 4 = o"o'o
d0 de
dy_dyd0
& d0&
aw'e
asec0 tan0
= cosecQ
t t
dy dy dx
dt &dt
= (cr,sec0){2)

N e =a.4 =zcosecL=4
6.dt 6

(iii) Equation of tangent at point R :

y - a tAn d = crls eca (x - a s* a)


y = a(tut d - cns eca sec a) + x cos eca
Equation of normal at point R :

=- cos 'eca (-x-aseca)


I
y-atand
( a\
y = otana -(sina;l x - ---- ;
\ cos4,/
y =2olana - xsinrl
(iv) At point T,/ = 0
0= a(tana - cos eca s* a) + xaseca
a(tana - as eca seca\
X=
CCIS aC

= sec (1-sin2 )

= OOS

At pOint N,y=0

O:2a Wra - xsina


2a tana
sind
:2a*ca

lorl lorl = la cos all2a s* al


=2a2



O =


SO / =lil, =::

1111=2

6 +12=0 =-2







Alternative mtd : A normal to



:1rQn

or QC or rRC

( 3 =2 =:

=: ,BR=3




Area ofBC2=:CO BR=:(:5


Alternat e method:

CB=31:

Area=: X

= = Cans hsud gm)


Section B
5(a) No. of ways the Chairman(only I way), the block of 4 subcommittee members and 5 others
can be seated : 6!
No. of ways the 4 subcommittee members can be seated within the block = 4t
By Multiplication Principle, rotal no of ways = 6t4l = 17280

5(b) No. of ways the members(excluding the subcommittee) can be seated = 5i


No. of ways the 4 subcommittee members can be seated benveen the other 6 members = 6p.
By Multiplication Principle, total no of ways = 5! 6 P, = 43200

) Lt Xbe r.v. "height of sunflower in cm".


F- iv6ao,29y
n
p(7" > 18r.5) <0.01:+ l- p(7" < 181.5) <o.0t i p(7, s t81.5) >0.99

IZ >099
>2.326

>2164
Lcast =217



o80, )

P( 20>178)=08271
-Lct y bc no Ofsamples with mcan>
178 out of 100 sample"
y B(100,0.8271)
E( =lClll X O.8271 83

P(a raCer wih a dynalno bcionging to a male)=09x06x001

=0_0054

P(a raCCr without a dynamO)=(09X06+07x04)0_99


=0.81i8
P(it be10ngs to a girl ifit is not a racco

=P(be10ngS to female not a racer)

=P( )

0.3x04
0.lxO.6+0.3xO_4
=0.667

r = 0.932
The scatter diagram and the value of r suggests a strong positive linear correlation between r
and y.

(ii) Regression line ofy on x : y : O.348 + 0.271x

:
When r 58,y = 9.34t * 0.273(58) : 16.2
So moisture content is 16.2%.

(iii) When ;r = 10, y : 0.348 + 0.273( l0) = 3-08. So moisture content is 3.08o%.

The estimation may be unreliable as r = t0 is outside the range ofthe given data.

(i") No. r will be the same because the value ofr is not affected by a linear transformation on y.

Let be the numbcr ofca s madcin a one hour period

P )
P( >9)=l P( 9)=0413(to 3 sO(ShOWn)

Let y bc the numbcr ofperiods whcre morc than 9 calls are made out of pcriods
y (1,0413)
y
(0.4131,0.24241)apprOXimately
Givcn that P(y 20) 9
P(y 195)>0.9
P( 19.5) 01
P llil
:;:: )
2

19.5-0413 -1.282 0.2424
-47_2>1.53
Lt rbc thc total nulnbcr ofcalk madc in one day.
r P (9o)
Si >10,r (9o )app Ximately
Oo r loo)=PC89.5 r 99.5)aner continuity rrection
Probabi requircd= rdd (89.5,99.5,"; )
=0.363 cto 3 s

mdL L

1
TFlttal:
b h grallrl m6s Ofa largc pac h graln

(a) Required probability =


2P(>960)P(L<960)
:2(0.8413X I 0.8413)
:o.267
-

(b) kt X be r.v. "no. of small packets of mass less than 710 g in a box of 20,'.
P(S<710)=0.1333
x - B(20,0.1333)
P(X>s) = I - P(X-<a1 = 6.1 17

) (1"+ I"+ I")-(5, +s, +s, +s,)- N (30,624)


P(L,+ I-, + r, -(S, +S, +S, +S.) > l0): 0.78g

(d)
P(E7ls) > 0.9
P(
\z.7ts-t'\<o.t
e)
715-P
9 =-t.2sz
1t > 726-5
Least value of the mean mass ofa small packet = 727 g
H )ItiS nOtnecew to, urllc that tt amolmt spcnt per customer fonOws a mmal d bulon
bccause dlc salnPling di bution ofmearls is approximatcly nonnal by C ntral Limit
nlcorem(sinCe salnple sizc largo

l ,`bc dlc alnt sPcnt pCr customcr.


Test ff : =215 vs : >215

Under ffO, 215, = (215J )

At 5%levcl,the cttical x valuc is invNorm(095,215,lo=231.45


So ff reJ ted when I>231.45

Alternative method:
At5%levcl,the ci z value is invNormal(0_95)=1.645.
If I>231_45,
-215 23145-215
10 10
->
z>1645
So ls reJected at 5%lcvcl

ll(
()unbiaSed estimatc ofPopuhtlon mcan=I= +10=86.875%869
8
Unbiascd estimate ofpopulation variancc

=)(47343-
:ll)
=9.268
%927
=88 vs 1: 88
Test ff :

GC:T st,Sttts, =88,I=86875, = 268=3044,


=8, o Calculatc_
Since
=0165>0.1 rt=-1045>invTo ,7)=-141)
Do notrq tI .There b insufrlclcnt evidence at 10%levd ofsignincance that thc
icssons had made an in provcmcnt in his gol[

Assumption: re in a round follows a Nollllal diS bution



(D h COnclusion would be thc salncCe. Win not be reioCted)

SISbCCan the md = X

l' 1s bigger than r(the test statistic value found in part i)) utren s'>9-27 , hence it will
still not lie in the critical region
OR(comparing p-values): P(T <t")> P(f < r) = 0.165 > 0. I
SERANG00NJUN R COLEGE
24X17JC2 PRELIMINARY EXAPIINA ONS

MA 1LMATICS

igher 2 974OIPzpr I

Wedneday 12 September 2fi17

Additional materials: Writing paper

List of Formulae (MFl5)

TIME : 3 hours

READ THESE INSTRUCIIONS FIRST


Write your name and class on the cover page and on all the work you hand in.
Write in dark or black pen on both sides of the paper.
You may use a soft pencil for any diagrams or graphs.
Do not use staples, paper clips, highlighters, glue or correction fluid.

Arswer all the questiors.


Give non-exact numerical answers correct to 3 significant figures, or l decimal place in the
case
ofangles in degrees, unless a different level ofaccuracy ls speciriea in the questiln.
You are expected to use a graphic calculator.
Unsupported ansr'r'ers from a graphic calculator are allowed unless a question
specifically states
otherwise.
where unsupported answers from a graphic carculator are not afiowed in a question, you
are
required to present the mathematical steps using mathematical notations and not
caiculator
commands.
You are reminded ofthe need for clear presentation in your answers.

The-nunber of marks is given in brackets I I at the end ofeach question or part quesrion.
At the end ofthe examination, fasten all your work securely togeiher.

This question paper consists of8 printed pages and no blenk pages-
[Turn Over
2

Answer all questions (lfi) marks).

% e ttdnoqual
> 131



n n"uality >
b
2


Show that can be w tten in thc fonn ,

T
=:
2 :
Hence, or otherwise, find ) 2_3 +1)( 2+ _1)
(

Usc the abovc resultto show thatthc sum to inflni

+ +
=
121
is less than I .

Sketch the locus of arg(l -2 z)--L inan Argand diagram- Find the least value of lz+cl

where c = l-i. 141


/re0*

Find also the range of vatues of&, where such there is exactly one comPlex

pl
number z, that satisfies both lz+cl=.k and arg(l-22) =
,3
2
4 (a) A sequence {.r,} ofpositive numbers is defined u ,*r=##forzfJl*-
The sequence {rr} converges to a number 2 as n tends to infinity. Find the
exact value of 2. 121

+3)% for
( A sequen (%}iS deined as lrl=3 and
+1=( l+. nd

first four terms of the sequence and show that these terms can be written as

= for n: 1,2,3 and 4 where a and 6 are constants to be determined.


", #*
With the values of a and b found, prove that = *e method of
", ffiAV
induction for z > l. {51

5 (a) Find ` if

,
121

0n( )

21

(b) Givcn that


=( -5y)3,

nd h us ofF and . pl

[Turn Over

(a)

The points Sand lare the ends of the diameter of a circle with centre O- Uisa
point such that it lies on dre circumference ofthe circle. Taking the centre 0 as
the origin, the position vectors ofpoints S, i'and Uare s, t and u respectively.

Write out the vectors SU and TU in terms of s, t and u and hence show that the
lines SUand IU are perpendicular. I3l

(b) Given that OAPB is a parallelogram and p is the mid point of AP. OP meets BO
at X. The position vectors ofl and B are a and b respectively.

-) -)

Find OP and OQ in terms of a and b. 121

If R is the point on BQ produced such that BQ = QiR, show that R is on OA


produced and that OR -- 2OA. 131
5

(a) The diagram below shows &e graph ofy = (:). On a separate diagram, sketch a
clearly, the larbeled graph of y = f (.r) .

13

(b) The functions fand g arc delined as follows:

f:rr-+(t+2r)-r, r<*1,

g,r-1,
x-l -rcx<*

(i)


Find the maximum value f can take such that fg exists.


(ii) With the value of t found in (i), find fg (.r) and state its range.

(iiD Obtain the first three terms in the expansion, in ascending powers of.r,
of
fg. State the set ofvalues ofx for which the expansion is valid.

3

[Turu Over

B 0)

A cylindrical silo with a hemispherical roof top is constructed to store animal


fodder on a cattle ranch.

The cost ofeach unit area of the roof is fixed at P dollars and the cost ofeach unit

area of the curved surface ofthe cylinder is fixed at


'3 lP dollars.

The total cost of the roof and the cuwed surface is a fixed amount, C dollars'
Given that the radius of the hemisphere is r, show that the volume I/ of the
stn cture can be expressed as

3C _-l-r
y.. = _r 7tr t .
131

Show that, as r varies, ,/ is at its maximum value when the cost of hemispherical
.3^ 131
,7ls -L
toD .

2ro'3'1
[Curved surface area of hemisphere = 2rr2;volume of hemis nn"n =

g
(b) lf v = tao(2x -A), show that =ZQ* y').
4" dr

By repeated differentiation ofthis result, show that

#=rl,#.,(*)[#l
a'
Hence find the Maclaurin's expansion ofy, up to and including the terms in r
61
1

9 (a) The sum to infinity of a certain geometric progression is 12, and tle sum of the
firsr three terms i. l!. rina
9

(D the common ratio of the progression, 121

(it) the least value of z for which lhe sum of the first n terms is more than 997o
ofthe sum to infinity. 131

(b) A fund is started at $1000 and compound interest is reckoned at 4olo per annum. If
withdrawals of $50 are made at the beginning of each of the subsequent yearg
show that the amount in the fund at the beginning of the (z + l)th year is

$r2so[r - ]r,.*r] tst

Find the amount in the fund to the nearest dollar after twenty years, and also how
many years it will last. 121

r0 The curve C has equarion , = "-' :::*' - lr is known rhar C, thar has asymptotes r = 3
utd Y=a -1, cuts they-axis at -2.
(D Find the values of q, b, c and d-
141

(iD Sketch the cuwe C, stating clearly the tuming points, asymplotes and any
intercepts with the axes. Hence deduce the range of varues of r such that
the
equation

ar2 +(b-k)x+c-kd =0
has two distinct real roots.
Fl
(iii) Sketch rhe graph of

=1

11
stating clearly the tuming points, asymptotes and any interceps with the axes . [31

[Turn Over
lr
I




ll (a) rina Jr3
cos(r2)a'. 131

(b) Use the substitution d=tan-l(r2) to find the exact value of


f

. 131

) Sketch the graph o f (x -l\2 + (y -Z)2 = t.

The region enclosed by the graph of (x-3)2 +(y -z\2 =l is rotated completely

about the r-axis. Calculate (, the volume of the solid formed. 141

The same region is rotated completely about the ),,-axis to form a solid with
volume I/2. Deduce whether Z2 will be larger than (.

End of Paper







1
q >






dVethC

Solution>
-9
>0
+2 o+2x 4)
X+l
>0
(x+2x -4)

1-l 4

--2<x<-l or x>4

l
2_9 2
2+1>
4(x4_ 2_2)

2 (2x2)_9
2)+2>
(2 (2,2)2 -212,12 -s
Thus,

-2 2x2 _1 or 2x2 >4


( tCd)

=12>2
= ,, J) orr. -J7

[Turn Over
2

Show th 2_3 2+ _1)Can be wdttcn h thc fonn 2_3 +1 2+ _1 lll


( + (

2 -1
Hence, or otherwise, find ) 2_3 +ix 2+ _1)
(

Use the above result to show that the sum to infinity



l l l l
+7+4252+
1222+


121
is less than l.

<Solution>

n2 +n-l-(nz -3n+1)
2_3 +1 2+ 2_3 +1)( 2+ _1)
l (

= Wn_
F
]:::li;;)
FlIT, Sh

-1
(

=:

2

2
-3 +1)(
2+ _ )
(

1 /11
1 / 1

5
/ /19
l

29

41

( - l)2 +(N-t)-l
I

,t+N{


2_ _1
2+ _1
]

=[:

2_3
For 3, >
2
+l and( +1)2: > 2+ _1

1 _



<1 (2 -1)
23 +lx 2+ _
( 5
t
:

l 1
l +3242+ + +_L 1(shown)

1222+2232 5 4 36

[Turn Over

3 Sketch the locus of ary/|l-221=L in an Argand diagram. Find the least value of lz+cl

where c=l-i. I4l


Find also the range of values of t, where t e 0
* such that there is exactly one complex

number z, that satisfies both lz+Q=* a d arg(l-24=A.


4
I3l

Solution>
arg(1-2z)= arg(-2xz
:)=
a (-2)+arg(Z
:)=1:
arg(Z =
:)=

l 1)

Locus of

arg(1-2z)=

From the Argand diagram,

To have exacJy one m cx number z thtt satisfy both, +J= and

arg(1-2z)=1:,from thc Argand diagram, =d:or d2

Now d2= =rL8o Lo =:


Or

4 (a) A sequence {:,} of positive numbers is defined * ,ru=*r^*3for z ll lt .

The sequence {r,} converges to a number 2 as z tends to infinity. Find the


value of 2 exactly. 121

Solution>

If {.r,} converges to 2 as z tends to infinity,

then rn+r:r. =, as a to infinity.

Therefore 2=X *z .
2).+3

Solving wegetz :-{a U"n"redsince-r, rOy o,


fr1

[Turn Over
6

(D A sequen (%) de6ned as l=3 and%.l= f r l+.Find the


+2+2
flrst four tcrrns ofthe sequen and show thatthcse tc.llis Can be wr n as

=2 f r =1,2,3 and 4 wherc and b are nttn"to be decrrnhed


+

+
With the vJues of and b found,prclvc that% by the method of

151
induction for PI 1.

Solutio >

= J =:
=

From thc above resuLs, =2 and =-1


" 2 l iC_

br
":
Ld L ttt
l+"
Statcme


= h Plls tr

Assum L -4=
To show P l ls truc.lc tl=21+1

Thereforc Pa.l is true_

P is truc l P l is truc.

Since PI is truc,by Mathemadcalinducjon P is truc for a!! 1,


l+


7

(a) Find tt r
dF

1)
y= n
)
121121


) G en that y` =( 5 )3,



in t s Ofx and .
131

Solution>

(aXi) y= = Ox+1 :



(2x+1) :(2)

3
2
(2x+

y=!n( 2+
+1) in(1+COS X)
)=ln(

= +

(C) y`y=( 5 3

+ -5y)2(1_5
` :F=3( )


dF (1+y)` +15( -5 )2

[Turn Over
8

The points S and I are the ends of the


diameter of a circle with centre O. U is a
point such that it lies on the circumference
of the circle. Taking the cntr O as the
origin, the position vectors of points S, I
and U are s, t and u respectively.
->
Write out the vectors SU and
-+
fU in terms
ofs, t and u and hence show that the lines
SIt and TIJ arc perpendicular. t3l

<Solution>
-+ -)
SU=A-s_ and TU=U-L
-+ -)
SU -TU = 4 -A- t' L
U. L- t' 4+

=lul' - a' L - <- L>' 1 +[l' l{cos(r 80' )


=lal2 - a' L* u' t+lul' l4(- r)

=0

Thus SU and IU are perPendicular.

Alternatively,

-+ -)
su-rU =(u-s_).@- L)
= (g -s)-(q +s)
2_
= =0
9

(b) Given that OAPB is a parallelogram and p is the mid point oflp. Op meets Be
I
atx. The position vectors of,{ and are a and b respectively.

(i)
-+ -)
Find OP andOp in terms of a and b. tzl
(ii) If R is the point on ^Bp produced such that rO = OR, show rhat R is on OA
produced and that OR:2OA. pl

Solution>

o
-+
(D oP --q+Q

-+r
o(2 = 4')*
)l1q. 4 = llzq * ay

(ii) Now. OO = OB+oR


-+ ll| /-) --) \
- 2\ )
I

-) _+ __>
+OR =2 O2-OB
-)
= on =2.!(za +b)-h =2s

-> =2
+OR
--)
OA

Since O, R, A are collinear R is on OA produced


=
Altematively, Bp produced such that Bg =
OR
-+ -+ -) -+
+BR =2 88.....+OR =2OA .

ffurn Over
10

(a) The diagram below shows the graph of7 = (.r). On a separate diagram, sketch a
clearly, the labeled graph of y = f(r) .


31

<Solution>
ll
(b) The functions fand g are defined as follows:

r'r*(r+zr)-3,,.-j; g,,-;!, -t<x<k


(D Find the maximum value ,t can take such that fg exists.


(it) With the value of t found in (i), frnd fg(r) and state its range.

(iii) Obtain the first three terms in the expansion, in ascending powers of .r, of
fg. State the set of values ofr for which the expansion is valid. 13

<Solution>

(i)For fg to exist,Rg DF

Now DF=( , :) Rg ( ,-5

Hencc from graph,max =1.

fg( )=f(:JLT)=ll+2(;J[T)] 3=(


)_3

,-1 l
( )

Using GC,Rfg=(H ,0)

[Turn Over



12

Oil)

fg(X)=(
) 3

=( _1)3(.+1)3

2+3x-1)(1-3x+( 3)(-4) 2+.


=(.3_3
)

2+.
=(x3_3x2+3 1)(1-3 +6 )

=-3x2+3 -1
_9 2+3 6 2 +_
2+.
=-1+6x-18

Expansion is valid for lxl < I

-l<x<l


13

0) A cylindrical silo with a hemispherical roof top is constructed to store animal fodder on a cattl
The cost of each unit area of the roof is fixed at P dollars
and the cost of each unit area of the cuwed surface of the

cylinder
'3 is fixea at lp dollars. The total cost of the roof
and the curved surface is a {ixed amount, C dollars. Given
that the radius of the hemisphere is r, show that the volume

Zofthe structure can be expressed *y ={r-4rt. I3l


2P3r-

S:

[CurVed surfa arca ofhemisphcre=2


2;volume ofhemisphere=:
r3]

Solution>
Cost for r f=SP pcr un area;
C u dhearca SIP

Total cost=SC.

Arca =2 2+2 (wherc is the hcight Ofthe cylinder)


2(P)+2
=) C =2 rk(:P)

2+
=' C =
)
(3

=(3 2+
)

=, = 3C _3
2 P

/= : 3+ 2 =2
3+ 2(

3 _3 )

3+lC _3
=
3
3 2P
3C 7 3
=
2P 3

[Turn Over
14

d/= C-7 2=

dr 2P
= t a)
(reJ

Vdume
= 0 m

Thus,cost ofhttnispherical top

2P
=2

=2 3C P
14 P
3C
7

15

o ry= (2x ,shW = +y2p By pcatd dffere on oftts



rCSultShOWthat ll =4[y:[ +3(:f)(li Hence flnd the MaC!a dn'S


)]
4. 161
expallsion of up to and including the tcrms in

Solution>

y=tan(2 )

80






[Turn Over
16

9 (r) The sum to infinity of a certain geometric progression is 12, and the sum of the

first thrce terms i, Z!. rind


9

(i) the common ratio of the pmgession, 121

(ir) the leas value of z for which the sum of the first n terms is more than 99olo
ofthe sum to infiniry. I3l

{olution>
==12 0)
1-


3)
(1_
= (2)
1-

3)(1_r)
1_ 3=
a(1_ 76 1
(2) l):
1-r 9 12 27

3=

27
2
=
3

S">

>



(:)
>0"
0 01
(:)


" 8

Thus icast value of is 12


t7

(b) A fund is started ar $1000 and compound interest is reckoned at 4yo per annum.

If withdrawals of$50 are made at the beginning of each of the subsequent years,
show that the amount in the fund at the beginning ofthe (z + I )th ygar ig

rrzsofr -]<r.orr].

Find the amount in dre fund to the nearest dollar after twenty years, and, also how
many yqrs it will lasf. pl

4olution>
Year Arntat bcginning of

(1)llXD
(2) 1000 + t96q6.64;-56 = 1000(1.0a)-s0
(3) u000(r.04)-501(1.(X)_s0 : 1000(1.04)2_50(1.04)_50

(z+t) 1000( 1.04r_ 50( l.M)*r _ .... _ 50(1.04) _ 50

Thus amount at beginning ofthe (rr l)h year


:1000(1.04f
- 50u + (1.04) +...+ (1.04)*rl

:l0oo( l.o4r-sof <rxr'oa'- Il I


L (r.04_r) l
:1000( 1.04r-l2So[r.o+, r]
-

02"
0[

A er 20 ycars, =20

Thus amount= 1250[1-:(104)20]=7022=702

FOr numbcr oF ycars, 1250[1-:(104) ) =41035=41 ycars


]=o

[Turn Over
18

2+ +
10 1he ctlrve C has equation y=` .Itis known that C,that has asymptotes x=3

and y = 5 - 1, the faxis at -2.


"rts
(r) Find the values of a, b, c and d. 141

(iD Sketch the curve C, stating clearly the turning Points, asymptotes and any
interceps with the axes. Hence deduce the range ofvalues oftsuch that the
equation axz + (b - k)x + c - /ry' = 0 has two disfinct real roots. 151

the gratt J dtty



asymptotes and any intercPts with the axes' 131

<Solution>

00y=

+ (b
ad)
+
F
)
_( 2 +
= =

(b )X +
I(b )X + ( )
(b )

ttC y= +0-0+ 0

Given asymptotes,,=3,y= l and whcn =0,y=-2

Gy=J+

Comparing widl(1),

=L =-3,b =-1 =-4;

Whcn =0,y= , = =6

19

Thus C:y= -1+


/t

(1_27,-1.46)

2+(b_
) +c =0

2+ +`=.

For twO distinct real Ots,thc linc y=t rnust lic abovc thc!inc

=5 46 and be10wy=_146_

Thus I>5_46o -146

[Turn Over



20




//
.73,5.


1.27 1.46) (1:27r146)





21

1l (a) Eina
Jr3
*r(r'z)ar.
131

( Use the substitution =tan 1( 2)to nnd the exact value of

131

(c) Skerch the graph ot (x-3)2 +(y-Z)2 =t.

The region enclosed by the graph of (.r-3)2 +(f -2)2 I i, rotared complerely
=
about the.r-axis. Calculate the volumeof the solid formed.
4, 141

The same region is rotated compretery about the y-axis to form a sorid with
volume Y2. Deduce whether Z, will be larger than (.

<Solution>

Let ==
2
, =2x

Let 2)=:sin( 2)
`=,cos(

Jr' "or (r2 ) a, = Jr2 d,


.
cos1.r2 I
'
= |:2 sin(.r2) - J.rsin1.r2;a:
= j12 sin1,2) -[-]cos1x2y] +c

= jr2 sin1.r2) + jcoqx2l +c

e = w-t (r2)+ tatto = x2

DFwi
Difpwrr .,s 2
=2,
"*zo_.,,_dO ^ dx
sec2 =2x
-^^^2^
d,r sq,2Ze
=S
dr d d 2x

[Turn Over
22

When.r-{), d=0 and whenr:\ e=i.

fl ,, .Y'o ae


=
Jo Jt*t^'e 2x
d ot
lo "nt
ln6*e+tne\(
h(J2+l)-ln(l)
h(..6+l)

_2)2=1
(x-3)2+(
=)( 2)2=1_( 3)2

+z
=y=trlt-(,-r)2


=uf,'fi-{'-t\'*
=39.48
39 5 unit3

V, will be larger than \ because the radius of rotation is larger'

End of Paper
SERANG00N JUNIOR COLLEGE
2007 JC2 PRELIMINARY EXAMINAT10NS

MATHEmTICS
igher 2 974MPapr2

Tucsdey - 18 September 2007

Additional materials: Writing paper

List of Formulae (MF l5)


'hours
TIME : 3

R.EAD THESE INSTRUCTIONS FIRST


Write your name and class on the cover page and on all the work you hand in.
Write in dark or black pen on both sides of the paper.
You may use a soft pencil for any diagrams or griphs.
Do not use staples, paper clips, highlighters, glue or correction fluid.

Answer all the questions.


Give non-exact numerical answers corrct to 3 significant figures, or l decimal place
in the case
of angles in degrees, unress a different rever of aciuracy is sfuinea in the question.
You are expected to use a graphic calculator.
unsupported answers from a graphic carcurator are alrowed unless a question
specifica y states
otherwise.
where unsupported answers from a graphic carcurator are not allowed
in a question, you are
required to present the mathematical steps using mathematical notations
not
commands. "nd "ui"ul"to,
You are reminded ofthe need for ciear presentation in your answers.

The-number of marks is given in brackets


I rat theend ofeach question or part question.
At the end of the examination, fasten all your work securely together.

This question paper consists ofg printed p"go oo bl"ik pages.


"nd

[Turn Over
Section A: Pure Mathematics [40 marks]



The curve C is defined paramehically by x=aw,O andy=a61r6, where a is a fixed
constant. It is known that the point P with coordinates (as*o,atana)lies on C.

Show that the equation of the tangent at P is given by x-ysin9=acosd and the
equation normal at P is given by x sin4 + y 151
=2stang .

The tangent and the normal at P cut the r-axis at I and 1V respectively.

Prove that OT.ON =2a2,where 0 is the origin. 131

Find the modulus and argument of the complex number z,


vherc z=
(a)
/'



giving your answers in exact form-




Hence, using de Moivre's theorem, evaluate z6 .

(b) Show that for any complex numbered, *=:'+(*.*), 121


e -t - &\


Write down the solution to the equation ,5 -l=O leaving your answers in

the form re'd where r > O and -zt <0 < zt .


21

Hence, find all possible values of the complex number w that satisfies
* r+yi where:,ye0
| | =t,leaving your answers in the form . 131
\rv-ll

3

3 A differential equation is given by

9: P(')y+QG)y' .

(1) ForP( )=4 and 2( )=4,flnd thc gcneral solu on ofthe difFerential equation
when =3.

I1704
`ar _ aS as cr I l 151

(li) ForP( =land2( )=i W,using the substitution y=` ,the


I 'Sh
difFe" d"utton can L rcduccd o =
WhCn =0
131

Given that y = Q vvhen .t = l, solve the differentiat equation giving y explicitly in


terms ofr in your answer. t2l

4 The line /r and the plane zr have equations r=2i+2i+ pk+.l(i+qk) and
r.(i+ j+k)=2 respectively. Given that /1 lies in the plane zr, show thatp:_2 and find
the value ofq. Verify that the pointl(_I,2, l) lieson /;. I3l
Find the equation of the plane z2 which contains the origin and intersects z; at the line /r. [31

Given thal the point B has position vector -i +3j+2kand plane z3 has the equation

r =(2i+j+3k)+s(i_k)+r(3i_j+2k) wheres,r e[ ,

find the tength of projection of 7 onto the plane r3. 131

By finding the solution of the following equations,

x +Y+ z=2
x+z=O
x+5y+z=10,
or otherwise, give a geometrical explanation on the results you have obtained. I3l

[Turn Over

Section B: Statistics [60 marksl

,{ The random variable X has a normal distribution with mean p and variance 4.

Given P(Y 5)=P(r>31)

O Showthat ,-t+- 12

Given that P(x > s)=6.3a13, find the valueof&.


41
ftt
fft Given that XyX2 and X3 ar three independent observations of X.

21
Find P(x, +Xr+Xr>2o).

/' Three boys Alex, Benjamin and Charles agreed to meet at the theatre. AII three boys
forgot which of the theatres, Palace, Queen's or Royalty to meet at. Alex tosses a fair
coin to decide between the Palace and the Queen's. Benjamin tosses a fair coin to decide
between the Queen's dnd the Royalty. charles tosses a fair coin and if it is'head, he goes
to the Palace, otherwise, he tosses again to decide between the Queen's and the Royalty.

Find the probability that

4f Alex and Benjamin meet, tU

Benjamin and Charles meet, l2l


-b>
121
li*, all thre boys meet,

. !9 alt three boys go to different places, 12t

ltl at least two boys meet. l2l



f The table below gives the observed values of bivariates x and y.
,A
20 30 34 35 36 40 42

32 25 22 26 18 19

It is given that the equation of the regression line y on : is y = 43.5 - O-6fl2L


Find the value of a correct to the nerest integer. l3l


Write down the equation of the regression line r on ), and the value of
product moment correlation coeflicient between x and y.
I2l



OTll(
:) The fO
Owing are the summary Ofsix pairs ofvalues ofthe variablcs x and :

lx=tt,Zy=ztq, lx2 =rue2, andly2 =9274.


r=9 o*X
It is given further that when the estimated_value of yis37.\ tr S.\
Find the equation of the regression tin", on r. Isl
\s
r'!":s+>'*.;

4 The amount ofdonation collected by each student in a particular college is


$x. Data is
collected from a random sample of50 students and the results are summarized
by

Ix = 4O00 and Ix2 = 360250.


g Find the exact value of the unbiased estimates of the population mean and the
population variarce ofthe amount ofdonation collected byihe
studenS. pl
/
state clearly the distribution of the mean amount coilected by the students, giving
W
-

rts mean, vanance and any assumptions made. I2l


Find the probability that the mean amount collected exceeds $g5.
121

Find the smallest possible sample size if the probability that the mean amount
exceeds $85 is less than 0.03. l4l

[Turn Over



/g _Oft a student's homework solution on hypothesis testing for a random variable Xwas
found to be in a bad condition as shown below.


The test conducted used a sample size of 50.





i.c- ruSecl ll. ...rt'en Ztu+ 7
Z+.+ <

G du 10n

Sincc Z =2 :21 ife 10 J.._a dr o,

i ( Tr

Find the sample mean, leaving your answer correct to 3 significant figures' [21

Find also the least value of the significance level such that the result will
be in accordance to the student's conclusion. t3l
I

,{ 9{ fn principal ofa private college claims that graduates from his school have an
average sta(ing salary of $2050. A private body checks the claim by interviewing
a random sample of60 graduates from the school.

The data obtained is summarized bclow, where r denotes the monthly salary per
person.

l{, -zooo) = zt+o ana f, {.r - zooo)2 = 162001

Carry out an appropdate test at the 3% significance level to determine whether the
principal is overestimating his claim. t5l

10 (a) According to the school rules, a student who arrives at school after 0730 hours is
considered late.

During the school term, a boy cycles to school on 5 days each week. On
any given day, the probability ihat he arrives after 0730 hours is 0.1. For a
period of4 weeks, calculate the probability that he is late on at least one
day but not more than 3 days. 121

When the boy has cycled to school for the nth time, the probability that he
arrived at school late at least once is greater than 0.99. Calculate the least
value ofn-
l2l

A girl travels to school by bus on 5 days a week. Overa long period of


time, the variance of the number of days per week on which she is late is
0.8. Given that p denotes the probability that she arrives late and that
p < 0.5, evaluate p. l3l

[Turn Over


The Royal Sports School runs an intensive training programme to groom sports
talents. The school believes that when the students participate in the annual
national sports heats, 807o of them will qualiS for their events.
Among those who qualifo, 97Yo of them will eventually be in the top three
positions and the rest will receive consolation prizes.

100 such sfirdents attempted the sports heats. Using an approoriale-


*approximation, find

the pmbability that the number that qualifies is at most 90 but no fewer


than 70.

the probability that at least 5 will quali$ and receive consolation prizes.



End of Paper
2OO7 JC2H2MATH PRELIM PAPER 2

I The curve C is defined parametrically by x: asrre and y =s1a16, where a is a fixed


constant. It is known that the point P with coordinates (asecd, a tan d) lies on C.
Show that the equation of dre tangent at P is given by r-ysin9=acosd and the
equation normal at P is given by.rsin d + y = 2atanq - 15

The tangent and the normal at P cut the.r-axis at I. and ly' respectively.

Prove that OT -ON :2^a2,\Nfue 0 is the origin.


t3t
<Solutions>
C: x=asq,0, y=atanQ
!=on et^ne and !Z=a*:e
de d0

' d.r asecorano=.*r=(*)


-dy-asec2o -t -^^.--o
trna-l11fr=a=*""'0
( cos d./
At P( P(asecd, atan9) ,

Equation of taneent: t- aulne = l=(: -asocd)


+ ysin0 - asin9 tan 0 = r - asq?
|
= x-ysinl =asecl-asinlane =o(\cosd-rinB.tind)
cos? )

=_3=(r_.i"ra)
o
=cos d -*"2e
+ -ysin d = acos d (shown)
.x

Equation of normal: y - atm? = -sine(x - awe)


=-xsin0+atan0
) xsin9 +y =)s tand (shown)

Aty:0,
r = acnsO (from eqn oftgt) T: (acosd,O)
i.e.

rsinl =2atanl *r(freqnof


=, =cosd ' normal)i.e.N=1-?a
tcosd ,g;

oTxoN =acosd .
2o:=2o2
(rho*n)
cos d

[Turn Over
) FInd the moddus and argument of the mp x number z,where z=(1-
5)4'


giving your answers in exact form.


Hence, using de Moivre's theorem, evaluate z6 -

Solutions>

=
arg(Z)=argl j)-4arg(1-j)
]=2argKl
=2( : )-4( ) (Can use g_c)

Thus z=:(coSi +fSini )= Z6= F+iSini )6


(coS


)z6= :(cOS5
+isin 5 )=

:

( Show that For any compiex numbera: , =: 12


:(COti:)

Write down the solution to the equation z5_1=O ieaving your answers in

thc form ' where r>O and 121

Hence, find all possible values of the complex number w that satisfies

(*l =l,leavingvouranswersintheform :+vi where'r,veD' I3l


<Solutions>

(` -1) 1-`
`
-1= -1)


-1)= 1-(
+

( )+l
I - (cosA +isind)
2-2cos
I . sin
=
2 2(1-coS )



=:
:(COt:)J (ShOWn)

z5_l=0 z5=l= (21 ) z= 5 ,


=0, 1,2

2`

5 , 5

Lct Z= =OW=
-l z-1

Th = =


5 _1
=::(C
)
=0, l,2

=05 0688 ,0_5 0.162

[Turn Over
4

A differential equation is given Ay


{= 4t7y QG)y"
+ .

(i) ForP(r) = 4 and Q@) = 4 , find the general solution of the differential equation
when z = 3. lYou need not expras y as the subjed in your final answer.l 15

(iD For P(.t1 = 1 and Q1x1 = -J-, show, using the substitution y = ,tx , the
x l+ x'
differerrtial equation can be rcduced to
q=-f= when n = 0.
dx l+ xz 131

Given that y: 0 when .r = l, solve the DE givingy explicitly in terms of .r in your


answef. 121

Solutions>
3=4 dy
(1) ::=4 4 (1- Xl+y) =4
(1- Xl+y)

Let, y(1-
Xl+ )=:

+I.y and by cover up mettod,
11

=L =:,C :


=4
+

h
:h J :h
+ = +q
2hlJ h h + =8 +Q
h 2_h _ ttC2
11+ =8

in =8 +C2

):f=: +
fi


C)

When =zr = +

_ 5

From(1), : + = +
:

l + d

+
T 7=
= Wn)
7 (Sh

muS,ld =

=tan 1 +

When 1, IIl( n y= )

0=tan 11+` c=


=tan-l

y= tan l

[Turn Over

4 The line /r and the plane zr have equations r =2i+2j+ pk+2(i +gk) and
r.(i+ j+k)=2 res@ively. Given that /1 lies in the plane rr, show that p =-2 and find
the value ofq. Verifo that the point A(-1,2,1) lieson 11. I3l

Find the equation ofthe plane z2 which contains the origin and intersects z1 at the line rl. l3l
Given that the point I has position vector -i + 3j + 2k and plane z3 has the equarion

r = (2i+ j+3k)+s(i -k)+(3i - j+2k) wheres,r e D,

--)
Iind the length of projection of lil onto the plane 23. I3l

By finding the solution ofthe following equations,

x+y+z:2, xl z:O and x+ 5y+z=10,


or otherwise, give a geometrical explanation on the results you have obtained. I3l

<Solutions>

[=:+'l and l:

SinCe rlileS On l )lil lll=2 ) =-2 (shown)





of fI2


Normal is



ThuS 2: 1

111=0


Thuslength OFproJcctiOn Of 3 0n
3= x


[;=

+ +z=2, x+z=O and X+5y+z=lo,intcrsccts at rl

Ottettisc mettOd

[Turn Over











('



atl intersects on the line



x + z -- 0.

fI,

p=z,a=-P

Solvine method

and

y:2,

+fll,fl2









Thtts

Let



I

Section B: Statistics [60 marks]

5 The random variable X has a normal distribution with mean I and variance 4.

Given that e(x < s) = n(r > rr).



Showthat ,=1!f .

Given lhat P(x > s) = g.8a 13, find the value of*.

Given that X1,X2 and X3 are three independent observations ofX.


Find P(X, + X, + X, > 20) . 121

<Solutions>

(i) Since P(x <s)= p(X >3k'),

p is the midpoint of 5 and 3t, ;. p =5 *=3k


z

(iD e(x>s)=s.3413

p(x<s)=o.ls87

P(z .-s-F\=o'tsst
\ 2)
5_O
2
=_,
It =7
"2. 5+3* _,
k=3
(iii) X,+Xr+Xr- NQt,tZ)
p(4+x2+\>20)
= 0.6136 = 0.614

[Turn Over
t0

y Three boys Alex, Benjamin and Charles agreed to meet at the theatre. All three boys
' forgot which of the theatres, Palace, Queen's or Royalty to meet at. Alex tosses a fair
coin to decide btwen the Palacrc and the Queen's. Benjamin tosses a fair coin to decide
between the Queen's and the Royalty. Charles tosses a fair coin and if it is head, he goes
to the Palace, otherwise, he tosses again to decide betrveen the Queen's and the Royalty.

Find the probability that



Alex and Benjamin meet,
Benjamin and Charles meet,
' irll three boys meet
all three boys go to different places,
at lest two boys meet.

<Solutions>

and B nlcc =P( ,321=:X:=:

O P(B and c meet)=P( ,%)+P(3R,CR)

l l l l l
= X + X =
24244

(lli) P(all mCCt)=P(all gO tO Q)

1 l l l
= X X =
2 2 4 16

0V)P( gO tO drerent pl es)=P(4 CR)+P(/QttC )+P(43R%)

=:X:X:+:X:X:+:X:X:=:

0 ieast 2 boys me =1-P lgo dttrcntphe =l


:
3
4
l1

9 The table below gives the observed values of bivariates x and y.


*ry[sl;'**
20 30 34 35 36 40 42

J7 32 25 22 26 18 19

It is given that the equation ofthe regression lineyon:.is y, 43.5-0.6C12

JiY of

Find the value a correct to the nearest integer. 131

gl Write down the equation of the regression line .r on ,, and the value of
product moment correlation coefficient between .r and y. pl
<Solutions>

(i) r=33.857, t=!!#


Substituting in the given regression line, we have

l4L* o
= +l.s - o.6o2x 33.857 = 23.1 r

a : 19.82 Z 20 (nearest integer)

(iD Using GC,

Regression line.ronyis x: 64.349 _ l.3lgy=.r: 64.4 _ 1.32y

and r=_0.89t (3 s.f.)

Altemative answers , when a = 19.82 is used:

Regression line.ronyis x= 64.087 - l.30gy=+:= 64-0g7 _l.3ly


and r = _ 0.88755 = _ 0.888 (3 s.f.)

[Turn Over
12

7 (b) The following are the summary of six pairs of values of the variables r and y:

l-r=zt, Zy=ztq, lr2 =vaz,analy2 --ezt+.


It is given further that when .r = 9 the estimated value of y is 37 .

. Find the equation of the regrcssion line.r on y. 151



<Solutions>

S- =)x'z- =448 = y2_ =148

,=13, y=39.
Regression equation yon.r, y-39=b(x- l3)

Subsdtu and =37 and so e, b=05

ie Iz Therefore S-. =0.5xM8=224


s- =0.5
Regression equation -r on y is

x-n=Z!1r-391
148 "
, x= r.5r4y-46.03.
.13
The amount ofdonation collected by each student in a particular college is $.r. Data is
collected from a random sample of50 students and the results are summarized by

Ix=,1000 and f.r2 =360250.


Find the exact value of the unbiased estimates of the population mean and the-
population variance ofthe amount ofdonation collected by the students. l1l
State clearly the dlstribution ofthe mean amount collected by the students, giving
its mean, variance and any assumptions made- t?ll
(lil) Find the probability that the mean amount collected exceeds $g5. t.l
(iV) Find the smallest possible sample size if the probability that the mean amount t-'
exceeds $85 is less than 0.03. 14
<Solutionp

Unbiascd ttmtte J cmtt


= =
UnbiaSed cstimatC OfthC Variancc=S2=
(360250- )

= =8211
7 7

(ii) Since z = 50 is large, by Central Limit Theorem,

* - lrfao,T] approximately.
\'7)

= 0.109 (3 sig. fig.)

(V)Lt be the salnp Jze

(Q7) pttmady

[Turn Over


14











l5

(a) A student's homework solution on hypothesis testing for a random variable Xwas
found to be in a bad condition as shown below.

The test conducted used a sample size of50.

l5 , l'1t
l4t: )t a.as
2
U,r.. rb, i - r-r ( zS I
:r"
)

dj.d x. @Li Zrr >


Ztor (
G..r,.q
S!.. Zl<! . 2r1l li. i" tt -t;d .qin,
-e ...i.d S. &n q h.{ s"tl-:.J
--\"y*rJJ t - s.i {r. ,.?rt

(D Find the sample mean, leaving your answer correct to 3 significant figures. pl

(iD Find also the least value ofthe significance rever such that the result wi
be in accordance to the student,s conclusion. 131

(b) The principal of a private corlege craims that graduates from his schoor have an
average starting salary of$2050. A private.body checks the ciaim by interviewing
a random sample of60 graduates from the school.

The data obtained is summarized below, where -r denotes the monthly salary per
person.

l{,-20oo1=2740 and
ZG-2000)2 = 162661

Carry out an appropriate te$ at fte 3% significance level to determine whether the
principal is overestimating his claim. I5l

[Turn Over
16

Solution>

2=52=25 where X
CaXi) =50 civen)
(25,

Z. = =2.121

>i =26.49977 = 26.5 (3 s.f.)

(1) TO t HO,Ztcst must ile in dlc cHticai region

Now P(Z -2121)=nollilCdf( E99,=2.121,0,1)=0016961

Thus 2 >0016961=O >339%


100

(b) Let Xbe the random variable starting salary ofa graduate.

Ho': P = 2656
H1 : p < 2050 (principal is overestimating)

(test at 3% significance level)

Un rH
(
byCmtt bmmTh m
-21X10)+20CXl=
F= (
+20 =204566667
60 60


17

,2 = -2KX10)2_12 il
](
:
= I

=591162flC11-21
]
=624.98870

Using g.c. Z*r= -l .3425 and p-value{).089693


Since p-value > 0.03, we do not reject Ho . Here we have insuflicient


evidence at 30% significance level that the principal is overestimating his
claim.

[Turn Over


18


10 (a) According to the school rules, a student who arrives at school after 0730 hours is
considered late.

() During the school term, a boy cycles to school on 5 days each week. On
any given day, the probability that he arrives after 0730 hours is 0. l. For a
period of4 weeks, calculate the probability that he is late on at least one
day but not more than 3 days. 121

When the boy has cycled to school for the zth time, the probability that he
arrived at school late at least once is greater than 0.99. Calculate rhe least
value ofn.
21

(ii) A girl travels to school by bus on 5 days a week Over a long period of
time, the variance of the number ofdays per week on which she is late is
0.8. Given that p denotes the probability that she arrives late and that
p < O.5, eYahnte p. 131

<SolutionP'
O tetxbe the random variable the no. ofdays a student is late in 4

P(1 3)=P( 3) P( =0)


=binomcd 20,0.1,3) binOmpd 20,0_1,0)
=0.74547
=0745

For no ofdays, r 3( ,ol)

P( 1)>099

= 1-P( r=1)>o99
P( =0) 001

0 01
(:)(0 1)0(09)"
=o in(09) in(o ol)

>h(0.0:)=43.70869
in(0.9)

Thus the lcast vaiue of is 44

(D r U" ttt" .anao. variable the no. ofdays girl is late in aweek

-L"t
19

y O, )

Varia =4 =0 8

5 (1- )=08

2_ + .16=0
=0.8 or O.2

Sin 0.5, =0.2

[Turn Over
20

10 (b) The Royal Sports School runs an intensive training programme to groom sports
talents. The school believes that when the studen6 participate in the annual
national sports heats, 807o ofthem will quali$ for tlreir events.

Among those who quali$, 97Yo of them will eventually be in the top three
positions and the rcst will receive consolation prizes.

l0o stch students attempted the sPorts heats. Using an appropriate


approximation, find

(i) the probability that the number that qualifies is at most 90 but no fewer
lhan 70, 131
(iD the probability that at least 5 will qualify and receive consolation prizes.
131 -
<Solutions>
(i) I-etXUette random variable the no. ofstudents qualifying out of
lClll

3(100,0_8)
Sincc =1001s!arge,4 =80>5,
`=20>5
Thus oO,16)apprOXimately

r 9o)__ )P(69.5 905)


P(70

=norlncd 695,905,80,)

=099133
0991(3sl)

(11) Lct ybc thc random variable the no ofStudents qualifying and
rcceiving conso:ation pizes out of 100
P(SuC SS)=0.8x003=0024

B(lllll,0_024)
=llXl is iarge,4 =24 5,
/ Pa(24)apprCIXInately.
P( 5) =1-P(y 4)

=l poisoncdt2 4,4)
=l-090413
=0_095867
=00959(3sl)
Candidate Name : CT Grotp :
lndex no :

PIONEER JUNIOR COLLEGE


JC2 Prclim in a ry Examinations

MATHEMATICS
Higher 2 97lo
Tuesday ,t16 Sept ZX,T

Additional material: An$xer paper

TIME 3 hours
INSTRUCTIONS TO CANDIDATES

Do not open this booklet until you are told to do so.


Write your full name, index nunber and CT group on all the work you hand in.
Write in dark blue or bhck pen oo botr sides of the papers.
You may use a soff pencil for any diagrams or graphs.-
Do not use staples, paper clips, h-tghttghters, glue or conection lluiri.

Answer all questions.


Give non-exact numerical answeE conect to 3 significant figures or 1 decimal place
in the case of
angles in degrees, unress a differen ever of accrracy is sp6.in"a in ttre quesriln.
You are expected to use a graphic calculato (.
Unsupported answers ftom a graphic calculator are allored unless a questbn
states othe.wise.
where unsupported answers.ftom a graphic carcurator are not aflowed rn qrestion, , you are
required to present the mathematical Jteps using mathematical notation
commands-
,il-;;tcalculator
You are reminded of the need for dear presentation in your answers.
The number of marks is given in brackeis [ ] at the enoLf each question
or part question.
At the end of the examination, arrange your solutions in numerical order
and fastLn them securely
together with the question paper.

Itrarks Marks
1 8
2 9
3 10
4
5 12
6 13
7
SubtOtal Subtota:
Tota:

This questioll paper consists Of 6 printed Pages.


PJC2007
[Turn over
1 Solve the simultaneous equations
i Z=-1-i, 2z cl w= ,

giving each answer in the form a+bi,rhercaui b are real. [3

had eB = rad % B=: , = and/C=ac en that


3 can be igno"t show that
is smali such that powers abovc


"[ )
[2]


Given that a: I cm and 6:3 cm, find the value(s) of d.

Comment on the validity of the value(s) of I .

3
- Find Yl,-r-rl
""- Al -!--l)l
- -"(r-r r/l ," terms of z. [4]

Hence state the value(s) ofz for which the sum is zero. [11

4 Expand
^
a series in ascending powers of-r, up to and including the tcrm in
,E
x2 . State the range of values of; for which the expansion is valid' [41

By taking x=:,use the.rst hee no rrns ofhe se es b es dc , Vhg



your answer zrs a fraction in its lowest term. [2]

A cylinder of radius r and height fi is inscribed in a sphere-of fixed radius a so that all
the ioints of the circumference of the base and the top of the cylinder are in
contact
with the inner surface of the sphere.

Show that the volume, V,of thecylindercan be expressed : l2l


^' "("0-l)'

Herrce find, in terms of a, the exact maximum value of Zas i varies' t5l

ITurn over


4

rhe rine/, hasequation .=[1J.,[:] ,re0 . rhe line/, passes through poins ,4 and

I with position vectors 3j +3k and qi +3j +5k respectively, where p and q are
constants.

(i) If q=f and p=4,, find the position vector of the point Con ( such thatl isthe
foot of perpendicular from C to/r. t4l
(ii) Ifthe acute angle between /, and /, is60", find the possible value(s) ofg. t3I

The graph of y = 11r; is shown below. There is a maximum point at the point ,{ with
coordinates ( I, -l) and the asymptotes are x = 0, x=2 and y=6.

A(t,-t)

Sketch, on separate clearly labeled diagrams, the graphs of


o y=2f(,+t), t2l
(ii) y'=t(*)+2, t31
(iiD y=f '(r),where f '(x) is the derivative of f(r). t3l

A curve has parametric equations x=l+2sind, y=cos9,nhere -LSe<L.


(i) Sketch the curve, labeling clearly the intercepts with the r-axis. I2j
(ii) Find the exact area ofthe region bounded by the curve, the.r_ axis, the y-axis and
theline;=2.
t6I

[Turn ovcr
9 Find
O



(Sinx)h(COS X)dX,

(b)

t0 The functions fand g are defined by


+6r-9, re0 -,
f :.r-)x2
g'.x'+
t^ a>0,x+a,xeO,
oz_r2,
h,, --s2-J3r-1, x> k.

(D Sketch, on separate diagrams, the graphs of y=f (x) ana y=E(x)- l2l

(ii) State the maximal domain of g for which its inverse function exists. tU

(iii) Find the range ofvalues ofa for which the equation h(x) - g(x) :0 has two real
roots, giving your answer in exact form. 13]

(iv) Find the least value offt for which the frmction Ih exists. t31

1l The quantities,and arc related by the difFcrential oquation

i
l=
di
.

By using the substitution =z ',Show that the general solution of hc equation is

Y=l-x+Ae',
where I is a constant. I5l

Hence sketch the two solution curves passing tkough the poins (t,0) ana (O,Z)
respectively. t4l

lTurn ovcr
6

In a single Argand diagram, sketch the set of points representing all complex numbers z
satisfuing both of the following lnequalities:

+3+

1 0 argCZ+3+
.
[6]

Hcnce 5nd the range of argcz+1+ )and the maximum vJue of +1+31. [5]

13 Sketch the graph of = ,ShOWing c!early on your graph the exact cOordinates Of

its tuming points and the equations of the asymptotes.
ts1

(D Find the equation ofthe tangent to the curve at .r =


-3. t31

(ii) Explain why the tangent to the curve at all xe0 \{l} will cut the graph exactly
once.
I2t

0
dn"
1
>
[3]

End of Paper
21X17J2Prelim Paper l sol d
l i +z=-1-i (1)
2z (1+1) =6+2i (
4 ll al

F m(1),Z= i -1-i (3)


Sub O)intO(2)
-21 -2-2i iv=6+2i
-3i w=8+2i
8+2
=



-1-3


"=-2+21: z=1+i



2- Using sine rule'

[
+:
sind sln. 1r
-6
=1+ 2

a = 2bsin9 2 8 :


="(,-+)


= 1+

hOm) f()


=2ba






=lcm and b=3cm,


Substitute



]
+1 0



=-253,0 167 or 2 36 rad
Sincc can ot bc ncgativc,rg t =-253
ltls assumod that ls ttal:,reJ : =236
0nly l possiblc answer For ,ic _167
Thcrefore, =
nC" e arls o

3 -2-
I= 1
:(:)+ ( )
[ 5

=2-2 -
=:+ +
+3-2- n e =3 =
mttr a wc

+4-2


*1^-21 -z-u(

.1"-r1 -z-2"( 1

+ n -r-U(

=
\)La
* (, - z))-r(, - I
) - r"lr - f
]
l-1:)
/= 2
= =
2 _4;) hOWn)
( (

=ll,-zl-zr,.-rt

n3 =
2_
_ (r- lXz-6) ( )

2
h-ll{n-6\ #=0 (a2_ )=0
,2 tozrro =
Sumequal " '=0
-n=lotrn=6

(
iatteCt
)

Since a> 2-+ z = 6


Substiauting back into ,/,
(ii) y' = f(1)..2


O When

n mttmum
eFore,/ = `

cD


(iii)y=F ( )




E. Using GC in pcramctric stting

Changc WIN W setting to

m=:,T =

Graph wi:l bc thc` op''halfofan


cllipsc with hori ntal dialnctcr 4,
(11) B= :+2k vcdi l dlarnetcr 2,ccnter at(1,0)
G en acute anglc bctween 4 and
,


When x=0, = :WhCn,=2, = , =2 s

cos 6Oo = Arca und""ph=f


=[( S )(2 )

JI,IV *4 =2q - q =fl = (2 s2 )


7. i. y =2f(x+t) =[(
"+

=[L;22+
]: =2:]+:; unitS2

q t") I sinrln(cosr) dt



Suppose graphs intersect at
trt r=lo(cosx); v=sinr
dz _ -sin x
f*=-oos,
=2=--l-
d, cosr
["' - l,)
J sin x ln(cos x) d nt
_"=r{,,
= [(-*.,)(tn (*',))] -l(ff)c*',r* checking graphs with GC;
NT
=[(-*.4(t"(*"))] -Jsinra
"r{r,
= [1-...,)(r" (-.,))] +cosl+c

0
=:
iv)Orexissif&94=n-
Largat ( =(-o,0)
Solve h(x) :0,
=:

k=-s
= 3


:
: F

-. dv dz
y=?-r=,--1---l
ll. -dxdr

= .rn. ldv x-2



Substrtutrng |=
-=
y t, y

-
--t-(4-,\-,=,-,

(:-r)\dx (z-x\

(dz \
l- j-+r=r-2

|
--l )
\dx

dz
_=z_l
dx

l t a,=fia,
J---l !
lnlz- r+s,
ll = r',/hgre c is a constant

lz - rl= e"'
z = Ae' +l ,where A = !e'
y + r = Ae' + I y =I -r+,4e' (shown)
=

When curve passes through (1,0), A=0 = y =l-y


(ii) Maximal domain = (-@,01 o. When curve passes through (0,2),,{ = I
[0,co) = } = | - r +e,
(iii) g(r)-h(x) = 0= s(r)= h(r) (Notice thatj/ = I -r is an oblique asymptote for
Sketch the graphs ofh(r) ard g(t) on the same diagram. y=l-r+e')
For ho)tO be deFlncd, > =:,h( )=2
:W

.v = t(r)
12 lz +:+ ril <
lr
+ Jzl = l, -(-:-:i) < J: 0
=3 t=:,



=1- =:
0 arg(z (-3 )

Im`z
Eqn:
:)=:( (-3))(or uSingy="r+
)
(
7 1
=T +I

J tt AsI ,

4-;-
11r...6o."-'dx

Thcrefore, any tangant to the curve at the points whe.e


x>l+Ji and r<l-16 will have a gradient -+ l.
To nnd rangC oF arg(z+1+31): Thcrefore, ir will not inters6t the curve again.

sin = =
2 3 At rhe region where l-J2<r<l+J2, r +l,the
gradient of the tange'nts will tend towards --o at the left of
a (z+1+ )
the asymptote and tend towards cD to the right ofthe
asymptote. Therefore, it will not cut the curve again as
well-
MaX +1+31
2 (See graph)
By cosine mlc,

PO'=( )2+(2)22( )(2) s()

=3+4-2(V5)(2)(
F)
P2 V u ts

= J
=

Asymptotcs arc y= tt l and =1


Sketch the graphs ofy = lf(r)l andy = r +3

(il 2=0 I=1


Turning points are (t+Jz,zrzJi) u"a

l-Ji.z-zJi).







h. -li c 't.,

(l ,22 Solving for points ofintrsation by GC, r = -1.618,


0.61E or 2.
From graph, r<-1.618 or 0-618<r<2, r+1.
l

- Erd ofPaper-

Candidate Name : CT Group :
lndex no :

P10NEER JUN:OR COLLEGE


C2Perrmfaa namhatbps
MATHEMA CS
Higher 2 9740

Tuesday 18ul sept 2007

Addltionai matettl: Answer paper

TIME 3 hours

INSTRUCT]ONS TO CANDIDATES

Do not open this booktet until you are told to do so.


Write your full name, iMex number and CT group on all the work you hand in.
Write in dark blue or black pen on both sides of the papers.
You may use a soft pencil for any diagrams or graphs.
Do not use staples, paper clips, highlighters, glue or correction fluid.

Answer all questions.


Give non-exact numericl answers correct to 3 significant figures or 1 decimal place in the case
of angles in degrees, unless a different level of accuracy is specified in the question.
You are expected to use a graphic calculator.
Unsupported answers from a graphic calculator are allowed unless a question states otherwise.
Where unsupported answers from a graphic calculator are not allorved in a question, you are
required to present the mathematical steps using mathematical notation and not calculator
commands.
You are reminded of the need for clear presentation in your answers.
The number of marks is given in brackets { I at the end of each question or part question.
At the end of the examination, anange your solutions in numerical order and fasten them
securely together with the question paper.

Marks Marks
1 8
2 9
3 10
4
5 12
6 13
7
Subtota: Subtotal
Tota:

This question paper consists of l0 printed peges.


OPJC2CX17 [Turn over
\;

Section A: Pure Mathematics [40 marksl

The 3'd, 4m and 66 term ofa geometric progression are consecutive terms ofan arithmetic
progression. Find the sum to infinity of the geometric progression if the first term is 2,
giving your ansrl,er in exact form. t51

A sequence uo,t\,u2t... is defined by uo:2 arrd u,,, =l-)y, wherez>0. Prove by

inductionthalforalla>s, t4l
",=l[l+s(-z)"].
State, with a reason, whether the sequence is convergent. tll


Find, in rei'form, where r and dare exact, the roots of the equation wa +2J1+2i4 .

Hence find, in similar form, the roots of the equation I + 2.,f3zn +2iza:O.

A company manufactures three brands of instant coffee, namely Super, Top and Ace. The
percentage composition of the mass of coffee, sugar and milk in each packet is shown in
the following table.

Brand of Percentage composition


instant Coffee Sugar . M k
coffee
Super 60 30 10
Top 40 30 30
Ace 30 70 0

The company mixed Super, Top and Ace to produce a new instant coffee named STA in
packets of 50g which contain M%o coffee, 387o sugar and l87o milk powder. If each
packet of STA contairs x gm Super, y gm Top and z gm Ace, show that

where A is a matrix to be determined.

Hence, determine the mass of Super, Top and Ace in a packet of STA.

[Turn over
7

The diagram shows the circle with equation x'7+(y- l)' = l. The region R is bounded by
the circle, the .r-axis, and the line x = I . Show that the volume of the solid generated
when R is rotated through 360" about the x-axis may be expressed as

l[2-

2_2
(1-x2)]d [3]

The line y = a cuts the circle into 2 regions. ,S is the smaller region bounded by the circle
and the line. Shade, on a clearly labeled diagram, the region S. tr1

Find, correct to 3 decimal places, the volume of the solid generated when S is rotated
through 360'about the.x-axis. tzl

The plane fI, (i-5j+3k)= l. The line /, passes through the poins ,4
has equation r D

and B with position vectors -i+2j+4k and j+5k respectively, and the point ,t4 lies on
llr '

Find
(D a vector equarion ofthe line 4, tlI
(ii) the acute angle between fI, and /,, t3I
(iii) the perpendicular distance from -B to fI,. t3l

The plane lI, contains /, and is perpendicular to fI,.


Find
(iv) the equation of fl, in Cartesian form, t3I
(v) a vector equation ofthe line which lies in both fI, and fI,. t3l

[Turn over
8

Statistics (60 marks)

7 Two archers A and B take turns to shoot, with archer A taking the first shot. The
of A B hitting the bull's eye *" ] I -
probabilities archers and
'65in each shot und

respectively. Show that the probability of archer A hitting the bull's eye first is f2 . t4I

8 Seven men and five women from the Executive Committee of the Mathematics Society
need to arrange themselves in two rows ofseats fora photo shoot.

ld Row 1 2 3 4 5

2nd Row 1 2 3 4 5 6 7

Find the number of ways that the committee members can arrange themselves if
(D members ofthe same sex cannot sit next to each other, t2l
(il) seat number I and 7 in the 2d row must be occupied by men and seat number 3 in
the lst row must be occupied by the Chairperson who is a man. t3l

9 It is believed that the probability p of a randomly chosen pregnant woman giving birth to a
Down's Syndrome child is rclated to the women's age x, in years. The table gives the
observed values ofp for 5 different values ofr.
25 30 35 40 45
000067 000125 000333 001000 003330
(D - Give a sketch of the scatter diagram for the data. t I l
(ii) State, with a reason, which of the following would be an appropriate model to
represent the above data.
b
A: P=a+-,
x
B: P=a+be",
C: lnP=s+6r,
where a and b are constants and 6 > 0. I2l
(iii) For the appropriate model, calculate the values ofa and 6, and find the prodrrct-
moment correlation coefficient. l2l
(iv) Obtain an estimate of the probability of a 19 year-old woman giving birth to a
Down's Syndrome chjld. tII
Comment on the reliability of your answer. ttl

[Turn over
9

l0 The average number of errors transmitted over an intemet data trammission system is
known to be p per day, where p > I . It is fourd that the probability ofthree errors being
transmitted in one day is equal to the probability of twelve errors being transmitted in
twelve days. Calculate the value of p. t4I

Company ,tl uses the above internet data transmission systm, but Company I uses an
altemative system which is known to have an average of I error per day. Random
samples of 60 days and 50 days are taken from companies I and I respectively. Find the
probability that the differenc between the mean number of errors per day for the two
companies exceeds 0.5. t6l

ll A film distributor claimed that the tickets sales for a particular movie hit an average of
I I 85 tickets a day. A random sample of 56 days was taken and the sales, x tickets, for
each day were recorded. lt was found that )(;-IOOO1=9SZO and
)1,-rooo)' =182043e.
(i) Find, correct to 2 decimal places, the unbiased estimate of the population mean and
variance ofthe ticket sales. t2l
(ii) Test at 2% significance level whether the distributor was overstating his claim. t51

(iii) The ticket sales for a threeJay sneak preview were recorded instead. State, with
reasorL what test you would use in this case. tU

It is known that the movie was screened over a period of time and that there was a
difference between the ticket sales during weekends (Saturdays and Sundays) and the rest
of the week. Suggest an appropriate sampling method that can be used to obtain a random
sample of56 days to assess the ticket sales, and describe briefly how this can be done.
l2'l

12 A Physics examination consists of two parts: a written paper, and laboratory-based


practical assessment. The marks obtained by the students taking the examination are
normally distributed, with means and standard deviations as shown in the following table.

Mean Standard deviation


written paper 48 23
laboratory-based practical assessment 61 8

Three candidates are chosen at random. Find the probability that exactly one of these
candidates has a written paper score ofat least 53 marks and exactly two of the candidates
have laboratory-based practical scores ofnot more than 60 marks. l4l

[Turn over
l0
The point entry requirement ofa Physics course in a local university is calculated based
on the sum of 70% of the written paper mark and 30% of the laboratory-based practical
assssment mark. To qualifu for the course, a candidate must attain at least 70 points.
Find the probability that a randomly chosen candidate can qualiS for the Physics course
in the local university. t4l

State the assumption you have made in your calculations. Explain briefly why this
assumption may not be valid in practice. l2l

13 The time taken for a certain brand of solid fuel to bum completely may be assumed to
follow a normal distribution with a standard deviation of42 seconds. It is found ttprt l2o/o
ofthe solid firels took longer than 379 seconds to bum completely.

O Find the mean time taken for this brand ofsolid fuel to burn completely, giving your
answer to the nearestsecond. t31

(ii) An adventure company buys 120 of this brand of solid fuel. By using a suitable
approximation, find the probability that more than l8 of this brand of solid fuel will
last longer than 379 seconds. t4l

A shopkeeper repackaged and sold this brand of solid fuel in boxes of 12. Each box needs
to be exchanged if more than 2 ofthe solid fuels do not last longer than 2E0 seconds. Find
the probability that a randomly chosen box ofsolid fuel needs to be exchanged. t3I

A boy scout and a girl guide buy l0 boxes and 5 boxes of such solid fuel from the
shopkeeper respectively.

(iii) Find the probability that the shopkeeper needs to exchange 3 boxes of solid fuel with
the boy scout. 121

(iv) Find the probability that the shopkeeper needs to exchange a total of6 boxes ofsolid
fuel. t2l

End of Paper

ITurn over
21X17J2Premm Paper 2-sollltion
t r6 = r3 3 4= -2 4e r

5- J=


r2 =
)for =0,1,23
(

( 2)( 1)=( 2)( 1) Sub =0,1,2,3 into abovc,

Obtaln w=L % t L
Since , 0; ),v < ), )

3_2
+1=0
=0
02+ Use dlc ttbstitudcln =1,
7 1Kn 1=
"
),vzi ),v% ;),v (
Z
)

For sum to ininity to cxst l Z


l

Z= ), J , ), )


-1

1 =
2

= =
Amt in g
or each 50g packet oFST

herefore,
CofFec
22
Sugar
19
Milk
9

2 ta thc iatctnent=:[ 5(2)"] 0_6 +04 +o3z=22

cO, 0
03 +03 +07z=19
When =0, 0: +0_3 =9
LHS t7 =2

RHS=:[+ =2
=

Hen ,the statcmentis lme FOr =o : :: l;:=:il


Assume statement`true For a wherc (o, o,ie

C/t=:[:+5(-2)=]

: :=liII
To p vc 4J lme,lc J]
J=:[1+5(2)
= L.l=1-2 TherCfOre,A=


L ]


=:[1-2(5)(-2)1]

!]
=:[1+5(-2)1
10g ofAce
P. 4.iS true
Sincc Po t mue,PI PIJ is me,thereFore,by 5 ,2+( _ 2=:
0( 2= 2
ma cllnaical lndu On =:[+5(-2) r all
]
cO, 0 =r,=lti(l-rT
The pan ofthe circle we are interesled in is

=:+: ( /=l-.f -,') .

;6.r-1 *
J Vot generaed =
(2)
undefned
" I[, -
ThcrcForc,sequcnce is not cOnvergcnt
= l[1-2J FII ,+(: 2)]
=
" lf, -,, - z$41 a. 1.r,o*ny
( =
`X

=1111XI:

=lil=21il

2:<i j-2k)=( + +4kxi j 2k)


=-11

Volume generated by S
Cartesian ttn: 2z=-11
=: (l)2(1)_ l[2-,2_2 (1-,2)]`
(v)Direa10n vector of 4=4x22
2_2
=: (1)2(1)_ l[2- 11-,2)]`

=!r
3
- 0.095E322 = 0.746 unis
1:IXlil

=lil

7 Trec Diagram:
1


5
B


:X:X: + :X:X:X:X:

(iii) Since,4 ties on n, , perpendicular distance required is


eA. ,\
---i--t- '
lqt
:

8 (1)7!5!= 481Xl

Perpeadicular distanc = 1.52 uniLs 2X2 91


(li)
=10886400
9 (1) z{ - No. oferrors transmiaed in I day by company ,{
L2:proba ty
I - Po (1.21E)
*)
Since r = 60, by CLT, 7 - ,vlr.Zrt,t oo
''
\ J "oo.o*

B - No. oferrors transrnitted in I day by company 8-


a-Po(l)
sinc,, = so, by cr-r, ;- r(lj) +o--
Ll: Age (r) -3 (0218,00403)

iden ,modd C P( >05)



Possiblc reasons:

Shape of the points follow the shape of an =e (7 -8, o.s) + r (e -E . +s)
exponential tth
: normatcdf(o.s,toe,o-z r t, J6oao:)+
incrcases incrcascs(beCause the other
2 chois havc p dccreasing as r increases)
normatcdf(-t0e,-o.5, 0.2 ls, JbJ4o3
)
Forrn L3 as in p =00802(accept O_0803)
LinRcg(a+bX)Ll,L3
ln = 1247+01978x 1 )
=-1247 -125 HO,_l lxlKl= (
b=01978=O198
leXl)2_E

=09937 0994

(i') lt p = -t2.47 +O.t978 ( t9)


s2=J
l
(

= -E.71 l8 =:178143=::78
: :4, s2=787016=78702
p = 0.00016,16 0.000165

Extrapolarion done in calculating p when r = 19.

There may not be a linear relationship baween i) : = 85 H:: 1185


Inp and x forr < 25. Z Tcst
inpuL Stats
l0- X - No of errors transmitted by intemet data :1185
transmission system in I day, i.e. X- Po( p) i V787 016
f - No of efTors traosmitted in l2days, i-e.Y-Pdl2p'1
x=1178:43
(r= :56
Civen P(X= 3) = P 12)
Sketch the graphs :

Yl = poissonpd(X, 3) and p vallle=0 0337 Kac pt 0 0336)>002
Y2: poissonpd(l2X,l2) Since p va:uc is not morc thatthe levei oFsigniFlcancc,wc
do not reJect Ho therc is insuF6cicnt eviden ,at 2%
ieveithatthe mcan tickct sale is icss than l185

01) USe t_test since sarnple slzc ls Small

Suggest stra ned randclln samp:ing to bc d

No orw *cnds sarnplcd should be:(56) 16

ebm = ) No oF wockdays saraplcd shou:d bc:(56) 40

=L218=122
12 N(48,232)
N(61,82)

Prob Reqttred

=P( 53)xP( 60)xP( 60)x L

= s,o,,a* (stff , a, J * l(,-"ao(-r o',e0, o r, f-))'


=0252
tr=0.7 +03

r N(07x48+03x61,072x232+0_32x82)
r N(519,2649

T>70)=0.1331 133

Assumption ls that and arc indcpendent


Not va d because the pcrFonllan oFthc nrst paper


would afFectthc pcrForrnance ofthc sccond papcr


i3 i P(r>379)=012
P cr 379=088
379-
=InvNorrn(0_88,0,1)

=3297=330(nearCSt S nd)

1/No oFsolid ttels that last:ongcr than 379


s onds,out oF 120
/ B(120,012)

since =120 is large,4 144>5, 1 2) 1056>5,


/ N(!44,12672)apprOX

P(Y>18)=P(Y>185)cc
=nomdf(185,1099,144,V12672)
=01247=0125

P (X< 280) = normcar(-to', ZtO,ra.l,+Z)<.t ttl


No oFsolid ttels that do not iastiongcrthan 280
seconds,out oF 12
B(12,O H83)

KJ >2)= M 2)=01616=0:62

S No oFboxcs that needs to bc cxchanged,out oF 10


S B(!0,0:616)
P(S=3)=0147

S+G No

ofboxcs that needs to bc cxchan ,Out Of


15
S+G B(:5,01616)
P(S+G=6)=00182

YtsHutt JutttoR Cot-leee


PRELIM:NARY EXAMINAT10N 2007

H:GHER 2 MATHEMATICS 9740 1


PAPER l
17 AUGUST 2007
FRIDAY 0800h-1100h
Addttbna:matena:s:
Anttr paper
Graph paper
LIst of Fomulae(MF15)

JUNR60 rrytlI ttOR COLLaC /rsI I v R 60 EC 6u Rc E S uVoR Cat`


uvoR OLCF rrSrf OR 60 EC Z6
Vyt/vroR CO O Z6 V OR 60LI
JUNR60 aC u RCOttC
" S 70R COZLaCF
S uv NoRGO
=
JUNRCOttC Z6 N R COLEECa S JUNRCO 77S Vyt17VrOR COL``
V VOR OELECF S OR CaZaFc Z6 VORCO FcF ittUN MOR60
S V OR CO C S uv ,wOR GO`
: Z6 V 07VroR caL FcF rrsrr N R CO
: VOR CO S V uVR 60at E S RCO ES uv OR 60LII`
VJONRCO EC OR COLL ju RCOtt S uv J
"
VOW OR CO EC `ISHON
/1SII V R
rrsrrt/1 `
=OR CO
S V M R CO FC 4S V ttWOR CO `
V
0 OR CO Fc /1slaV aVO 60 /1SI V OR CO /1SIt/7V V R CO `
OR COLa O tts v MOtt CO s avsu cOLL SV VOR CO
7/1 JfttV R CO CF= V V rr2 att rt V
GF risH /1
R CO 70R CO O y=SH( V V R CO [
`

T:ME 3 hours

READ THESE INSTRUCTIONS FIRST

Answer all the questions.


Give non-exact numerical answers correct to 3 signmcant figures, or 1 decimal place in the
case of angles in degrees, unless a different level of accuracy is specified in the question.

You are expected to use a graphic calculator.


Unsupported answers ftom a graphic calqJlator are allowed unless a question specilically
states othenivise.

V1/here unsupported answers from a graphic calaJlator are not allowed in a question, you are
required to present the mathematical step6 using mathematical notations and not calculator
commands.

You are reminded of the need for clear presentation in your answers.

The number of marks is given in brackels I I at the end of each question or part question.
At the end of the examination, fasten all your work securely together.

This question paper consists of 5 printed pages including this cover page.
1


l
2007 YJC Pre:iminary Examination
Higher 2 Mathematics 9740 Paper l

-L' The eqtation of a curve * o'-+=#where x>0,y>0 and t is a positive

constant,
pina the coordinates of the stationary point on the curve, leaving your answer
$Y
exactly in terms oft. I4l
[You need not determine the nature ofthe stationary point]

,.-(iU For the case when & : I , find the equation of the normal to the curve where the

curve p dlough tt polnt


) hg yOW mh exact


{ fi Without using a graphic calculator, show that x2 + 2.r + 4 is always positive.


fina ,n" range ofvalues ofr that satisfu
@ 2x+4





Deduce thc range of valucs of that satisfy 2 12]

+4

/ Show that
2_ +2

=;
:}
V forthc sum of SN wherc
QFindancxprcssionintcrmsof
S =,::






Dcducc that s
l
[2

-r. The first three terms in the series expansion of (o+r)-b, where aeE and 6> 0, in

asccndhg"wers ofF arc: +

[5]

xb xPanSiO [1

Page 2 of 5

2 7 YJC Pre mh F mination
Higher 2 Matheantics 9740 Paper l




2_8
=O and y= (2 - x) on a single diagram.[3]

Sketch the hs of y2+4

On your sketch,sI w that tllc o graphs intersect at =0 5 and =2. h

By using thesubstitution sin z = r - l, n"a frfi - 1, - rf ar. E4]


;liif
tt the exacta
ShOW ofthe region bounded by dle x axis and thc graphs



2_8


f y2+4 =O and y= 12- )is

14]

.6 rl Sotve z4 = S(JS -;), f.rlring your answer in exact exponenrial form. 14]
'
,,i{ oru"nt}rat (a + i)a = *(J, - r, show that one of the roots of a can be
CXprCSSelas(c and 9 are nstantS tO bC

(;:) (Sin( :)+9)),WherC


determined.


SbW deJy on a Argand dagralnthe bc


vCn by: arglz

The set of points in the tocuslz + I - 24 . , li". in the regionl s arCQ+t)sl.
Find the exact maximum value ofr. t3l

The equations of the line /1 and plane rI I are as follows:

. t1, . =[-t,l.J],],**.2isaparameter;
[*.1 l.oJ
flt:. xa+ z = 5a+4, where a is a positive constant.

-(if If the angle between /1 and fI I is1, show that d = l. [3]

--{iif Verify that /1 and fI I inrersect at the point l(5,-1,4). [2]

(/T) civen that C(7,-3,4), find the tength of projection oflCon fI 1. [3]

{fi firra ,n" position vector of N, the foot of perpendicular of C on fI I . [3]

| (v) Point C is obtained by reflecting C about fI 1. Determine the vector equation of


the line that passes through lC t3l

Page 3 of 5



2007 YJC Pre:iminary Fv minat10n
Higher 2 Mathematics,740 Paper l

h dttm
+
q

Usc your rcsult to flnd an expression in tenlls of for]


:T
[3]


9 The cunre shown in the diagram has equation y = f(r). It intersects the axes atx = 3,


=4,y=L2and has a maximum point at

y=1

=2 =5

On spamte diagrams, sketch the graphs of:


G) /=fftl),

V(ii) y2 =-f@,

I
.tr(rr) /=7G+D-

In each case, indicate clearly the asymptotes and the coordinates of the intersection of
the axes and tuming points whenever possible.

Page 4 of 5



2007 YJC Pre m:naw Examination
Higher 2 Mathematics,740 Paper l

l0 In a certain farm, the growth ofthe population of prawrs is studied. The population of
prawns at time t is denoted by x (in thousands).

It was found that the rate of birth per day is twice ofr and the rate of death per day is
proportional to x2.
(i) Given that there is no change in the population of prawns when its popularion hits

'dt5
l0000,strowthar *=rr-* tzl
(ii) Its owner decides to sell away 1800 prawrs daily. Modift the differential
6q'ati61 above and show that the resulting differential equation can be written as
&--ll--oY--rl
[21

Given that the initial population of prawns is 13000, solve this modified


diflerential equation, exprcssing r in terms of t.



Deduce the long term implications on the population of prawns in the farm.

The functions f, g and h are defined by

ix
,x 0

g:r r-. J2-r, x<2


h:rp> lnx, 0<x<a
(i) Sketch, on separate diagrams, the graphs offand g. [3]

, (ii) Show that f-' does not exist.

If the domain of f is restricted to x < b , state the largest value of b such that f-r exists.
Hence, using the new domain off, inverse.
determine its t6]
i1 {iil; fina *r" exact value ofa such that gh exists. t2l

END OF PAPER

Page 5 of 5



Yishun Junior College oMathematics Department
2007 JC2 Preliminary Examination
Higher 2 Mathematics 974OPtper I

SOLUTIONS

STEPS/SOLUT10NS
Oucstion I
(i) Di fferentiate wrt.r:

_ 2+:i
{,,*)-,,-*=,-*=+
r)
l-r'*-Ll=o |

' sin""
2ry
+o
t ,,;)
',t
.'.y'= 3

2rl

sub in 2=





')
*r^r. are
Hence, coordinates of stationan,y porn* ^.-( l 'f
;, 'z i
\
I

sJtr,o )
(ii)Atthe glven pomt and t=1:

=:

31-(i)2+ : )=-17

Gradiettof mmJ=

Equatbn ofnomJ: = 4)

y=

Ouestion 2

1)
2+2x+4=lx+1)2+3>O forall c m

D
:



2+2x+4>0 _2 0
<x+2) 0
-2 0


IX+2) 0

=>-2 0
Since >0,no valuesof x sa thiSinequahty o o SOlu ons)

Oues on 3

2+ _2
1 1 2+ +2- 2
D 2_ +2 2+ +2 4+342+4 4+3 2+4

ii)Using previous result in (i),




=:
i(7= )
=:[(: +(
:)+(: :)
T )]
=:(:
T)

= 2 2+
+2)

Sincc >0 CZ+,= S.


:

Oueslon 4

b(1+:)
(2+ ) b=

=
[+ U2_]
Comp ng tte rt rcc nlls: =: (1)


(b)= _(2)

0+1) _ 2 I
2
=
n .......(3)
b 2 3
Sub in(1)int (2): = = )
3 2

Sub. in (r) and (4) into (3): e!+ =
2o2

+l,x l==
2 91
)2 27
20+1)=1
9b 3
=2

=3121=3
2

2=: +:)2
)0+ Expallsbn valdr

1 3

Ouesdon 5

i)

2
+4x2 - 8x =o

1)2 = ''[-rir,', ("oru ar)



_
os2 d

=:
s 2 +1)d

: 2

= + +
4 2
OS
` + +0
2
lly-1)+
lx-1
+
2

iii)Notc:



1-lx-1)2

-1)2

Area of shaded region =



11)2+s 1)15+:(:)

21

11

7

-+
3 2

Ouestion 6

(:+2
a)1) z4=8(V5- )=16 )

+1
z=2 ( ) wherc =0,1,2,3

23

=0:z=2 24 =2:z=2 24
H
t=1:z=2
j .24
24=2`
24 =3:z=2

ii) Comparing G * il' s(..[ - i) r(.6 -


= and. za = ,,

+f=2` ('+ ) wherc t=0,1,2,3



+j=2` 24=



+0:


One ofthe r ts of




=( Slgi)

=21
( )

arg(Z+1)
:

Max valuc of occus when thc circlc touches l and 2

From abovc:sin(:;)=: maX Value of r=

Oues o 7


l
=
z(o' *r)= +,'
a:+l
:. a --l(a = -l rcjected since a > 0)

N
/_




























=0

C',
=

Since lr' is midpoint of C and


DP mJ mLJ
:
Hencc, land
:
li)Sincc l and

v)
VCCtOr"uat
Vector equationn or n p8
oi line ng ulrougn
passing Ac'
through A :
=t .,[|] , C"
+rt
[;l
Ouestion 3

L dle Sta ,

ht =1:LHS= =: and RHS=1- =:


ustucbrmme , cZ+
,

We wantto show that 4.ls alSO mc_

,

LHS=,: +1:!:
+
=1+

=1+
=1-1:11
[

=I HS

mC J JSO tue.

Sincc true and P.me 4J truc,


by pHnciples of mathematicalinducion, iS mc cZ+


=
=[
fil,F) [ ffi )

=0
)

Ouestion 9

i)


9








(z.s,e\




=2 r=3

Ouestion l0

9
i)'dt = rate of birth - rate of death = 2x - u2
Atx = 10, O=2{10)-a(l})2 >a=0.2

o-r ^ -r

ii) Selling away 1800 prawns daily, D.E- becomes = r. - { -r-f


!dr5

10
dr=2x 2_2
dt 5 5

=_:lx2_1 x+9)= -9Xx-1)


:(

h pting b J m_
=




=
:dr
= c m tti mt
:+Q whe
"
= ,/=`
`

/= =: _9 ` 5
X-1= 3
=
24
8 +l
3- 5

8r
In the long term: as , --) co, eJ -+0. ...t=L*1-9
3-e i
Hence, the population of prawns will stabilise at 9000 in the long term.

D1


=
1
LI



ii) The liney = -2 cuts the graph oflmore than onc. Hence,;fis not one-one atrd /-l does not exist.

t argest value ofb such that /-l existsisb=-l: (--,-l] or xS-1.


-Lt v=-x2 +1
'2x
x2 -2xy +l = 0
(x-yf +t-y2 =o
x= y!J7 t
Since x<-1, ,=y-,15 -- fa(r)=r-,[? -t' r<-l .

iii) For gl to exist, R6 c D, + R6 g (- -,2)

Sincey = ln-r= x= et, -'. o = e2 -

12
Yrsnux JuttroR Colueee
PRELIMiNARY EXAMINAT10N 2007

H:GHER 2 MATHEMAT:CS 97402


PAPER 2
21 AUGuST 2007
TUESDAY 1400h-1700h
Additional materials :
Answer paper
Graph paper
List of Formulae (MFl1)

tt CO S yt7VrOR 60 ECF S ttOR CO rISa7v M R60 ECF JuVoRCa l


9 OMOR C
OR C L aC %6 O rrsa/NvoR 60f ECF yrsII aV OR 6




S JUNRCO ECF V VOR O
S V R60 aCF yrsI r 7R c
] O C ,SuV tt cO 30 "
S uV R 6 EcF 6 JUNR60
V `OR
" "
OR Car acF rlSIJriuN ORCaz Ec
VOR COLECF rrSrruriuNRC Fc
6 JU oRO"C vUNR60
S R CO aCF t71 R60


V V OR CO ECE V OR CaZ c ntlI

RCO FC %6 V `
V C ,7uV OR cOL EcF `
rrSIur VOR CO FCF V`VUNOR60
`OR CO
V VOR`OttCF%6


V V OR `
yN Ott CO ECF s VjuNRCO c
yV UN=OR CCL r Z6 av R cO E
V
,N aC xM Jf/17 R /1 7,NR
CO S 77V V R COL CF
INR
`G

TIME 3 hours

READ THESE INSTRUCTIONS FIRST

Answer all the questions.


Give non-exact numericel answers cr_rrecr to 3 significant figures, or 1 decimal place
in the
case of angles in degrees, unress a different rever or accurac! is specmeo in th;,il;;td. " -

You are expecled to use a graphic calculator.


unsupported answers from a graphic calculator are allou/ed unless a question specffically
states otherwise.

lMrere unsupported answers from a graphic calculator are not aflowed in a question, you
are
required to present the mathematical steps using mathematical notations and not #cubtor
crmmands.

You are reminded of the need for clear presentataon in your answers.

The numberof marks is given in brackets [ ] at the end of each question or part question.
At the end of the examination, fasten all your work securely together.

This question paper consists of 6 printed pages including thb cover page and 1 blank page.
2qr7 YJC Preliminary Examlnation
Higher 2 ,ilathematics 97,10 Paper 2

Scction A : Pure Mathematics

l Givcn thatthe Curve y=flx)pasSCS h ugh(0, =h(


)and that(1:)tan ),

Dducethe Md cxPh J h(

in x2. [11 -

2 (a) The first term of an arithmetic progression is l0 and its common difference is

I . Firrd tt e least number of terms such that the difference between the zum of the
4
first n terms and the sum of the first 2rz terms is more than 200- t3l

(b) It is giventhat T,:2t8'o,where z e9 and neZ+ .

(i) Show that T1 + T2 + T3 +... is a geometric series.


Hence, obtain the range ofvalues of u such that the sum to infinity of this
series is convergent. t4l

N/l _\ l-ets,N*l
(ii) Show that I[., . r,
,)
= t3]

'-;E,

re /1 have equatio* .[i]: 8 and /r :' : . r['J


",, [i)
respectively, where 2 is a parameter.

(i) Find the position vector of the point iir; the foot of the Perpendicular ftom

B(3,1,5)to IIy
Hence, or otherwise, find the exact distance btween h and II r t5I
(ii) Find the cartesian equation of the plane Ir 2 which contains /r and the line through

B and N. t2l
2 /YJC Pre:lminary Examination
Higher 2 thelllatlcs,740 Paper 2

paramet c equations ofa cllrve are y=sin2r and x=a cosr,0 and

O 1_

O SketCh the curvc,indicating clcarly all interccpts and i dng points. [2]

O ShOW rJ"h On tte curve, HenCe,fmd tt rate ofchange


(

Of
inmnt cn decreascs a a rate of:

0 P
_
( )Find thc cqttdon ofdle linc that p^ses ugh d oHgin and thc point

at r= _

6 [21

The region R is enclosed by the curve, the line found in (iv) and the y-axis. Vp is the
volume obtained by rotating R completely about the y-axis. Find Vp leaving your
,
answer exactly in terms ofa. t51

SectiOn B:PrObability and Statistis

(a)CalCulate l10w nlany differcnt numbers altOgcther can bc fOrmcd by taking Onc,
two,drcc and fOur digits ttOm thc di ts 2,3,8 and 9,rcpctitiOns not bcing

allowed.
[2]
( IfOne Ofthe nulnbcrs in( iS ChOScn at random,calculate thc probability that

it wIIi bc greater dttn 300


[3]

In our school canteen,33% of the students patronise the westem food stall. 3 students
outof 7, independently felt that the priccs at this stall were reasonable.
(a) At a particular instant, there were 40 students at the canteen eating. Find the
probability that at leasr I 5 of the studenrs were earing westem food. I2l
(b) Using suitable approximation, find the probability that in a group of 150 studenS,
between 25 and 60 (incrusive) students fert that the prices ar the westem food stall
were reasonable.
14]

Page 3 of 6
2OO7 Y& Preliminary Examlnatlon
Higher 2 ltathema66 9740 Papr 2

7 At a construction site, dengue fever cases occur at an average rate of I in every 5 days
from April to July (total of 122 days).
(a) Find the probability that during these months,
(i) at least I dengue fever case occurs in a given period of 7 days, tzl
(ii) in a period of 5 weeks, no dengue fever cases occur on
at most 3 ofthe weeks, I21
(b) Find the leas number of days during these months that gives a probability of at
most 0.9 that at most 3 dengue fever cases would occur. t4]
(c) Suppose the rate of occurrencs ofdengue fever is I in every 15 days for the

rest of the year, find the probability that 5 dengue fever cases occurred from

April to July iffifteen cases had occurred for the whole year, taking a year to be
365 days. t41

8 In testing the length of life of electric light bulbs of a particular type, it is found that
12.3 % ofthe bulbs tested fail within 800 hours and that 28.1 % are still operating

ll0o hours after the start of the rcst Assuming that the disaibution of the length of -
life is normal, calculate, to the nearest hour in each case, the mean p, and the standard

deviation, o, of the distribution. t3l


(i) A light fitting takes a single bulb of this type. A packet of three bulbs is bought,

to be used one after the other in this frtting. calculate, to two decimal places, the
probability that the total life is more than 3300 hours. t2l
(ii) calculate the probability that all 3 bulbs have lives in excess of I100 hours to give -
a total life of more than 3300 hours. Explain why this answer should be different
from that obtained in (i). t31

g (a) A.company produces one thousand cans of milk with the following distribution:

Flavour Number of Cans

Ch olate 280

Strawberry lClll

Vanilla 400

Unflavoured 220

Page 4 of 6
2OO7 YJC Prelimlnary Examlnatio0
Higher 2 ,{athema6cs 97,tO Paper 2

In order to ensure the quality of the milk is maintained, the quality control team
takes a sample of 50 c.n< of milk for testing during each shift
(i) The team takes 50 cans of milk produced from the first hour ofeach shift.
Explain why this method of sampling is not appropriate. I2l
(ii) Describe a suitable sampling method for the above - I2l
(b) A private hospital a.lmits an average of 85 patients daily with a standard
deviation of 35. In a sample 30 days, what is the probability that the mean
number of daily admission is less than 95 ? t3I
If there is a probability of unre than 0.75 that the mean number of daily admission

is between 80 and 90, find the least value ofthe sample size, t51

l0 (a) In a particular workshop, it is claimed that the masses of components produced


have mean 69 and standard deviation 0.8g. If this claim is not rejected at the 50%

level of significance based on the mean mass obtained from a random sample of
l0 components, between wlEt values must the mean mass of the l0 components
in the sample lie? State any assumptions necesv4r for validity. I4t
(b) The manufacturer ofthe workshop replaced the old machines with new machines.
The manufacturer claimed that the masses ofthe components produced by the
new machines are not less than 6g. Based on a sample of 10 components,

)tr-+l=14 and ){r-a)' =26-7 were obtained. Given that the masses of
the components follow a normal distributiorq carry out a test at 59lo level of
signifigance to determine whether the claim is justified.
r4l

ll (a) Explain why it is advisable to plot a scatter diagram before interpreting a


correlation coefficient calculated for a sample drawn from a bivariate
distribution.
Sketch scatter diagrams indicating the following
(i) a linear product moment correlation coeffrcient close to zerq
but with an obvious relation between the variables.
(ii) a linear product moment correlation coefficient close to +1,

but without a high degree ofconelation between the variables.


[2

Page 5 of 6
2OO7 YJC Preliminary Examination
Higher 2 ,ilathematics 974O Paper 2

(b) The yield of a particular crop on a farm depends primarily on the amount of
rainfall in the growing season. The values of the yield f , in tors per acre, and
the rainfall X , in centimeters, for seven successive years are given in the table
below.
12.3 13.7 14.5 tt.2 t3.2 14.1 12.0
y 6.25 8.02 8.42 5.27 7.2t 8.71 5.68

(i) Find the linear product moment correlation coefficient between X alJld Y .

Hence, comment on the value and its implication on the data. I2l
(ii) If the values of the yield and rainfalt are re-recorded in kilograms per acre

and metres respectively, do you expect any change in the value of the linear

product moment correlation coefticient and why? tl]


(c) For a given set ofdata for X and f, it was found that the regression line of
s
X on I/ is y=-lx+5.5
'8 and the linear product moment correlation coefficient r -
between X and Y ,' =! .lt
,is given by'25 is also known that (1,fl= 1+,t1-

Find the regression line of Y ot X . t3l

End ofPaper

Page 6 of 6

nior COllege oMathematics Department


reliminary Examination(Pure Maths)
)r2Mathematics 9740 Paper 2

1 _ SOLUTIONS


KE STEP /SOL TIONS

n(
2+m(
)
2
)=
r



DifFercntiatc(2)wrt x:


2scc2 y tan
( )3+2sec2

+l
` +1


I )+m( 0

2sy,c?
)=
AtF=0:f(0)=:
f.(0)=0

F"(0)=: Hence, Maclaurin's series:, = + * t *! *...


4424
f'''(0)=:

MaClaunn,s SCricSOflCoty)in( + +;:)=:+



)=::(
Oues on 2

0S2 =2 -1(:)]=20 +:12 -1)


[2110)+12

Sa=:[2110)+ -1(:)]=104+:: -1)

lsr, -.s,1 > zoo

[0,*14-4,zoo
| 881
Using GC, n < -39.8 (rcjected) or n > 13.4

Hence, least number of terms is 14.

(b) i) Note: Tr +T2 +T3 +...=ett' * * *...


"'ru' "'"'
,= f,-
Tr_t

elgu"

etBu
/ "\
tcl
=e
\u^lI Since r is a constant, the series is a geometric progression.
= elg"

For series to be convergent, 'ttlrt*l.t


-l < elc' <l
Since ela' elc' <l
> o, then o <
' +lgu<O
=0<ucl



I)



` (1-`rg )
=1+ rg
1_
+1
+
rg _ rg
1-


1-
l

1_

Ouestion 3

(1) iS On 7:V



lil=8

rpCnd J ={il


.'. Distance between Z and /r = =
lffil



!=13

x+2z=l

Oue.stion 4

i)

Y = sin2t
-r=acosl
... dv dv dt
tll --: =---:-x-
' dx dt d-r
'
I
=2sinrcosrxdsin I -
__2cost
a
: --t2( x\I
-
a\a )
2x
-2
ll

When ::+() (

D =


ncc O 1:0 2 1 >1

OA = : = ,y=:
l

g = =
2

Hence,"uat10n ofr : =

Volulne VR= 1)2(:)


flaCOS )2(2 sin cOS )dr+:(

6



)dr+

=-2,rJ*'o''11 *""
Lall 6
16

=-zo'Jo- 641
sl**'
L 16

llto2
32

YISHUN JIIJNIOR COLLEGE


H2MATHEMATICS
JO(21X17)PRELIMINARY EXAM STATISTICS
LIARKING SCHEME
Solution
The number of :

onedigit nos. = 4
two-digit nos. = l2
threedigit nos. = 24
four{igit nos. = 24
4+12+24+24=64
No. of tfueedigit nos. that are grcater than 300= 3 x 3 x 2 = l8
No of four digit nos hat are greater than 300=4 3 2xl=24
Rcq red P ba
=18+24=
Let bCtherv.``no ofsmdcnt(out Of 40)cating wcstem food at cantccn at a
p cular instant''
B (40,033)
P( 15)=1-P( 14)
=0_326 using l bi F(40,033,14)

ht y bether.v.``no ofstudcnts(out Of 150)who fClt thatF pnces at ulc westcm food

stall are reasonable".
r - ni"lso.f)
1
Since n=150 islaBe, p=! is neither large nor small, np=$4'3>5atd nq=85'7>5'

Y -
N(@.3,36.71) aPProx.
P(25 60) P(245 605)
Q Solution
=0 265 or O.266(3s )
7al Lct bc 3r.v. no ofdcngue fcver cases cu gma7 day" Od(bm Aprilto
July)''.
(7 5)
P( 1)=1-P( =0)
=07534=0 753 o sfsp
using l
"w (7/5,0)
7ali Ld L thcrv . fwccb Of 5),cach OfW ch
(6bm APrilto Jul ''

y 3
(5, )whCre =l-0753
P( 3)=0.915(3 2o slng Jz (5,1-0.753,3)
7b Let bc the r.v. `ncl:ofdcnguc fcver cascs c_g ln a pcHod Of days m Apnl
to July .

P ( )WherC X=:
P( 09


Using GC tO sketch graphs Of y=09 '_(1+X+


i+il)and solvc fOr

>o and
O to Ob in 17447696

17447696 872
:

7c LCt A mcr_v
nO Ol dengue tcver cascs occumng Om ApHitO July"and S bc thc

TIS r dlC t fthe ycar

R+S (4016)

lR=51R+S=15)= '

:][
_P(R=5)P(S=10)
P(R+S=15) =0024086=00241
8 Let x be tlte r.v. "Life (hours) of a randomly chosen lighr bulb- and
X_ il(
2

Given P(X < 800) = 9. 123 and p(X > I 100) = O.281 + p(X 4 l0O)
= 0.7t9
r( z.8@: t')-0.123 and using GC,
\o)
800-z
-; = -l.l60l ---------- (l)

fl^1- rroo-,\
z < ---------!- l=0.719 and using GC,
\o)
Q Solution
HCXl
= 5799
(2)

Solving(1) th(2), =172 and =10


8i X, + X, + X r- ru(r" tooo, 1 x172'? )
i.e. X, + X, + X, - Ar(3000, 88752)
P(X | + X 2 + Xr >3300) = 0.16 (2 d.P.)
81i P(Xt >llUJ r\ X2 >HCXl > )
=(o.ztt)' =o.0222
The two answers are different because the two events are related as:
proper subset of
leach of the 3 bulbs tns life > I I 00 ftrs) is a
lpoul life of the 3 bulbs >3300)
9al The sample chosen would be biased as it might not represent the quality of those produced
tluoughout the shift and allo the diferent flavours of milk that were being produced'
9a -
A stratified ramdom sampling woutd be suitable. This would require a selection of 14
cans ofchocolate flavoured milk, 5 cans of strawberry flavoured milk,20 cans ofvanilla
flavoured milk and I I cans of unflavoured milk. These would be randomly selected from
all the cans produced from each shift.
9b Lct rbcther v no.ofpaticnts admittcd into a p vate hospital ln a day''
G cn E( =85 and =35

+ 30)
"= 1+2+
Lct



rcm.

:
icJ( E99,95,85,35 V35)
( 30 95)=0941(3s ),uSing "

Let "= 1+ 2+ +
)

smmgttt a ls hgQ by Cedd I t Thc et " 85,3 )


Glvcn P(80 " 90)>0 75
1-2P(7" 80)>075
P17" 80)

PIZ )
0 125

UJng




>l15035 , 15035
7
>64_84
Least =65
10a
Q Solu6on
Hr: P*6
Under I1o, the test statistic is Z =I+-l{(0, l) where p=g,6=Q.g, n=le
/J;
l,evel of sitrificancn: a = 5Yo
We reject Ho tf 14>1.96 and do not reject the claim if lzl <1.96

i.e. -1.96 .-I:!- .1.96


/Jto
5.5(X <; < 6.496
i.e. 5.50<;<6.50
10b .Ir'. r,--"
Hr: P <6

Under flo , the test statistic is Z =
( -1)WhcrC =6, =10,

-4)+4=54,s=
= lx-4)2
=08882
Leve1 0fsifniflcancc: =5%
We rCit HoifT -1833
=
Altemative
Using GC, trc p - value =0.0307 < 0.05
Conclusion:
Hence, we reject rb and conclude that at 5% significance level, there is sufficient evidence

+at
m8s of$e comryne -new machine is less than 6g.
1la The scaner diagram gives a good orerview of how theGi abG,
Hal "*,"lat"d.
Q Solution
Hal

Hbi r = 0.98 .
High positive linear correlation between X arld Y.
1 lbli r remains unchanged. It is because r is a measurc ofthe degree of scatter and it is
urrchanged when there is a change of scales.
1lc Regression line of x on :

8 44
y= X= y+
:x+5.5
Givcn r2= b(
:)=
,

b=
25
Shcc , )=(4,1)nes on hc une of grcssbn line of y ,

Hcnce the rcquired"grcssbn h"L

1= ( -4)

18 97
y= X+

10
RAFFLES JUN10R COLLEGE
JC2 Pre:iminary Examination 2007

MATHEMATiCS 9740 01
Hlgher 2

Paper 1

12 September 2007

3 hours
Additional materials : Answer Paper
List of Formutae (MFl5)

READ THESE INSTRUCTIONS FIRST

Write your name and CT group on all the work you hand in.
Write in dark blue or black pen on both sides of the paper.
You may use a soft pencil for any diagrams or graphs.
Do not use staples, paper clips, highlighters, glue or conection fluid.

Answer all the questions.


Give non-exact numerical answers conect to 3 significant figures, or 1 decimal place in the case of
angles in degrees, unless a ditrerent level of accuiacy is specifieO in the queston.
You are expected to use a graphic calculator,
Unsupported answers from a graphic calculator are allowed unless a queslion specifically
states
otherwise.
Where unsupported answers ftom a graphic calculator are not allowed in a question, you are
required to present the rnathematical steps using mathematical notations and not catculator
commands.
You are reminded of the need for clear presentation in your answers.

The number.of marks is given in brackets I I at the end of each question or part question.
At the end ofthe examination, hsten all your work securely together.


This document consists of 6 printed pages

RAFFLES JUNIOR COLLEGE


RJC IES 2 7 Math Dcpartment Tum Over

2

l Find the set ofvalues of for w ch

e.2_8
>0
X-1

HCnCe SOIVe >


4

2 Prove by induction that, for neZ',


2_( _1)2+( _2)2_( _3)2+. 1(1)2=::(
+(-1) +1) 151

3 A sequence ofnegative numbers is defined by 'Jto.


,*, = trn _4
*1r"." ,, =-1.
7

(i) Write down the values of x, and. xr, giving your answers correct to 3 significant
figures. 121

(ii) Given that as z -+ @, xn -+ /, find, without the use of a graphic calculator, the
value of l. I3l

4 The graph ofy= )undergoes in succession,the following three trar sfo.11:atiOns:

:A tratlsiaton Of l unit in the negative rdircction


3:A rcnection about ulcy axis

C:A stretch parallcl to thc raxis(with y axis invariant)Mdl a scalc factor of 2

The cquation oftt stthg curvc y=go w gO=tti



(1)Express fKx)in the fo.ll.gcax+b),Where a and b arc constants to be found. 121

(m)ske h the graph ofy=g ),shO ng clearly Jl the intesections the axes,the _
asymptotcs and the c rdinates of tuming points(if any) 141

5 G cn that ftr)=1,Wherc r is a posiivc lll ger,ind a single exprcssion for


f(r) f( +1)_

He e, nd th sull1 0 2 trlns ofthe


cs:+:+ + + Fl
Deduce that tt suln 0 22 tems ofthc d + + +l+ Sthan L _

RJC 2007 974: 7 [Turn over


3

Given Ch +3 2il(2 and +5+211 31, luSmte the locus of the Point
rcprescnting h complcx nulnber z in an Argand diagraln. i41

Hencc flnd thc least possible valuc of arg z


121

The graphs ofy=lh )l and =hO)for > arc as shown bc w:

y=lhO)


_ (D ExPlain"1 h <o fOr-1< 2
Hcnce sketch ttc graph ofy rx> 3,showing clcarly thc asymptotc and
coordinatcs qfdle s dOrn
I ?f

(li)SkCtCh thc graphs of

_ (a)y=h' )Wherc


h'is tt dedvat c function Ofh,


f r >_3,
_ (b)y=
showing clearly all tllc asymptOte(s)alld thc c00rdinatcs Of the statiom
pOint(S)

Find

a)
tan l: ,
131


141

The region R is bounded by 4


the curves Y=/+1,!=lnx, the x-axis and the
line x = 2. Calculate the area of R_
13I

(0 C gbn 2 bOundcd by e curvcs y= ,y=h and the ne x=1.Find


the lulnc Of tllc solid of rcvolutiOn fo.1llcd whcn 2
tatcd cOmplctely about thc
LaXlS
141

974 107
[Turn over

10 A glass w o xed pellimetcr P,is in thc shape Ofa scmicircle th radius r,and a
rectangle as shown in the diagram below The scnticircle is made of tinted glass and the
rectangle is madc ofclcar glass_Thc clcar glass icts ough a nstant mount oflight per

t areaj le Inted 6sI mugh: muCh t a c dea


"r
glass.Let bec total aIIIount of hght at the entire ndow lcts through.

(D ShOW that r= +2)]. 131


[P (:

(1)PrOVe that to let maximuln amount of light dhugL dle radius of ndow is
3P
141
(sr + t2)'
tinted glass

clear glass


n O nd
121

.l
(ii) Expand (f-Arr) 2 as a series in ascending powers ofr up to and including the term

in , simplifuing the coefficients. Write down the set of values of


xa r for which the
expansion is valid.
31

(iii) By using the results from parts (i) and (ii) above, show that the Maclaurin's series for
sin-' (zx) is given by
3+ x5+..
2x+ 121

(iV) Hen ,sttW that


[21

974 1 07

12 The line /, has equation x-l=2-y=-2.


The line /, passes through the point A(-2,1,7\ and is parallel to the vector -3i + j + 5k .

Write down the vector equations of lines /, and /, . l2l


(10 Show that the lines l, urd l, intersect and find the coordinates of 4 the point of
intersection of /, and /r. t3l

(iii) The acute angle between the lines /, and /, is denoted by 0.By finding cos d, show
,hut
"ioe
=20.
J3s
Hence find the shortest distance from I to the line /,, leaving
your answer in exact form..

41

13 on I't Jan 2000, Selina deposis $ 1000 into an account which pays a.,fxed interest of $200
per year, credited into the account at the end ofevery year.

On l't Jan 2005, Hebe deposits $1000 into an investment account and receives an interest
of $100 on 3l't Dec 2005. Thereafter, the interest retumed at the end of the year is 1.5
times the interest retumed in the previous year.

Taking year 2000 as the first year,


(i) Write down the amount of savings that Selina has in her account at the end of the n6
yeat.
tU
(ii) Show that the arnount of savings that.Hebe has in her account at-the end of n 6 year
is
$(1000 + 200(1.5^t -t)). I2l
(iiD At the end of the #
year, Hgbe saw that her savings finally exceeds Selina's savings
for the first time. Find the value of /c and the interesi that Hebe receives in tt . tli y"Z.
t4t
After t complete years, the interest that Hebe receives for subsequent years is
l/3 of
the interest received in the previous year. Determine, showing yorr reasons
. the rnaximum value of Hebe,s
investment. "learry,
I3l

974 1 7
[Turn over
6

14


x=ct,!=

A curve has parametric equations c is a positive constant. Three

P( , ,JmdR( , m curvc ShW ntt da


" ),2(

(a)ShOW that thc equation of the mgent at the point P( ,il)tO the curve is given by

x+ 2y-2 =0 121

(i) ThlS tangent meets the x axis and the axis at and B rcspcctivcly Write down tllc



_ ordinats of and Prove thatthe arca oftnangle 03 is 2r2 131
ts thC
(il)Given that the nc PO meets thc curvc agaln at O and thC Straight hne BO m
x axis at _Exprcss,in tenlls of and fmd the area ofthc triangle 20Z in te....s of .

[41

0 rtt mmalttR( ,;)mcc Curve again tt S( ,:) bW that r3s+1=

END

974 l1lm7
2007 JCz Preliminary Examination
ID Mathematics (9740/01)
Paper I
Solutions

1 From GC, SolutiOn Set=(- 1.9Q-0.114) (L

>0
1 >0

Hcnce,-1.96<! !<-0.114("icct)Or! l>1


SolutiOn Set fOr =( ,-1) (L)
2
1(1)2=:(
+(-1) +1),fOr Z .:1

+1)=1 L.H.S=R.H.S

l.e. 12_( _1)2+( -2)2_( _3)2+..


(-1)1 1(1)2=:( +1)

To prOve that P :is true.

o
_
H
,

+1)2_ 2+(._1)2_( _2)2+...+(-1)


(1)2=:( +lX +2).
1 e
(

L.H.S=( +1)2_.2+(1_1)2_( _2)2+. +(-1)


(1)2
=( +1)2_[12_(1_1)2+( _2)2_. _(-1)1 l(1)2]

= ( +1)2_:( +1)

= 7 [2( +1) ]

= :( +1)( J 2)=R.H.S.
L truc L truc.
Since Pl is true,by mathematical induction,P is true fOr all z+.
JC/7 Examinarioo
H2Mamlnatlcs Papcr i(9740/01) Pagc:Oflo

3o 2= 0 586 and x3 = -0.820 (to 3 s.f.)


As ,
+l
`,
= 2-3
-4
-4)= 2-3`
2_ _2=0
`(

(
+1)( -2)=0
(reject '.' / is negative)
`=-l or `=2
4(D Before C we havey = g(2x).
Before B, we havey: CeL.).
Before l, we havey: e[-2(x - 1)]=g(2-2 )=fO),1 C =-2, =2.

Alternatively,
After A,we havey :
l). f(x +
After B, we havey= (-x + l).

Alr C wC havc =
::+1)=g0
g(-2)=1'+1)= g[-21`-1)]=ftX), 1 e =-2, =2.

2_2 _3_( _3x +1)


= -4
v: c@): +2 +2 +2

=-2

-1

(0236,

2tX17 RJC JC2 Prc:imi"ry Exalllinalion


H2Mathcmatics Papcr: 740/t11) Page 2 of10

5(D
f(r) f( +1)=:
+= ( _ 1)

1+1+ + +...2 6 te.1 1


2 6 12 20
_
=

= )] [f(r)_f(r+1)]
=l

=[ 1)
+[t t ]+[ ]+. +r) 21)]
=f(1) f(2 +1)

=1- 1
2 +1

0
=
<
Hencc J:+J;+J: +. 2
6tem
J

= < =1- <1


7

31
So, Ieast possible value of arg z
2




1.97
2tX17 RJC Jc2 Prclinintt Exalnindion
H2Madlcmatics Papcr: 740/01)
= 3 oriO
7o In tt graph of =lh )L ttre is a graph for-1 < <2,indicating that
hO>0 0rhO o fOr_1 2.
In the graph of =ho,the iS nO graph r-1 < <2,indicating that
hO)<O fOr-1< <2.
IIen ,hcx) O fOr-1 <2.

=ho

(11)(a)

=h' )
-3

(11)(b)

=-1 =2

27 RJC JC2 Prelinin Exaninatlon


H2M hatics Papcr l(974001) Page 4 or10


= tan 1(:)


2+11
+`
:hl

u-Jr;i
(i Ld


u2 -2x+l
=
=
1
X
d
zuL-z
dx




=2 ,tttid

=0(:J
)+


9(1)

(12)

=7

! =lnx
91421


Area ofR = I,t'u'nu ,n, * * !'r.u.u 4+1

=0.16864+0.12607
=0.295 units2

2 7 RJC JC2 P limi" ExalninatiKIn


H2Math atlcs Paper l(974KVl11) P 5 of10

9( ) Volume of solid formed when Q is rotated completely about y-axis

0, * ,
[:n"o' (*)''
=o o, - n(r\, (z)
fo.n ,o, ff,
- o(0.$aoz+z.tset-z)
-1.232br
:3.87 units3

10o Area ofsemicircL= r2


2

Lengtll of L=
L
(P
=2 )
Area ofrechngle=(2 1)=
)(
:

2)
=( )[ (P -2 )]+(:
)(:

= r[P +2)] G)
(:

11:=Z[ (: +2)+P
+2)]


(:

= +4)]
[P (:

=(
)(: 4)< 0

is maximum when

r= (AG)
(ShOWn)

1(2

dX=:[Sin-1(2 )]i
11(1)
)

2 7 RIC JC2 Preuminay Exalnination


H2Mmatics Papcf l(9740/01) Pagc 6 of10


1_

2)

11(i
(1-4

2)+ 2)2+...

=1+(
:)(-4 (_4

2+:(16 4)+...
=1+2
2+6 4+.
=1+2
ExpalllsiOn is valid fOr
21
1 <: =)
<
14
:

Set ofvalues of fOr which exp 10n is valid=(


:,::)

:Sin-1(2
)=li

1_4 2 d

,in-'1zr; =z t+ 2x2 + 6xa +... dx


J'

.'.sin-r12.r)
-L*43 *
='rl, * 1'
5'"Jo
^ 4x1 l2xs +... (proved)
=zx+_+_

Sin l(1) 2(1)+:(:)3+t


(:)5
+
:+
2009
__
-
6
3840

12(1)


H2
: :
; F




12(11) If l =-2-3 1)
and 2- =1+ 2)
and =7+5
(3),
wchavc hm(1)&2), =3 and =-2 whch satisics(3)since
R.H.S of(3)=7+5(-2)=-3=L.H.S of(3).
4 and interS t and the ordinates ofE is(4,-1,-3).

= =

in = ,WheF p isthc sb


=
=4

and Hebe have at


Let S. and Hn denote the amount of savings that Selina

h year respect cly.


the cnd Ofthc

S"=1000+200
6 whcre
41( D(1 5) >5
=1000 100+100(1.5)+100(1 5)2+.

=lllCXl+

=looo+2oo(1.5"-'-l)

When I1, > S,, "



+ zoo,

looo + 2oo(1.5"-5 - l) > tooo


i.e. zoo(t.s'-5 -r-n) >o


=12 _

From the GC, the value of /r is 12'


2 7 RJC JC2 Prclinina ExaninatiOn


H2Matttmatics PttCr i 0740/111) PagC 8 of:0
Let z* denote the interest that Hebe gets in the P y"rr.
Then ru,, - 166(1.5t2-s-t = I I 39.002s,
)
so Hebe receives $l 140 in interest at the end of the 12tr year.

^ n>12, ( t\'-"
rnenlor un-lr) t0.
The total investrnent yield from t}e 136 year onwards therefore

cannotexceed *ut?
F!- -Lr..
2'rz'
Hencg ttre maximum value of Hebe's investrnent is
I
Hrr+r-u,r=$4790 (to 3 s.f.).

14o)
x=ct,y=c,wherec>0
t
dr dv -c
dt -'dt i
. _-l
dv
a-7
at e(cp,9),,= p.9=a
"dx p2
U p)
Equation of tangent at the point 1 _ p is
c -1 ,
y__=_=lx_cpl
p p-'
P2y-cp=-x+cp
x+ p2y-2cp-0 (proved)

0)() When the tangent mcets the x axis at


:

coordinates Of
(29,o)
en the tangent mcets the Laxis at B:

coordinates OfB is(o,


)

Area oftriangle O =:x


)X(29,)=2
2

"1 FT

9=




Coordinatcsof2=(


,1



Gradient oflineBg






Equation ofline32 1S =Lx+



As(



=Q ttrdinates








iS=:X(


Areaofdangle 20



)=il



) (








Gradient of tangent atR is +.




R:


Gradient of normal at 12





Equation of normal at R is






-9 - 12 (, - "r), i.e. ry' c - rj (x - cr) - - - -(l)



'(c\



t[*



When the curve meets the curve again at ,



;,1'





, =9s intoequation (l),


substitute x = csand






9-c -r3(o-"r)
s
r-s=r3s(s-r)
-l= r3s

.'.r3s+l=0 (proved).

Alternative sotution to (b)


Gradient of normal at R is 12.
_c cr -cs
c
GradientofthelineRs=
s r = sr = -'
cs -cr cs -cr sr
I
Since r2: - , therefore r3s=-l = r3s+l=0
sr

End
21X17 RJC JC2 Prellmina Exarnination
H2Ma atiCS Paper l 740/01) Pagc 10 o 10
RAFFLES JUN10R COLLEGE
JC2 Pre:iminary Examination 2007

MATHEMATiCS 9740 02
Higher 2

Paper 2
19 September 2007

3 hours
Additional materials : Answer paper
List of Formutae (MF15)

READ THESE INSTRUCTIONS FIRST

group on a[ the work you hand in.


Yv.lle )royr la.n_te and CT
w-rite in dark blue or black pen on both sides oithe paper.
You rnay use a sofr pencil for any diagrams or g.aphs.'
oo not use staples, paper ctips, ir6ntfunters, gtie Litrection
nuio.
Answer all the questions.
Give non-exact nurnerical answ-els t-o 3 significant fgures, or 1 decamal place
angtes in degrees, untess a different:?rregt in the case of
tevet of accuLcy
You are expected to use a graphic calculator.
; il1lfi;; in the question.
answers from a graphic carcurator are.arowed
:il.tr:x"t unress a question specificary states
\ /here unsupported answers from a graphic
calculator are not allowed in a question, you
present the ma,rematixl are
ffiH".:: ,iNg Lil"-r-iicar notarons-ail-rli"ti[,ru't ,
"t"p"
You are reminded of the need for clear presentation
in your answers.
The number of marks is oiven in brackets at
I I the end of each questpn or part question.
At the end of rhe examinition, t""t"" rrr-i.i,j,
*o; ;i:"re;?Hil"r.

This document consists of 6 printed pages.

IUFFLES JLTNIOR COLLEGE


Math Department
2

Section A:Pure Mathematics 140 marksI

O The X nulllbers z and s h that z= and =H3 ,whc md

b are real.G"en dmtz = ,flnd the exact values ofa and b. 131

( ShOW that
COS
Cr)(w C + w
(W"
i
)= ,

where and b are real nmbersto be dete.l.lined.

Hence flnd the roots ofme equation

z6_ z3+1=0
in the form of reia .
61

The follo ung systcm ofltx r equations is given:


2x-2 +z=-4 - 1)
2x+3 4z=1 -2)
-3y+z=2 --3)

0lVIng i
n f this sct Of
[:
:Ittl : 131

l and arC givcn by equatlon(1)and (2)


(b)ThC Cartcsian equations of plancs

_ respcctively Find a vectOr pCrpendiCular to both dlc Ormals of l and 2 121

Hcncc,ftnd
l 2
21
(D a VCCtOr equation of the lim ofintersectiOn of and

WhiCh pasSCS thrOugh dle


(iD in the fOHn r.(:=1,the equation Of the plane

t:) d ttrpedt
"
4 and 121

" (L

974 2ru7
"2007
3

3 The functions fand g are defined by

f:x-+-2+ -f- . xe[


- xz- +2x+3
g:x-+e', xe0,r<0.
(i) sketch the gaph ofy : f(.r), stating the equations of any asymptotes, the coordinates of
any stationary points and any intersection with the axes.
121

(ii) show that the composite function fg exists. Express fg in a similar form and state its
range, grvrng your answer exactly.
141

( D ShOW ht f dOes not have an lnverse_nef tion f has an mvev ifits dOmttn is
restricted tO x .Fm the largest possible value oft and deflne the inv_mncuon

f-l corresponding to this domain for f.


[51

Water is being purnped intO a nlmadon dc cc at a constantrate Of 5 1itrcs per nunute The
flitratiOn device proccsses dle watcr and discharges it at a ratc proportiOnal tO the volurnc
of watcr currently in it At timc r minutcs,thc vOlumc of watcrin thc device is
litres Thc
ratc ofdischarge is always icss han 5 1itres pcr minute.

O WH wna d d cquatbn esc ng ttc ab mOdeli oM




#and

11
(ii)By solving the difFercntial cquation in(1),ShOW that the gcneral solution is

t = eel l where I and t are positive constanrs.


!F- [31

(iii) Sketch the family of solution curves representing the variation of v with respcct tO
time in this context.
131

The filtration device is initially empty. When r


= 5, the volume of water in the de
2 litres. Find the volume of water processed
by the device when , = I .
141

RIC 7
974002S 7
[Turn Over
4

Section B: Statistics [50 marksl

Find the number of different ways in which 'CONNOISSEUR' can be arranged, if



(i) there are no restrictions,



(ii) a vowel must be separated from another with exactly one consonant.

Aled and Ben are involved in a duel.The rules ofthe duel state that hy are to pick up a
pic and ow at each other sinultaneously.If one or bodh are hit then the duel is over.If

bo they repeat process.Suppose ttat the results of the dlrows are
1 ss,then
indc"nde and that each throw of N d Ben has p babl ,and each thow
of Ben hittmg Alod has probability .

Find the probability th


111
(a)Al ed is not hit ater thc irst round,
111
0)bOth duclists are hit ater the iri round,
(C) ducl ends atthc round
3

A medical statistician vnshes to cany out a tCSt tO SCe if thCrc is any correlation betwccn

W lltt :
i1ir )beloW
:

Of and
One particular data point has becn reCOrded incorrcctly with itS Vaues
Ill
interchanged ldCntify thc point

(i)Makethenecessarycorrectionanduseasuitableregressionlinetoestimatethelength
of the baby whose head has the circumference of
(a) 34.5 cm,
(b) cm. I3l
45

(ii) Give a reason why the estimation found in (i) part


(b) may not be a good one' tll

lucky dips to customers'


As part of the Christrnas promotion, Bishan Gift Shop is offering
a customer who
A customer can choose to aip ior eitr,er a small or a big prize' on
average,
aip, f*,t*af prize has a one in five chance of winning and one who dips for a big prize
has a one in ten chance of winning'

Inaparticularday,80customersdippedforasmallprizeand60customersdippedfora
that day
of prizes won_on
;ig-p';. Find th!'approximate distriLution of the total number won on that day-
*?'n"*" nna * app'roximate probability that at most 20 prizes were
16l

9740j02 07
RJC 2007
5

Thc studcnt population of2,54Xlin ajunioroollege ls made up ofmalc and female students
who arc cidlerin JCl or JC2 as follows:

JCI Students JC2 Sndcnts


e 750 650
Female 550 550
A sample of 50 students is to be taken ftom the population to investigate the number of
hours of sleep a student from this junior college has on average per night.
(a) State and describe in detail a suitable method of sarnpling. I3l
(b) The students are now listed in alphabetical order and they are numbered from I to
25@ consecutively. A number betw,een I and 50, inclusive, is selected at random. The
corresponding student and every 50ft student thereafter are included in this sample.
(i) Identifu this method of sampling.

11

(ii) Give an advantage of using the method of sampling you have described in (a)
rather than this method of sampling.
tll

l0 Defects are found in toys manufactured by Toy company. An average of 20 defects is


found per 1000 toys. Ifa toy contains one or more defects, it wilr be discarded.

(i) It is required to estimate the percentage of toys that have to be discarded. Corsider the
following statement: Since 1000 toys contain 20 defects, approximately 2yo willhave
to be discarded. State, giving a reason, whether you agree or disagree witr the
statement.
111

Find the probability that


(ii) a toy will be discarded;
121
(iii) one out of three toys will be discarded;

21
(iv) there are more than the average number of defects in 1000 toys. 12

An aptitude test for deepsea divers produces scores which are normally distributed on a
scale from 0 to 100. A random sample of 160 divers were assessed, and each of their
.individual test scors, r, was recorded. The results are summarized
by
\x =er2o, I(,-;)' =3577s.
(i) Calculate unbiased estimates of the population mean and variance.

21

(ii) Assuming that the unbiased estimate of the population mean obiained above is equal
to the population mean, find the value ofc such that there is a probability of 0.05 ihat
a sample of 160 scores has a sample mean that differs from its mean score by more
thanc- l3l
(iii) Comment on the validity of the calculations if the scores had not been known to be
nonlla y disdbuted
121

7400N07
[Turn Over

12 A hospital sees a number of babies born. For each gender, it may be assumed that the
weights of newbom babies arc normally distributed, with average weights and standard
deviations as given in the following table.

Gender Average weight Standard dcviatlon


Boy 2.9k5 0.6 kg
Girl 2.6kg 0.4 kg

(a) For each of the following caseq state, with a reasog whether or not a nolmal model is
likely to be appropriate.
(i) The weight ofa baby chosen at random from all the babies bom in the hospital.[21
(ii) The total weight of a randomly chosen baby boy and two randon y chosen baby
girls bom in the hospital. l2l

(b) One baby boy and two baby girls are chosen at random from the hospital- Find the
probability that the average weight of the two baby girls is at least 500 grams less than
the weight of the baby boy. I4l

13 The widths of a sample of eight beetles of the species 'genus promethezs found in Bishan
Skeet 21, were measured and found to be 10, 15, ll,a, 13,14,b, and 17 "mm, wherea and
b are both positive integers , and a > b. In addition, it is also known that the unbiased
estimates ior population mean and variance of this sample are given by
i = 13, and
'7 s' =]rcspectively.

(i) Find the values ofa and b. l4l

Previous extensive measurements of'genus prometheus' lxetles had shown the width to
be normally distributed with mean 11.5 mm.

(ii)
- Carry out an appropriate test at the 5olo level to find out whether the beetles from
Bishan Srreet 2l have a different mean width from the main population' t4l

(iii) updated the population standard deviation of widths of the


' ' Entomologists have now
,
g"r^ pr6^erherzs, beetleJ to be 6 mm. A test is caried out to find out if the beetles
from Bishan Street 2l have a larger width than beetles from the main population. Find
the smallest levet of significance (to 4 decimal places) which-would result in rejection
of the null hypothesis. I4l

END

9740 2Sr07
2007 JC2 Preliminary Exam
H2 Mathematics Paper 2 (9740102)
Solutions

Section A: Pure Mathematics (40 marks)

1(a)l methOd l
lz = 13bi
z=1-13bi
-5i
C en:z=3
l+2i
(1+21)z=3 -51
(1+2ix l-13bi)=3 -5i
(1+26b)+(2-13b)i=3 -5i
Compare real and imagina partS:
1+26b=3 (1)and 2-13 =-5- (2)
From(2):b=

Frorn(1): = )=5#

Method 2
z=3
-5i =
z =
+51
1+2i l_2i
Since z*=
b
+5i=(1-21xl+13bi)
3

3 tt Si=(1+26b)+(13b-2)i
Compare real and imaginary parts:
1+26b=3 _(1) and 13 -2 =5
(2)

F m(2)b=



T:
2

2

Method 3


3a-5i 3a-5i l-2i (3a-10)-(6a+5)i
l+zi l+2i l-zi 5

Given: z* : w
(3a-10)+(6a+5)i _, .

5
-,-l3Di
(3a -10) + (6a+5)i = 5 +650i
Equate the real and imaginary parts:
3a-tO:5 -(l) and 6a+5-65b -(2\
From (l): a=50

Sub into(2):
=7=f:
C eFp )

=w2"_ F+e r)+1

="2 _ 6S +isin tt cos(
)+i Sin(_ ))+l Where = 2and b=1_
=1 _w" Os +iSin tt cos isin
)+1
=1 2cos w +1(shOWn)





2 7 RIC IC2 Prc Ininay Exalninatlon Papcr 2


H2Mathcmatlcs 974002-Pttc 2 of 10
2o) (z -z r :-4\
The augmented ,,.utri* =lZ I -+i f
[+ -r ti2)
I

(t o -o.5io)
The RREF of the augmented matrix = I 0 I -l i 0
[o o o it)
I

The final row of the RREF shows that 0x+0y+02 = l, which implies that the system
of equations is inconsistent. .'. The equations do not have a solution
The 3 planes represented by the 3 equalions do not intersect in a point or line.
Furthermore,sinceneitheroftheplanesisparatleltoanyother(...neitherofthe
normals is parallel to any other), the 3 planes form a triangular prism.

(b)

.'. avector -L tobothnormalsof z, and


"r-(j)

2r+3y-42=l
--42)
When z = 0, 2r = 4 ---{4) -2y
and 2x+3y=l --{5)
(5)-(4) givesy = 1 and x = -1.
.'. (-1, l, 0) is a poinr on both r, and ztr-
/r\
I z I is parafel to both r, and r,.
\2)
... avecror equa,ion or/ is . =[iJ-r[;J, ). eD.

(1)

A vector normal to z is

A vector equation of is



2
ly:FTr2

3(1)

-1

15



Range of3 L=(0,1]and Donlah oft Df=(oO, )

Since Rg DF,COmpositc functiOn fg e s _

fgO)=f(C )=-2+ , xC ,X 0

fg:x -2+e2 C 0
+2c +3 'x

Range of fg= [-11,


:)

(liD

Sincc any hoHzontal linc y= Whcre t c(-2,-1.5)cuts thC graph of y=( )at
more than l poin fis not onc one. Hcnce f does not havc an inversc

OR Sincc the horizontallincy= c(-2,-1.5))


::(Or any Other valuc ofy such that
cuts one. Hence f does not havc
an inversc.
"ph of f at morc than l point fiS nOt One

OR 0)= -2)but-2 0,hence fis not onC One and thus fdocs not havc an
:=
inversc.

Largcst possible value of l=-1

2u17 RIC JC2 Pre minary Examinatlon Papcr 2


H2MathmattCs,740102-Pagc 4 oF10

Lct y=f( )= 2
2+2x+3'

1

x2+2x+3= l
+2

( +1)2+2=

(since x < -l)



-2
+2

f 1( )= -1-111lF12 , -2 x -1.5

=5- , >0


51:
=fl dr
d

=-Lhls-kvl=t+c
+ll,rls-al=-t6 -1,.

=5-kv=Ae-tu (sinc.e 5 >fv,5-tv>0 so lS-frl=S-t )


=:(5-/c )
=:

For the device, when t = 0, v = 0.


I
Hence, o =7G_rl)+ e=s.
r = 5, v = z. s" z= (s _ se_5r) k = 2.s00 (4 s.f., from GC)
f =
Amount of water in the device after the first minute

= : r.836 rit
*o(r-r"-"m) ",
Amount of water poured in after the first minute : 5 litres
Hcnce,amount Ofwatcr prOnossed=5-1 836 1itrcs=3 1 6 1itres(3sl)


24X17Utth



Section B: Statistics (60 marks)

5(D
Number of ways=

=4989600

Numbcr of ways=75, X
ttX3 =32400

6( Pttl3d iS not hio=1-

( P( th arc hit)= x

(0 P(either Alttd is hit or Ben is hit or bodl are hit)


: pA(- ptY'(t-rr)" + pr(t- p)" (t- pr)n-' + (l-pr)'-t(t - pu\"'' p^p,

7 The pointis c=51, =34


(1) A suitable regression line is J=19.722+0.86981 .

(a) hen =34.5, =49.7_

(b) When c=45, =589.

(li) 11 extrapolation has gOne beyond the intcrval ofthe provided data_
ereby,relationship benccn thc variables may not follow thc smc patem as dlc

Let rdenOtethe random variablc representing d number oFcustomCrs Who WOn a


small prize,out of 80
B(80,0.2)
Since =80 is iarge,r =16>5, (1. )=64>5,
N(16,12.8)apprO matcly.

Lct ydenotethc random variablc represendig the nurnber ofcustomCrs who won a
small pHze,out of 60
y B(60,0.1)
sI c =60 is iarge,4 =6>5, (1-p)=54>5,
y N(6,5.4)apprO mately.

/ No2,18.2)apprO mately
P( +y 20)=P( +/ 20.5)by continuity correction
=0.363
Stratified sampling.
JCi Students JC2 Smdents
Male 15 13
Female

Within each strata, carry out random sampling, or systematic sampling to obtain the
required numbers of studenB for the sample of 50'
Systematic SamPling
Stratiflcd sampling in(a)is able to g e a good reprcsentative sample of population.
24X17 RJC IC2 Prclimulary Exalninatlon Paper 2
H2Madlmat6 974 02-Page 6 oF10
10(1) Disagree as each toy may contain more than one defecl

Let rdenOte the random vanable representing thc nutnbcr ofdefects in a toy.
P 0.02)
Rcquired probability=Po 1)=1-P =0)=0.0198(to 3 s.o

Oil) Let ydenote tt random variable represcnting the nulnbcr oftoys,out Of3,that will be
discarded. y 3,00198)
Required probability=P(y=1)=3ci P(/ 1)P(y=0 =0.0571(to 3 s.o
(iV) Let denote the random variable representing the nulnber ofdefects in ll100 toys.
LP 20)
P( >20)=1-P(7 20)=0.441 o3s.o
11(D

U ddimateJttpoputtmmean I
= =".
Unbiased estimate of the population variance l,s, = =35775=225
-1 159
Lct dcnote dlc random vanablc reprcsenting the sarnple rnean aptitude test scOrcs of
160 randomly chosen divers

Then,F N( , ).

Assuming = =57(6bm(i))and estimate bys2,


we have N(57,
ma ly.

Now,Pl =)app
5J> 05
)=0
P( -57 )+P( -57> )=005
P( 57-c)+P( >57+c)=0.05
= 2P( Y 57- )=0_05 (by symmetly)

P( 57- =0 025
)=
57- =5467576863(ushg hvNonn(0025,57,N 25/160))
=, =57_54.67576863==2324231373
Hence, =2.32.(3s_1)

If the scores had not been known to be normalry distribute4 we can still apply the
central Limit Theorem to estabrish the normarity of F since z: 160 >
50 is large.
Hence the calculations in (ii) would still be valii.

2
L
31 : :

12(1) Normal model inappropriate as weighs of babies bom belong to two different
( distributions.
OR
Normal model approoriate as the weights of babies belong to normal distributions with
means and standard deviations such that therc is a huge overlap of the probability
density functions as shown in the graph. The required distribution would be uni-modal
and symmetrical. A normal model would thus be appropriate for the weight of a
randomly chosen baby.

12(b) Normal as the sum of independent normal distributions is normal


Let C and denotc thc random variableS rcprcsenting the weight ofa baby girl and boy
rcspcctivcly.

Thcn C N(2.6,016)and B N(29,036)


3_q+G2 N(0 036+ :)

Q
P(3-
05)=0.382
Li
g

13(D =13 =),Ex=13(8)=104


10+15+11+ +13+14+b+17=104

=> +b=24. _..




(1)

2 7 RJC JC2 Preliminary Examinalcn Paper 2


H2M matlcs 9740/02-Pagc 8 oF10
)2=54
s2=21 =)'EX2_:(DE
7 8-1 7
=
],2_:( EX)2=54


){102+112+1524 2+132+142+ 2+1,2}_(104)2=54
2+ 2+1100-1352=54
=)
2+ 2=306.._
=)







(2)

Subsdtuting(1)int (2)yieldS:
2=306
(24-b)2+
b2_24b+135=0
o-15xb-9)=0
b=15 or b=9
Since > ,thus we havc =15,b=9

Since the population width is normally distributed, we carD/ out a two+ailed r-test at
5o/o level of significance.

Let X be the random variable representing the width, in mm, of a beetle of 'genus
prometheus' species.
H,: p -11.5
Hr: p*11.5
Since o2 is unknown, and z is small, we estimate o2 by ,'.
Significance level is a = 0.05 .

Under flr, , = !.r-+ - r<7), where 7 is the sample mean, and s2 is the unbiased
estimate of the population variance,
po= Ll.s,s = 2.77't,n : 8, and i = 13.

Using a T-test,
p-value : 0. 170 (using GC)
Conclusion:
Since thep-value:0.170 > 0.05, we do not reject f/0.
There is sufficient evidence to support the claim p * I I .5 at the 5oz significance level.

With known population parameters, x - N(r|.5,62).


Then we have X - Ii(l. 1.5. ]!).
,8,
Ho: p=11.5
Hr: p>11.5
Signifrcance level is a (unknown).

2 7 RJC JC2 Prelimina Exarninatlon P r2


H2Mathcmallcs 974 02-Pagc 9 oF:0
Under I1o, X - lV
(^, +),where 7 is the sample mean and
=11_5, =6, =8,and =13

Using a Z test, z= =11;


;
=0 707(3sl)

Value=0.23975(using GC)
In order o reJect ,pV uc=0_23975
Thus,the least value of =0.2398 d.p.)

80 ee
ee

2 7 RJC,C2Prc rllin Exalninatlon Papcr 2


H2Ma atlcs 9741m2-Pagc 10 or10
' VICTORIA JUNIOR COLLEGE
PreEminary Bomination

MArHEMArtcs 9740101
Higher 2
Paper 1
ptember 2007

. 3 hot.r

Additional mataials: Answer papcr


Gnph papcr
List of Formutrc (MF15)

READ THESE INSTRUCTTONS FIRST

Write pur name and CT groug on all the uork pu hand in.
Write in darft blue r blact pen on both sides of the -paper. I
You may use a soft pcncil for any diagrams or graphs.
Do not use staples, paper clips, highlighters. glue or corlection fluid.
Ansrnier all the questinos.
Give non-eo<ect numcrical ens{rers cdrect to 3 signiftant figures,
c 1 dccimal place in thc
casc of eogles in dcgrees, unress a different ua of accurac/is spccified
in the juestion.
You arc qpected to use a gaphic calculatq.
unsupportcd arurvers from a graphk caknhtor are a[ored unlcss question
a spccifica[y
states otherwise.
whcre unsrippo.ted ansrrcrs from a gnphic carcurator are not
aflorcd in a question, 1ou are
required to Present the mathematical steps using mathematical
notations and not caiollator
commands.
You are reminded of the need for clear presntation
in lour ansucrs.
The.numbcr of marls is given in bra&ts
At the end of the qamination, fasten al
I at the end of each question or part question.
foiri iort s.curety iogether.

Thjs document consists of 5 printed pages and I blank page.


Yictoria Junior Colcg @VJC a)0il [Turn over
The vanlt of a hk b lgctcd by I 4-tplo tsy, (a, b, c, d), ufrcrc a, b, c ud d irc
coostaotE Tbc dircclo of ttc bonlr ncods to li
away for an urgcot mecting and lcaves
thc tcy with 4 ecoic corployccs, Each oflbcrn ls givcn a 2- tuPlc share (x' y) srrch tlut
y= ax +.,[Exl +2t*'- d. Ttc sharcs are disibtrted as follows:

Employce
Scni Share
0, 2
Brenda r-2.-59)
Cbd (3,21
Debbic (1,1

Find thc kcy ofthc vaulr 14]

2 A gcometsic p'rogusion bas firs tcrm a and sccord tenn &, whcrc a aod 6 arc non-
,rro corstaDls. Givca tbat drc sum to iofinity of thc scrics is o + 4d' find

(r) z,
b
t3I

(ii)
i(*l Pt


%M tt ineqdity=L


3 .

Ded On set Of [31


":

The pph ofy= )iS Shown below.It passes tt ugh (0,5)and one Of its
asy tote is y= +1.

Y=-r+ I

] +u+5
Givcnthatf(x)=
-rT,

@ . find thc nalucs ofp and g,


(ir) sketch the gnph ofy = f(x).

State the range of vrlucs of t such fut f(x) = I 1t"t oo rcsl solutions.
5 Find tt roO"oFtheequatioEltf22-1).=l hthefom x+i leaving yow anwe
in surd forrr. [61

The positivc numbes ro saris$ thc rclarion

= +3, for"=1,2,3,...

As" ,X. .

(D Find tt exact value of . 21



,t
(ID PrOve dlat( 2= .
121
"_
Cli HCnceD or o show that if > fOr a of then
,

X +2> 131

O WritedOwn fd e 8 .
11

(lib ShoW that u dittmm Ol.quatiOn


3- tan x=2sm e
may be reduced by mealls ofthe substimon =.s r to

=2 sln sx e
.

Hcnce ttd the gend sOl n OF the direredal equatiOn9 81Vlllg yOur
answerin a fOllu cXpreSsing in te Of . [6]

FindJ

ectwe Cis demed P_ically


2
X=(1+ )3, =In(r2), _1.

OhOWthatthe m ofthe JOn enCIoed by c 1 =Qx=l the


Can d hthe m
)d ,who and

be daermincd.
l i "cOnstmts b
t4l
Hcncc cvaluate this area
ttl

[Turu over
ll

, (D Givcnthrr(r) =rh,showttst f(r- l) - XO=


Ofu t21

(lD Henccfindtbc sum toatermsofthcseri*


*-* * $* ... . t4I

(tD Dacrmirc thc smr to iofinity of thc scrics. - ttl

(tv) Showtiat r *|*|*f;+... isrcssttanz. 11r

10 Thc position vc<lor of lhc poinl,{ is 2l - k aod egcquation of-thc liac I ir

.=[-{).,[iJ
whcrc ,l is a paramctcr. .

Find thc position vcctors of thc points B and c, both lying on I, such that thc lcngth
AB= AC= 10. Isl

Givca that tbo point P is 6c nidpolat of 8c show that thc equation of lhe Planc rr '
rryticl cootaim I md is pcrpcodiodar to ,{P is

' [iJ ",


whcre a is an intcger to be dctcrmincd.
21

Thc planes fitrl th havc cquations




5 30d r

rerpcctivelY.

(0 Vaify6at,l licsinboth xtrr S.


(D Dcfcrminc thc position rrccfor of D, tie poiot of interscction bctrpccn zt
'
and rr,
(lii) Hcrrcg or othcnrise, find the volumc of the tctahedron ABCD '
I
I Volumc oftctahcd*o = ] , basc arca x pcrpcadicular hcight ]
yJc 2.ooJ Nni Ex"rn (fl2 flqlh, f.f* l)

0 0iVen =1+m.'Sb W that


(3) ( S2
+ =0,

O ccos2 10_sha =Q
HCn fIEldthC Mdau 'g scHcs foryup to and including thetm hP.

CtD L Adt regitOn botnded the ctwe y= , L = '



s and the mtated mpletev amund tt xtlin.Use thc
"
CXP
n r =1+tanx" btaintt WEOpcmate valuc forthewlllncof
rcvolution, IS

12 ( A sequence is defircd by

=1,
"= "_:+2",


(D Evalunte =1,2,3,4.
"for

0 By nsidcting
"+1,give acottecte fOr "in"nns of
.

0 Prove your ccm by h fOr 1.



0 Giventhatl = 1)o+ and P=:"20+ 2, suln
:

ofthe rust a te_ fthc serics

(l x3x 5)+(2 x 4 x Q +(3 x 5 x 7)+.... 14

(lx3 5)+(2x4x6)+(3x5x7)+ .+HJV+IX +2)=4 +lx47+lx47+5

ind in tentls of
[31

VICTORIA JUNIOR COLLEGE
Preliminary Examination

MArHEMArtcs 9740/02
Hlgher 2
Paper 2
Septernber 2fi)7
3 hourc

Additkrnal materials Ansnnr paper


Graph paper
List of Formulae (MF15)

READ THESE :NSTRUCTIONS FIRST

Write pur name and CT grorp on all the rrork you hahd in.
Write in dark bluc or black pen on boti sides of the p4per.
You may use a soft pencil for any diagrams or graphs.
Do not use stades, paper clips. highlighters, glue or correction fluid.
Ansrver all the questirxs.
Give non-o<act numerical ansl,ve s co.rest to 3 significant
fgures, or 1 decimal place in the
casc of angks in degrces. unless a diffcrent lard of accuracy is spcified in tfic qucstion.
You are expected.to use a gnphic calculator.
unsupported ansurcrs from a gnp0ric calculator are allorived unless a question specifically
stats otherwise.
vvhere unsupported ansrrers from a'gr.aphic calculator are not altqrved in a question, you are
required to present the mathematical steps using mathematical notations and not calculator
commands.
You are reminded of the need for clear presentation in louranslrrers.

The number of marks is givtn in brackets [ ] at the en{ of each qustion or part question,
At the end of the o<aminatio.t, hsten all your rcrk sccurely together

This document consists of 5 printed pages and I blank page.


Vlctori8 Junior College @VJC mo7 [Turn over
Section P 140 markgl

A spherical balloon is being In0ated In euch a way th its volume is lncreasing at a Gousta'Dt
a-1. At time t seconds, the radius of bau n is r cm.
rotp of 75 cm3

Find,77hen =iCms-1


Cm 8
Find u rate OF lncrease of the ttJ B Of the banoon when

=: 3nd =4rr
bmute olume and i

bndtheldireremdequ
that y=Cr2+:38a perd

::-2
+6= _ 121

m
(i) a porticular aolution wbm curve has o ssymptote Y :0. 11
t --) @. 11
(li)the range of duc of for wbich v
typ d mem
1
L
solutiou in O and

to aud includiug the term h d'

a,P""d in ase'odiag Pourera of o [31

# ecaaosion ic valid. tU
State the eet of ralueo of z for which the
Findl in t-30f :the coeadent of r in eJ<patsion iu asceuding Powors of z
For

1-
- 141
siurPlifYing Your tuEwer'

Tho functions f aod g a're defined W


Err- s+lDr, lD>0''
g:tr-+ , O.

l,
(i) Sho", tbat f-r ldsts. i

(ii) FiDd the wlue of o gu& that f(c) - fr(c)':
(tii) Sbo*, thot fg xists. i

(lv) Defae fg in a similar form a,ud state ite ran$e'


Ou tho sono Arga.nd di8Srsr& aket'h the locl

(t) lz'zt - r0il : 25'


(ii) lz - 2oil : lz - 481.

h
1
I
;

. Section B: Statistica [Od marksl


I

A factory hae 880 operators. A aurvey is to be *itn A0 operators ftom the factory.
"oodrdt"a
Describe brie0y how a systematic aample caa be ob{ained, t21
Of these E0 operators, 5 raodorrly selected forlaa intewiew. Dscribe b,riefly hor a
a.re
random aample c"" be obtained.
i l2l

iables X and Y a."


"u.U[tngt
I
^, X N(2s 2d),,
"
- (t,+)
aod P(X < zp- o,f2): 0.159, conect to 3 eigd6c4t 6gure* Without the uce of a.graphic
calculator or statiotical tablea, 6nd P(0 < X -ZY f p <2o). [4]
i

.i
Miss chia is a member of Koog see car shar'r,g. i Eech time she takes a car out from
Kong see car sharing on a saturdqr or a sunday, efie keepe it for a mean of 8 hourg and a
etatdsrd deviation of 2 hours. on any other day of |he week, she k*pc the car 6r a mean
of 3 hours and a standard deviation of 0.8 hours. i
Apart from other charges, Kong See Car Sharing A6rg.o , ,r"ge fee of $ l0 per hour on a
Seturday or e Sundey, aad $ 12 per hour for the ot{er days of the week.
If the time Miss chla keeps thc car follows o ror-oi distribution, 6nd the probability rhat
the usage fee occeeds $ 360 when she takes c"r d,ut oD aeven consecutive days doing it
once eech day. "
; lSt
State if the central Linit Th reIIl is uged in the probability.

a 43
a tt h agame,
T dd
b b 1 that a ayer,
"obtained.

in3aprta m b o

F b]l
121

( )Det ine the prObability that a 3d playeriAi Du,reqtilr"mOre thatl eight roL
ofthe die tO wh a prLe. i 141

[\rrn over



A v.ehicle ratsl coEpa,Dy has vaog whi& it hirea out for a day at a time. The number of
demands per day for a va[ follov,8 e Poiseon distribution $rith etandord deviation /c.

(i) If the probe,bility that no tan is htued out for the day is 0.105, determine the value
of h. 12l,

(ii) usiDC a suitoble approcimation, calcularc tbe probability thst there is e demand
for more thoo 5 nans in 6ve coosecttive daya. [31

(iil)
' ' Find th lest nuobcr of vrns that the compoay ahould heve so that, for each dey,
the probabitity thet tbe demand for the blre of a van ftrr that day hac to be refrrsed
is leas than 0.001' [31

According to ar overseas study, achool chil&n aleop aa 8r'6.age of 6'5 hours es'h
Dight'
Mr Fbrnfrw belienes that the children in his Bchool sleep anea less than that.
If a 5 % eignlfico.ace test and a 10 % sigaific$ce teet are conducte4 stete which of the two
tests is more likely to reject the ouU Uip"tlolt
qrhen it is actually true' [U

In a preliminary study, Mr Fbrngow took I ra'ndom aa'mple of 6 children from his


echool'

Ite number of'houra of aleep ea.h child gets at Eiglit was reported as:
5.5 6 6.2 6.3 6.6 8
evldetrce supports Mr Farngour's belief'
T6t, at tho 5 % tevel of sigrrificance, rrhether thls
mcde.


stating clearly ary assrrmption [51

Mr Faragsw conducted a furthet atudy involving a random sa'mple of 60 children' The


sleep eoch &ild gets
unbiased-estimetes of the meen.and the variance of number of hours of


at night are reported as 6.3 and 1.21 respectively'
Tbst'.8t the 10 % lerel of signiEcarrce, whether thia neq, evidence Gupport.s Mr


Fa^rn8ow's
'-.
belief. t3l


the opplee each
12 A ststistician bought applea ftom o farmer aod found tbat the heaviest 5% of
has mass more thaa 200 s. He chssificd these aprples as 'large"
the noct 5090 as 'medium'
*a tn" r*t of the "pptea- r" lr-"tt'. tt is givet iLt
the mas' of a randomly chosea qpple
follm a nomal digtributioa witb mean p g and atqodald deviation 6t0 8.
(l) Dstermlne the maximum mass of en apple to be clasdfred as 'small'' [31

(li) A raadom ao^mple of a opples wts chosen.


(a)Whena-S0,useoguit8bleapilrotirirationtofindtheprobabilitythatless
thao sevea applea urere'larget. [31

(b) Givea that M denotes the number of 'medium' apples itr the aa'mple, atate
the mean pr end t aria[ce oz of M, giv'ulg ]our atrawent in terms of n'
121

Whenalslarge,6adtheralueoflcforwhichg2%ooflhevaluegofMis

gester than p, + *o. 121





V 2007 11 ( 2 Mqlt tt
I

An iuraqtigstioo oa the nuuber of houra, t (correct the nearest hour), spqt gtudying for
or era,nirstlo! bD, dsht gtudots and the martr, thtry obtnhd was done. Hmam, a
value wue miesing from the report e.nd this wus by p bdow.

lte equatloa of..the regreasion lile of m on t ia m 0.94t+62.6.


Find the value of p eod wdte donra the equation of t ou rn.
regresaioa liue of
Deterqine the differrnce in mall<a obtained by e if he had studied for two Eore
houra.
l2l
Aoother three gtudatE were gunqad ad tbo -08 bu .

Give e sketch of the gcatter diagram for the data the eleven etudeut a,ud stete, with a
reason, whlch of the following modele is moet l

At m: a* btz
B: m=a+!
C: m:a*6lnt

:[=l and a and gtate



.'. {rt1 . X \lr rro n{ sldrcrrs
+ -+< K<4

5)
iltr
i cI
:
, 1)
f 1". r2 5,."C

2-1 c.
), k. ,: 2 ( C ) (t

2ral
: 2
e _2 sm : &
73 (lt et ) k=o : 2
.: \l: , c
C

n_k80: == 1) j r4r- : 2 CoS tC 5


+c
8 =2 ""+

Ic
=

kt 1,91) J--

= (1 1)

k321 2c (1 i 1)
= L( 1:) 3
1
`"'3
3 it : t = 1 JDi t 3 1
:
J:
C

6) tt) ltr rlo, r+r<, r.1 +4 =


3 _x F
1 `

... d- =,EE `

( 1)1)1
= Ct3), t4_
o(r-4-3 - o
61 s rt lii

},rc r. >o , < = -&+ lJE


(n) tr.+. )' - <z s (t t ,) - Cct 3)
3 ai -{
(i.r) I{:.>{r tt-J2o
0

.'. <r..., )' - AL > o o'rr . f.'l a,


I (rr.. -r{)(xr+r + o() >o
I xirr - .( > o C'.' r-r. + o( >o)
' f-l' u ,r'1 . f 1r+t1-i rt
! o.eal
- .-.- lrrr 74
Sinrc tr > ( E) Lrrt ) { rvh.. r=o, 11,t1l ;q
ain r{ I :q}r>4 +,-r ( Q=-1
rr...& x.r , t( 4 L.tr. ) d r:r, 11+11i . r

r+t ! 'l c'- I


) ) .
ca` ` C
",C t,-r.ro
S. -:)
_8 25 LC 1
``)= r ' (r++)l ='
1 SeC
u3

3 C C
tr' ic''t1*

L=2" `


't
StC g
` '




)=t


9)tt1 k_


10) R


= t=(:,)





=( 4)
1 _
1(7 1 1 t
'I
t (
1 L
1
=Llll
, 1

=
(
` .

= `"_` 1) )
1)


h r k.r'rs
.'. (-rr11f+ (rr-ra)t+ (-lr ra1|- = be
. t'-+r+l,o

: ( ( a-,) r o

ti`[:tr J= _1`P:
u."ns
'l-4
or. (j) . ,e,(i)
n.1 .t 3

di. (i) . fi. l-lt


.: r. (i) . 6-". l.f) r... B.rr)
. t6
gr'rc' ( lC
C t tFt )-16)] '
Ll-1 Ge"r rr' r. lil'r, r : c.{i,r),r
:

r.)
S l
7: .t "' '] l-ir) ' ti)' lir) . z.o*r.s r iGrr

:A )
'i ' ( -i) ' (i)' lf) . ..o-, :r ,Ar,
".
( ) 1
LS
14t, Ti
ls h,1 2 1lfj::


::
&


1 1


1:)
= II)
=4 (!:) 6i. O-C
:11(_ .Al)

' (*)
=41! tr"rr o-F. ii . o.

te l+!
+ frr. f,
. 4(1) 2

`
) lln.a Yol o? .t+,-1ar.j tr6jo
't*r'...lltorlp
- sI i I
-l gC . Dl r R?
- (rtrc6,lEi_,Jra
-l


I= 1+ Ff I'o'r
'i": "

(o) I = *,1 :o t't tr'' z' Y


rr)ti, lrrl ' .rv, v'+'ir
I

(tr{'rr) ) ' I I ,, .,:]. t1 :|, 11 I

oo*,1....,rg. | ..,) q.l'$r lh+t=i,urrl'( \rl;32 I

+ trccixyy. o I ... *',1.'=tI,-, I


.: (t+{*ir)! 9r + tr.crrl;-. o | .r, ) lrt ?" bc +ra sLtrou"{ r...1.,-r I
(r.c'r ) y" o *.nrr
"r H r . I {r. :l ,o p, i, {,rrc I
celrr) fl + 1''o tsr') .i {nc f,r ,.-g gy,
-( | fitr-itc ?F I
rb) D:?? .d r, i. \, z.n _r
(c.rt) {. + (z-rr)(-rrr)}. . *.o
I I

3 I *, rlas : ,1 + 2(a! I
cccrre ) p.r -(rnu)S +r;$."
(oCt) Sr. + 1zy - nal)$ so I = (?Er-r) + a:+r' 1

= z (:H,) -r
tu I
tjtr;,'
I

tr'. L n ..""
(.,il1fji--',).i.t11-r.a)g
rz!, -raszr)s,,"
I -': l.'["r.:;,] .* , I
+
IY":".;::':, t I
f,t r.o, r:r , *r,.r{r.,2, S, -g
| *, ,,rrrr) + (rrhr.) { (}r5r + _.
'' r(,+e)(-++) -1;'
I

v: til',:'i.'-?,f.'..:0d'"'
t ,i,
tr)
I

,p-.., l'*...- I| .
El,:".;:'..:'inn.x,',rrr*^*,
I
aff* = {ocm,) f',tr+r)+4(rn+r)+\6J
.- i.'tn+,) (n!t 1^.r 20) I

(*.' r,tt =
".f] y ' l {" cn. )(n+r)chrs)
, , $ J, u I **=
l
I

e r gl ( t-l+r!-irt)dr
+ -'r'' or
trrr'e)'r (3rtrll +-'r n(r+z)ki+r) |
+
[rrrrr)
= ,.ra ,ro, ' to rn+rxtn +)(rr) |

*.n..", n q,,ls zhr


|
Ittrrr.i) + orrrc, + (1rsrl) t.-+ 2N (r{r!X$J.{)
| - tr,r")a*+ rxr..* t +)(rr'r{sil i""-
''f.,
| = rt.r.*,1r* +.) ( .(+s)
I t otnlr+2)
| $n. trq (n+rXrrq+r)(r".rs)
| 2n (1'r' ('lrt+r')(rrtr)
'
= 2cr (+rr+r )( aq ra'l(kn+t)
|
I v{rqrr- Nr 2r
I

I
VOC
St4t A
4p f= ,In ,
l)(: v3 1=

el .)
ac o
1 1 1 t) (I .)
c.3. ,
=
.: f,is r-r ,tt.')
fr cu.r{

)t.
l
).

2) ,101 x

= :
Sth<a
R3F`:'0)s
r) =pf
+:
`is.
C ) .{3c.r . e : te')
C" r
.'. ,i = " eL x, o

`

(r, (1 _)
'
5,

`:
,3 )
3) : `

`= , 3

= 1 3X o,
= 1
`
,2 3.
CS
C
`
E . .1`

`P

=
ttl) 1
` .=1111`l
. 6,)" g+lkiiil) r-,)-...l)-,
: tlr!l]| ,,rr- "8 11` 3iq}=1
1

nll. _ :
" '
. .r(..r) L.-r tlar3 n-"1e. l. , r,r, =
t*t t trr-ln -. _{.rd wr : l+tJ}.
z.i
(r{rtt) (r-rr)-r.
.,. 05 (d,) : r
`C
7 rt : (r) trutzF:+ (r)(n{)n2Ft
C
NoL {r.r 4.rrovr. .1 ( } rn g,,rc,at)
. n 2*l t (rrr) r (nql .'. q'l(trt) ; -tr
: nt zr-r J

, * I :i:. .""*. xt_: ri -rr.. 16,.3 ,.r3 lic.


{'cF rin'.lar A, ]
t E.f \i. (E) rt y.7qr.?.rrnl
s 3
q(`) Rta = (0't
6s f,Fl)
6) rF\l|o' rr'
3-* rrt 4 r'iirt" u'ith
'J <i'r
rrrq 4ert
: 7(_ 9n
qrt l io .t6.t Fr' ,rc,",( t . CvcS ll{ nr.3
.n \
"
"8" 3)
r(itrt.. (.n ha ett.r *.alrn3 '( `2"`
L,r{ti .llc lrt n rr.o ekL b t C.r.n{cl un `en ber 1 1 os

. . roalcq nr-r5r. : .t|rR.t.r . 1\c o 1
tt ib 13 3 "lls
"J
. trlq.t co.rlrra.t $ilil 90 ol.r$,, \.,<- 0(1, )
bt.r cl.lar.
. . Rc = PC Y` 2)

Alttli . e rr.r,rt6 {6,1 I .t 80 +'? rrc\ 8 g

.?(..tr. . llr r rqla.r-.rr.' i...rtrh- l.{ X L..lc $( i.-D& ol &ni..dr r.-
to ncretlc 5 I&SL tc,'rb.,. Lr {\.{ ")
rr1 t- r, .i-.. .
et..
x^ t (rr).
1) N` (r) p(r r.) . 6.1e(
I 1 .__
2 :)

1:

a_* _ q.roS
1 T2 )= y. ! r. so (rrl)

NO

?` t)L t'
(6) t t Y a.i.l. {rG n.-rrce rl Jc .dr

lm = T- . '"" .,. S &I,
Y t/` 2)
{ f(o<r-r(7<qr)-- ?(o < t-+ < r)
\- r. trr. eel 't
S|n< e t3 tr.a61 >to/
= ?(o< g< t) .f - x ( \r. 161, rr.rrl) ..?l.tt
o.9 -r..g1 , o.,+l ( f.F)
=
P(Y>s) ! p<t, 5.r)

t{t I

8) Y l.-+G tt .16r.. .t lbsl


t{it t Lrrp $C (^? .rt ., w(.fa-J
o'19?
" (ti) l. { N a...t.- {}s n ".b.. ol y.il {\.1
. r\c c.-t..J hcr
X - N( l, o.rr)
y-N(i, 1.) P(x>r.l)<o.ool
lc+ T: lz (t,+ -- + Xr) {. ro(y.ryr) + t(t {N) >
".111
Ecq , 12 (s)r!) r ro(r)(8) F'.s e(, p(x1tl: o.qlr]l
= t+o P (I g'l . o.Sttrrq
Ya-(rl : tLr (9)(o.sr) r ror (a)t:r)
: q80 "' t,rl 1: $
ro'[ si X. iflt'-,, 1"t1
... T - r.(l\.rtlo)
P(i>360) : o"t6l t<l I dr-rc rq.",.r.r .8 \.*r.f rLrS r'<rr
6r\a rl tt' fG:c-r'r s(h..t ldr cr ilr.l .
(r"r.ql L--il ftc-r- rt .of ucJ. Arrqr. X 1r nqqt.
\rr ,F f-,.\.- {\, actr q\ <g"pt tu") ,ron
9)` ) tel 31.. V

n" _ 5
``' 'L: r,6'5


"
rr. ./. < a,s
X 0( ,t) r-^..r .f sr j".t.* <1 . S l
Qrt= 7( =2).= Trd ru*irc , T= +#
3)t qrr5 t - {.rl .,:.lt V. n-r g
3L
( '
P-Y.tc . o. Tolrt > o.os
O 0311 `
.: tl. .t tl c<i&lcJ . .n.( ri irrht
c
"^
t,
r o` J=:
c
r."Gt
rF _,J J_r:yrlv-.1
r"
`
`1

2 T6PRLu_ARY m n uN rAr WL 110 _


"JC2 H
rr) H.. l. ..E t= , = n 3
tl. :7r < r.f
{q . 9o3rp
tctl .{ 1;7'. $6.ae- . o1
t 1 '
``
rtlil^
0 )

{!i.r3 t-.ni i,rt\ .! 6. ( r. 91; , `: `
t-rrFt, o.ot:l < o..o 13
.'. il. -rr r$4.a, {f!4. e rrglit.rl. ri.hq. '
J to
"

rl rol trvr.t .f rirt*.,rl {o r,6ort
& Fa,rf.r rr bll:{ . '
"
{ : vn- 6L
tr)(0 tal xq)r.irr lt .i$ .l . ...t.1, .1.r..
thi lhq of Ti rr t,
.rrt
N(
) m 0 101

,( )2 o 05


0P(16-). _ `t
` o.1r (e)
(a`
a). F

81
1 6 1

=117.
= i `
8 (119

R )t .

` l11 +ll

9' 2 )
`,


ll Ll,1 1 '
tw
:

,t
t re , 5) e
1, 3
le l l
r
' F.., 6C
P(Y )= P 8 61t.:4 . 1 (t
``)
0 ttl
03 1
Cbl 3(" 5) 20.0 11

i 1
5 = 0 11

s`1
5)


el(1)
`


t o S

1 ir 1
7(2,
)= 7
,
= .1
'(
P( K)=o.o3

r)
r k

]{3, )
1


1 '

TADIPINES JUNIOR COLLEGE


Prdi nary Exomin3dO 20o7
Mathemadcs HigLer 2
9740/01

l2 Scptcmbcr 2l[7 8:00-11:00 hr

Answcr Alt qrcstions,

L:
s,1
uSing my c s bracte hm 0re se{ S,

Find the number oFdi t vecharacter codestt bo fOmed t3l


b)HOW many ofthese 5v COdes begin and end widl ? tzt

The tunction y=(q-r)x, +(lr-2)x+l is obtained by mnsfOnlthg the ttdbn


y - px' +qx+ r through the following transformations:

Step l: Trarslation of -l unit parallel to the *-axis;


Step 2: Scaling of factor 2 porallet to the.y.o<is.

i) Showap+ 29 -3r = -2. t3l


ii) Hence, Iind the exact values ofp, g and r by forming three equations involving p, q
and r.
t3l

A scquence is detined by
ua+t=ua -3, uo = 102.
i) Find u1, u2, u3 tll,d uq.
tl1
ii) Obtain a conjecture for u, in rhe torm-IiI2-I@)lfotr n= t,2,3,4,... o<pressing/(z)
in terms of n
t2l
iii) Pmve the result of q" by l\rlathematical Induction.
t4l

4. i) Express the following equatioru in matrix form and sotve the equations
7x-tY+z =19
2'x-5Y+z =b
r + oy -22= 19


whena=3and6=-3. t2I

ii) For a = ard & = 0, strory that thc cquatiors have infinitely many solutions. Give
12
with reason, a geomerical interpraation : [4]

iii)' If the value of 6 is ctnnged from 0 to 7, comment on thc ncw geometrical


interPreationoftheequation-anddresotutionof$issetofequations' l2'l

5. a) Solve the inequality lt-z4rlr*4 graphically. 13

b) Using analyticat mesrod, solve ore inequality ;^'*


Hence solve2e''| , c-"2
? [51
-l

t
i) Using the substitution .x = lsin d , show that

-4,2 = _Sh 4 +C

6]

ii) R is ttre region enclosed by the curve y="'[Ji ' the 'x-axis' the linas;r = 0

*d , = . Find, using the result in part (i), the glgg! area of the region R' [3


21
7. D Find f xsin.r &.
ii) Show tlrat !il' sin' x A =f,. [3]

D The Shadcd region bounded by the curve y= -2 nx,the x aX and ule ihc

X=: tated mrou 360 abo he UJng yOW d mm pa o and



on fomed,glving your answerin tcrms of
Ci)ind ule value Ofthe solid ofrevolu
14]
8. i) Using differentiatioq find the coordinatcs of any tuming point that may be present for

2 dtty ttl
mtE
15]

li)On Separate di sk
=

"=1
0
2=

showing the axial interOep statiOnary point and equadons ofasymptotes 16]

9. a) The figure below shows a rectangte which is inscribed within a serni+ircle with
radius r = 5cm. The top comenr of the rectangle are to be in conaact with the
circumference of the semi-circle at att tirnes. La.?: c.rn and y cm be the length of the
base and the height of the rectangle respeclively. Find the maximum area of the
rectangle that can be formcd.
tsl

b)A curve has Paranneric equat10ns I=sh and =cOs

Fhdtt mpOlttPwhere =
12]
Henoe,flnd the equtiOn ortt nonnalto the curve at point .:
[2

ne mrnd b ule cttrvc P cuts he mrve agah 0


:
"
Find the coOrdinates ofpoint 0 "ittOwhe

21


:

10.A nmction is giVen by : (4+2)2, = 0 16


1)State dle condilion tt an invese mction to ex and deine dle invesc bnofr
in similar Fom. 131

1) 0nd = :O ShOWitt dearly
]

D pandll

h as dtt po up b dhg et h,2.state
:)
m Fan"Of J,for
Use tt fttt m t s oftL ex m ttpttmation"

c ordinate ofdle point ofh n between dte ptth ofy= Cr)and =3-x.16]

LT
u

She Uses I I matches to complae a second row of4 squares'

She uses 15 matchs to complete a third row of6 squares'

: F m
additional matches she needs to colnp ]


111
b)Ihe pOSitive ntmbes
quation

= + Ll,2,3,

Asn-+@,xn-+L.
i) Showrhat L2 - L-'t = O atl find lhe exact value of I' [3

iD Prove that r2,,-x,=L{L-l). 12


TAMPINESJUNIOR COLLECE
Prdimin3ry Ex3minatiom 2007
lathem tics Higher 2
9740/02

l8 Scptcmber 2(Xl7 8:00-11:lXl hr

Answcr All qucstions

Scction A: Prrc Mrthcmrtic [{0 merksl

. I . Given that tnro quantities r and / bolh vary with time ,. E)eress tho. following statemans
as differential cqr.tations involvingr,y atd t.

i) Th rate of change of y with respec to I is inversely propo(ional to .r. tI l


ii) The rate ofchonge of.r with respct to , is proportional to y. tI I
Hence form a differentiat equation involving x and y only, and prove that the solution
is y2=in CX"where C and n are constants
131

2r =
2,p
ve mto+ +2x +,2) =2
1:

0bttn m" On dttng ,



Find ulc Maclaudn's scries fOrtt up tO and induding dle tcan in
[7

3. a) Express t+iJJ in the form ofr(cosd+isind). Use Dc Moiyre's Theorem or


otherwise, show that

(r *iJI)'*(r -iJI)' = -z'


Hence, find rhe roots of ;i +r[(t + rJt)' * (r -rQ'] = o in the form or rer. [z]

b) Indicate on an Argand diagram, with respct to the origin


e the point p representing
the complex number r+ wtrcre Ocargw<L.
Also, sketch the locqs of P if on the same diagram. If lzl= lx,l, fird
lz-ul=lz+nl
possible values ot arg(z -w) in terms of arg (w) . t61

Shtt ABC h Hs garden The
4 As ofT
, lm nd FC.The cOOrdinate axes Ox and Oy _
are horizontal,and Q Vertical.hc ordinab of ande arc A(20,0,B( 2,9,1)
and Co,4,3), uni h me .

[21
zl) Find the length of the side AC.

14]
7il) Find the scslar product ABoAC, and the angle BAC'
HcncC flnd thc
i.i#ml, ttut 2i + 3j - 8k is petpordicular to Sre lines AB and AC' 14
Cartesian equation of the plane ABC'

itti C
'L

1

L)


_
t

1)
`th
)

i t
)

.(` [ ) t t)
%10 1
.



:
1_ 2
7




Scctioo B: Strabdcs [60 marksl

U A hctory produces chocdlate which arc pecked into boxes of 20 and dcliverEd to shops
for salc. A chocolate will not mcct thc minimum csiEis for pscking for sale if it ttighs
less than 20 grams. On avcragc, 2 % of tl* chocolalc prodmed did not mcct thc
minimum critcria

y Fitrd the pmbability that a randomly chccn box contrin u lcast I clrocolate that does
not meet the minimum eiteria. t2l

.)if
Fillrd thc pmbability ttrat out of 4 rardomly chosen boxes of chocolate, thcre arc
exrctly 2 boxes with at least t chocolate that docs not mect thc minimum criteria. [2]

K A nndom variable X has a Poisson distribution with P(X = 2) = 0.2.

)tl Find the mean 7r, where p > 3, and state the standard deviation q, giving your answers
to 3 significant figures. t31

ji| Using the values found in part (i), find P(4 o <X<2o2). t31

-f g A company prcduces a new drink called 'Whatsoever" and is interested to know the
proponion of people who like their. drink. . -The company decides o interview a
sample of shoppers. Comment briefly on the suitability of each of the following
samples

,i/ A sample ofshoppers who have just bought rlrc drink tu


ii)r A sample of shoppers consisting of 5 pcople agcd I l-2Q 5 people aged 2l -30, 5
people aged 3l- 40 and so oq up to 5 pcople agcd 7l - 80. tU

jiif A sample consisting of every l0r shoppr out of 500 shoppen.

@ *" random variable X has mean p and variance 9. A randorn sample of sizc n (n >
50) is uken from the population and thc sample mcan is denoted by X .

Find the least value ofn such that P(17 -,4 < o.s1> o.es. t4l




8 The rde of fuel con mption of a brand of car callcd Missan is known to have a mean of
rr krn pcr Iitsc. A ncw device has bccn intoduced in the manufacturing process to
improvc thc car's fucl consumptiot rate. A sample of 100'improved'cars selccted gave
the following datr: -

-17)=180 ond 2@-tzf =25e



und ulat de

T:Ld and
N Xtt tand eXP iOn`at 50/ sign Rcance level'in dle oontext
: [ll

9
6 AMg


e ib n"ebJL


A piayer chooses 4 distinct digits randomly from the digits{1,2,3,4,5,6,7)
Thee digi then manod ageinstttt digits On ,balL drawn

'
shw m ttpmb J play h ng y3matchingdJ .

Given that the player has atlost i matching digi nd the pobabnity Of him ha ng
exacdy 3 matching digi [31

rmheeven A and B are independent.It ven that

P(A)=6.25 and P(A


^B')=0.15.
jI ShowthatP(B)=0.S00 t31

Thc errcnts B ari C 8re muEally e4clqive and wlrencver event C occurs, event A
also occurs. i '. C-"
Find B)State tt largest possi e charu that event C wi occur. t3I


:,
T?Jc aool ?"tm tk (trz ttnttr P"f* :)

The number of people applying for the course in Biomedical Science in a particular
university in each of six sernesters is given below.

Semester (.r) 1 2 3 4 5 6
No ofapp:icants( 29 68 138 215 5

The admissions office believes that the number of applicants (y), and the semester (.r)
I
are related by the equation ,y = ,413f , iryh(r I
and are conslrnts.

,,if
.v Using a suitable trarsformation invotving Y = lqg y, givc a sketch of the scafier
diagrarn. Explain wtrctlrcr Ore scaflcr diagram provides evidence that the relation
is a reasonable modcl. 13]

ind ttc"uatiOn of ule ttmated l of regr6sim of r on x,and tte least


squatcs estimate ofl. 121

y'( e*6rin whether the correlation coefficient supports tlre reasonability of the
' model. 121

The'admission office realised that they had accidentally left out the data for a'tpecial
semestef when the course was also open to applications.

66O* inserting the data for 'special sanester", it is found that the equation of
- regession line of I/ on.r found in part (ii) remains the same. The equation of

\$ lar,'r.ntl\
regression line of x on I' is given by x = 2.@Y - 0.297 . Both the line of
9.r

\vr0- lt\ \\t{


regression of .x on I ard the line of regression of I on : pass through the same




:''fl\a point (i,7). Show that 7 = 1.98, corrcct !o 3 significant figures. Hence, find the
number of applicants in the 'tpecial semester", correct to the nearest whole
number. tsl

[l}q**tr''

( i. ', f.7
t

A piza @mpany receives an average of t.2 phone orders for pi"z^ delivery per minute.
-k
J Calculate the probability of receiving.more tlran 8 phone orders in a five minute
interval. I3l

jlzThe
- compary's wo*ing hours is from 9am to 9pnr. Using a gula&lg.-epprorilEalis0.
find the probability tlut thre are 8t least 20 fivo minute intervals in a day that have
more than I plpnc orders. t4l

The pizza. company sells two different types of pizza with the ctroice of additional
opping The urcight of the pizas and additional lopping per pia
ue lgrmdly
di$ributed with ttre following means and stardard dcviations:


Man(mms)

Standard deviation
P 20
P:"A
B 475 30
Additional PPing 50 6
per p:77

the probability that lhe lotal weight of 3 randomly chosen Pi:za B, all with
-iiilFind
'
- differs from four times the weight ofa randomly
additional topping chosen Pizza_ A
by at least 5Og t61

TPJC Solutions for H2 nthematics 9740/01 Prelininarv Examination 2 7

md
g O CscL C" = =30
Case 2:Con nS and one triple =]=20
Qse Does tt contah = =
Total no ofdifFerent codes=30+2Cl+10=

O N oFcod =]+1=4

' Q2. y= FE2 +qx+r


y= (X+1)2+7(x+1)+
= 2+2,+l)+9( +:)+r

2+(2 +9>+(
= +9+ )
2+(2
=2[ +9),+( +9+r)]
=2pF2+(4P+27), (2 +2,+2r)
By companng cients Ofx,
4P+ =3 -2


2P=9-
nstant
2P+29+2r=l

ing :
1 =8

:9==, =
=

TPIC 2007 H2 Papc i Rellm- 1




Q3, i) ul=",u2= u3=93, =90
1) u:=102-3(1)
2=102-3c2)
u=102-30)
u4=102-3o)
. .un=102-3 where =3

iiO I.t P' be the st8tr.leot rh = t0Z-3n


When n = I, LHS = 99' RI{S = 99
.'. Po is hrc'
Assnrc h, is truo i.c. rt: l(}2 - 3k
whenn=k+1,
u.*r=q-3=102-3k-3
= 102-3(k+ l)
Pr is tnr + Pl.r is true
Herrce bY irductiorL % is Eue'



is reduced to
TheaugmentedmatrlXII:: 12


by rrefusing G.C
2 95
X Z=

5 38
y
OZ=
Let z=29
x= -2 , y= +5
Since the plancs intersect in a linc and all planes have different normal,
equatiOns rcprcscnt sheafofPlanes,

TPJC 2m? t{2l,lr6t P.pcl I Ptdim Errm


When b becomes 7,



Augmented matrix is rduccd to

The plures have diffcrrnt nonnal and thOy fonn a triangtllar pttm. nce
equCiors have no solution.

Q5 o OraphiCal Medlod:

X T2
>4

(bXi)
1 2

2 -1
-> -2


,0
>0

(2x-lxx-2x-3 )'0
O o 1 2
2

(il)

e-'<0 or
*.'"-'"
8ut e-' >o ,.., h1<h;,.loz
2
hl <r < ln2
2
-0.693<r<0.69

IPJC 2007 H2 ds Pap :Prelilll Exaln


Q6.

Ld =:Sin


=
d 2


=
:Sin,(1- :Sina


= Jn"CCOS
=
:Sin,
cos'
`


=l 4Jn2 cos'

sh22
=
1
= d

=
L(: )+C
:Sin 4


=
n +c

A ofR mR
I

=i l

=i Sin

= )

=
4)
=

TPJC 2007 H2 Mldls P!p.r I Prelirh Exrrn 4


Q7.0 ,sin, =IIoosX)
COS X

= COSX+Sm,+C


f =f
=[:,
=1 ]

Volumc generzted = rl y' a


= rl(:-sin:)'a
,
=
"Jof
1rt -a.rsin.x + 4sin, *) d

= - 4r[-xcos.r + s,", li . u,FJ


" [fl :

fffi-*n,. o'={-4o*o'

Q8 i) =
(x - 3)'
Whm =0,
( -3x6 -1( -2o,2_10 )=0
6 2_10x_1&+30-6 2+20x=0
8 =30 =4
Whcn,= ,y=

Using sign'tcst,

X 10

Hencc( ,
am hum pht

TPJC 2007 234


`PBF I Prelim Lm

(ii)Ca) 15 25


y=3


=is
3 5
)



0 ( , )
4

=3

(C)

y=N
: 5 10-

y=

I?rC ZD7 H2 ir{tlt P.P!r I P,olin Exrn 6


Q9 (0
r'+f =57
t =25-r2
y=JG17 o>o)
,tnz,rt=ZtO, -ZrJiiT
=2(:(25-P) (20) :J25-,2

= 2(25- :+2J25-P
2)

Using GC:

35355( >o

"==3535S
=25oo2

Al madvc:

Using C.C., sketch the graph o f A= zxJ;;l


A

25

35355

(b) X=Sin , =cOs

=2cos4 Jn
Sh
= =
2cos2 2
Equatbn ofnKDan P

COS =-2( sin )

=-2x
T,C2007H2 Paper i Pral.
Nomal cuts ule curve
s = oin2
=-4sin cos
)s (l+4sin )=0

o =O or sin = : _
=, =Sh
( :)

SinC 0, =Sin 1(
: :)

=2sin cos 2(

:)(:4 )= 8

y=cos =

Henct O=(
)

ne ttCuon
Q10 (i) An invcrsc ttnction exists ifand only ifdle mction is a one

l*t y=(4+2x)2
4+2x =:(
2_4)
Y2 =

f -':r-+L(r' -4), 2
-'. '2' 6

y=,

TPrC 2d, Hi! Mrrhr PaF I Ptclirn Er.m



+ + o
=l x2+_
4 32
ExPansim vJ forl 1 l l
2

f(x)=3-x =,2(1+1:)=3-x
2
x=
3

Qll.0 k Y rd
6nal matches neded m etePc W=19
(li) = =4
= +( -1) =7+( -lx
=3+4
(ili) 7+ +15+19+. +(3+4 )
,"=: 7+c3+4
= 5+ )
5+ ) l
2+5
0

= =
4
"65
-2365 21.15
Since">o, =21
- H ,numberofro shc is abic to makc with 101Xl matches is 21

0) x+:=V7+x ll
As n , L
: L
L= 7+L
L2=7+L
L2_L_7=0
L2_L-7=0
L=1
+28=l
2 2
5rtt xn S POJdVQ L=l+

IPJC 20117 H2-h"I Prelim Exam


(li) X:J=7+x
x:+1-Xn=7 (1)
L2_L=7 (2)
(l) (2):
xl+1-x ll (L2_L)=0
x:. Xn= L l)

TPrC 2([7 fD M.rtu PrF I Prclim Exlm 10



JC Soluticlls for E2 M matics 974002 Prel nimw Examination 2CX17

Ql.(0 '=:
(iO ,=
_ =

=

Jih
=
2=2
IJ 2h
=:ncr2 32)
=in or, where C= , =

Q2. J, =(l + ran-r)z


Di fferentiating wrt r,

fr=4r*t^",(#)
Q+x\L=26a1*'1
Differa iating wrt.r,

(1+
2)
+2 =2(T T)
`
2)2 +2
0+ + 2) =2 eroycd)
DifFcrendattng wrt

+ 2 +4+ 2x2 +24+ 2) +4+3


=
+=2)2 + 1+,2)` +2o+3 2) =0

m =Q =L =2
=z =
By ttallrin's cxI IOn.

=(1+tan-1)2

=1+0)X+ ( + (-4D+

=l+2 + 2_:,3+.

TPlc aItT fX2 !.ldr Pg 2 Pnftu Exrrr I


Q4.(1) AC = OC - OA = 4i + 3k -Qt+zk) = -2i + 4j + k
L,engdr ofside AC = hq = {Gi6+ t = Jti
(1) AB = OB - OA = -2i+ k- Ql + 2k) = -4i -i
(-4\ L2\
AB.AC:l ol.l al=s-r=,
[-'i [,J
l*aZBAC=0.
cos g
. : AB"AC 7 7
=
--------::._:-
ffi =
ffiFzt
+ 0 = 6830

Vcctor equation of plane ABC is

Cartesian equation ofplane ABC is


2x+3y-8z=-12

(i OG=i+1, OH=i ttj+ k

H lics on plarle ABC.Thcn


2(1)+3(1)-80 )=-12
8 =17
=
8

TPIC 2007"Malhs Paper 2 Pre:un




GH=


(clr 2.125m)

Let X bc tt nulnber ofchOmiates out 6f20 that wci sl than 20_


X B(20,0 )

P =1-P(XO)=0.3324

Let Y bc the nulnber Ofboxcs out Of4 with aticast i chOcolatc that weiglls iess ulan
20 gras
Y B ,0_332)
P =2)=0295

PoC
P = =0.2
2=02


2!
FFOm C.C,

Since p> 3, p= 3.31


Hence, o=J]JT= I.82

(ii) P(3.31-.m< x <2(3.31)


=PQ.a9<X<6.62)
=
p(2s X s6)
= P(4 <6)-P(X st)
= 0.791

TP,C2007H2Mah Papa 2 PIclh




Q7.
(D Not suiuble, llre shoppcrs miglrt not havc consumed the &inls.
(il) Suitable. The random samplc coruiss ofshoppers from different age groups.
(iiD Suitablc and pactical.

0 Assumptions: n is largc, n > 50 or X follows a normal distribution


o
ByCLT, x-NG:)
n
P(7C <o.s) > o95
=) P(-0.5<X1<0.5)>0.95

= x-Y.z< o'f t,o.rt


--3 o'f , r.na
>
= n 138.3
[,cast value ofn = 139
=
Q8. (i) Let p be the meam rarc of irel consrmption.
Flo: F=m
flr: p>m
Test Statistics:

Un&r Ho by CLT, 7- N(m,


$) +n.*i."t"tv.
Using Z-test with n = 100,

i = !!9*
100
17 = r8.8

,'= - 1!o'1=22.a3asa35
$rzsso

Hcne-z--L-N(o,l)
' .l22.18/,tr'.8'll0,J
To reject IIo at 5olo lcvel of sigrificmcc,

188-" >1.645
224848485 100
+ m<18.0

of the following witl


(ii) Any-Probability get th mark.
of 5% that we would be wrong ifwe reject H6'
")'
;i ii"UuOifiti of wrongty concluding rate o-f firel consumption has not improved is
5%.
c) 5o/o chane of wrongly concluding that the rate of fuel consumption has not
improved.
question means thcre is a 0.o5
d) .sz" iigpinca"." lcvel. in thegocontcxt of this(the
p;;ili-Itry;i"-ngly rejecting when it is tnr
has been cf;fective)'
device

TPJC 2oo7 tt2 M.6s P.p..2 Prdin B(!!r


_

Q9. 0 0 PrObabnitythathe exacuy 3 m ing digi

=no.of waysOf havttexacuy3matchingdigi


total no of waysfor player tomakchissdcction

_3c3x4cl
7c.

4
35

0P(playcr has cxacay 3 mtting digi" has at i 1l mngdigi0


andaticasti matchn3dttio
=PCplayerhascxacuy3matchingdigi
POaSatleasti mtchingdigio

=PCPittrhaSexacuy3matchingdJ )
1 PCttnOmatchingdJo


1-


%
4
34

o o Evcn A and B are indcpcndent P B)=P( PO)


G en P(A' B')=015
1-P =0.15
1-P(A) P(B)+P(A B)=0.15
1-0.25-P(B)+025P(B)=0.15
075P 3)=0.6
PO)=0.81Xl

(D Sincc A and B areindepcndent P(An =P( PC)=0.25 xO.800


=0.2

Thc Venn diarm musti 5k like dlis

Largest posJble valuc ofP(C)


=P( P(A B)=0.25-02=0.0500

TPrC 2@7 tD Mdr P?..2 Hi! Er&


Q10 (D = (D8
=lo3/+ 10g
log
nc scattcr dm is plo,d With y=log agttnSt

From GC, thc points on the scahir diagrrn lie closc to a sraight line ,
so thc relatkn is a resonable modcl.


(1) FomGC,


a=.
b=.
ra=
r=.

of Y on x is:
the line of regression
Y =Q.781+o.127.r (3 sig. fig)

Since log A = 0.781,


A=6.039 =6.04

(lli) From GC,


since r = 0.995 which is very close o +1,
it supports the reasonability of the model.

(i
(;, F ).
The tiries must pass'ttuough
11sncc / = 0.?81 +0.3277 ---- (l) ]
and i. =2-ool -0.297 -- (2) )
Solving (l) md (2) simultaneously, wc have
7=1.97653 .1.98
= lr =lste53x7 = 1J.81574
. .log y = 13.E35?4 - fiog I I + log29 + lo968 + log I 38 + log2l 5 + log550l = 2'2'l 894
!. =lll,m = 190'08 a 190

TPrC 2q,7 H2 Mdts,+.r 2 Prllim EIlm




QH.O Lt X be dle numberofphone orders for p:" dcl cry in a l minute inttrv .

X P 1.2)
In a ve minute intcrval =5(1.2)=6

Let Y be thc numbcr ofphonc ordes for p177,delivery in a 5 minute intrval.


Y Poc6)

P(Y>8)=1.P 8)=1 IPO)+P(1)+PC2D.. ... 8)]=0.153

(lo From9alltto 9pm thm are 144 flve minuteinmals.

Lct C be the nutnber ofive minutc hmals OutOf144 dlat have morethan 8 phone
ordcrs.

Then C B(144,0.15276)

Sincc n=144 iarge,np=144(0.1527o=22032>5,nq=144o.84724)=122>5,


npq=18637
C N(22032,18637)apprO mady
P(C 20)=P(C 19.5)=Q721
Lct B and T be the mass ofPi
Pi"n B with additional toppings.

A- N(400,201 ,B -N( 475. 30, ), T -N (50, 62)

Let W be thc total weighr of 3 randomly chosen piz-,a B with toppings.

W=Bl+82+83+Tt+T2+T3
W- N(3 x 475 +3 x 50, 3 x302+ 3 x 52)
w-N(1575,2808)

w- 4A - N (r57s - 4 (400),2808 + 4?0))


-N (45, e208)
P(lw - 4Al> so)= p (w
-4A> s0) + p (w- 4 A <-50)
=0.2172+03n23
= 0.614

TPrC 2fln Kl Mldl! P+.r 2 P.clim Erra 7


_

NNANG ttOR
COLLEGE
2007 PRELIM NARY EXAMINAT10N

MttTHEMATICS 9740/01
Higher 2 ,12th September 21107
Paper 1 3 Hours

Addtiond Mttds:Allswer Papcr


LittofFomulac or 15)

READ THESE INSTRUCTIONS FIRST

Writc your namg class and nanrc of subject tutors in the space provided, as well as on all the work you
hand in.
Writc in dark bluc or black pen on both sides ofthe paper.
You may use a soft pcncil for any diagranrs or grapls.
Do not use staples, paper clips, highlighters, glue or conection fluid.

Answer rll the qucstions.


Give non<xact numerical answors correct to 3 signifioant figures, or I decimal place in the case of
angles in degrees, untess a diffcrentlevel of accuracy is specified in the question.
You arc cxpected to use a graphic calculator.
Unsupported answers from a graphic calculator are allowed unless a qucstion specifically states
othcrwise.
Where unsupporlcd arswers ftom a gaphic calculator arc not allowed in a question, you are requiretl
to prcsent the mathematical steps using mathematical not*ions and not calculator mmmands.
You are reminded ofdte necd for clear presentation in your answers.

The number of marks is given in brackes [ ] ar tltc end ofeach question or part question.
At fic end of the examinatio4 fasten all your work securely tosether.

NAME:
C : SUBJECT TIEOR:
For Examiner's Usc Ouly

1 5 9

2 6 10

3 7

4 8 TOTAL

This paper consists of4 printd pages.


:h8endingpoweroL up b and hdudngttc C
Expmd P Statc e rangc
ofvalu of for which ule sion is valid.

Hnd ovducofarthe temmdependentof hthecxpattsbn of i 161


_

By using an algcb=ic rncthod,flnd ule sOlulon sct ofthc incquality



:}
141

Hm , vethc hqud

f-2.
131

Evduate


2d"=:( 2_1)



shOw ulat (h
i

that +2 =Q
4 ry=
s ,p
Hencc or othenwisc,obtaln the Maciaurin's CXpansion of in tenns of up to and including the

inrt 131
term .

Using = ,ShOW that in 2=T i31


(1+96)

WithOut ulc
Thc paramctric cqu ons of a curvc arc x=in inO), =31n( sO),0
0
:

usc of graphic calculator,flnd the equation of the tangent to the curve at ulc pOint wherc O=:

Will this tangent mcet tIrc curve again? Justi$ your answer'

6 Sketchthecu*" /=l6k; for0<rcz. :

Thc area/cnciosed by the curvc = C


axis and,C linCy=l is rotatcd
,
comp:ctcly about ttc axis.Calculate
pl
O ule v01ume gcnerated
141
00 ule cxact arca .

Pagc 2 9740/Ol
NYJC 2 7 Preliminary Examination
civenthat .(, *",.n.2'
"=(,-i)('+)(,-*) #f)
(D Write down the values of U, for n= 1,2,3, 4 and 5. 121

I
(ii) By considering U"-
r for n=1,2,3,4,5, or othcrwisc, form a conjecture for U. in



terrrs of n.




(iiD Usc mathematicat induction o confirm your conjecore in prt (li).

= ,Shw dlat o D=L


VCn
121

HCn ,rlnd13(1 11- 141


)

(')
Dedu me cxprcssbn br
of I * h"r. I.R f(n)

in tlre form n and is a function

oF 141

A particular equation is defined as follows:

f-
y= Ar+B+ := - uc ral
=:: wtwe A, B,C, D
. (Dx+E)'
and E constants.

Detcrmine the number of tuming point(s) fie grdph has and state the condition for a

maximum point to exist. ift


When
22" = ,, and D = 2, the curve has a stationary point at.r = | and rhe
,/ = *,

asymptotes intersect at the point wtrercy=.]. Find tlrc values of8 and E Hcnce state
'4
the equations of alt the asymptotds). 161

Sketch the curve. 131

NYJC 2007 Preliminary Examination Page 3 9740/01


10 V that ,pOintO,4,o lieS CIn ttplane and r2'SuCh that 4: -32 and
lil

Z2:LI:=24.Givcn dlat rl and 2 inCetin dle lille 4 rlnd a VectOr equatiOn Oftt iine r,


fom r= + b. 141

plme contains line r md PNes gh dle point widl poshion vector 41+3i+2k.

Find dle vectorcquation ofdle pl r3and express l h dle fonn + + = 141
Deducc,or prove odtews ,dlat ttc sys ofequ lons
6r-5y-12
"
=-32
5x- y+32 =24
9x-2y+52=40
has an infmitc number ofsolutions. 121

The plane r. is parallcl to n, and has equ dlion gx-2y+52=50' Comment on the geometrical
representation ofth 3 planes 4, r, and rr. I2l


11 0 Sketch tt bilz
1,argc -2
= md arg(2-2
On a


single Argand diagram ISI

(li) t and B"prescnt ttc Pom of htcrscction bctween the bcus lZ J l with

of ag(z-2 -2o= and arg(Z-2J5-2) spect d


Lct O be the o giL E rcprescn the m eX numbcr 2 +2,and F represcn mc


mpLx nulnber +f

Lct C and D rep cnt the poin of h on Lpecnttc b lz l and dle


linc OE widl C nearerto the ongln.

Indicate thd poin` ,3,c2E and F deady in your dia_.Find angle/CD and
anglc ED,icavhg your allswcr in radialls i31

Dedu the geomemcal shape ofthO gure/CBa lll


nd ule area ofdle ngu DB1 / pl


END OF PAPER

NYJC 2007 Preliminary Exatnination Page 4 9740/01


NANYANG JIINIOR COLLEGE
2OO7 PRELIMINARY EXAMINATION
MATHEMATICS 974OIO2
Higher 2 146 September 2007

Paper2 3 Hours

Additional Malcriah: Answer Papcr


Graph paper
ListofFom0 0 1,

READ THESE II{STRUCTIOM; FIRI;T

Write lour namg clas and namc of subject trtors in the spacc povidcd, as wcll as on all the work you
hand in
Write in dst blnc or black pcn oa bodr sides ofthc paper.
You may uc a soft parcil for any diagrams or graphs.
Do not use stE les, paper clips, highlightcrs, glue or correction fluid.

Answer all the qucstions.


Cive non+xact numerical answers correct !o 3 significant figures, or I decimal place in the case of
angles in degrecs, unlcss a different level ofaccuraoy is specifred in the questioo.
You are expocled to use a graphic calorlafior.
Unsupportcd stsntrs from a gnphio calurlator are allowcd unlcss a question spocifically states
otherwise.
Whcre uisuppotcd answers from a gra$ic calcularor arc oot allowed in a questiog lou are required
to present the rrathcmatical stcpc usirry ma$ranatical notatiors and not catq.llator commands.
You are rcminded ofthe need for clear prcsentation in your answers.

The number of marb is given in brackeas [ ] at the end ofeach question or part question.
At the end ofthe examinaciorg fasten all four work seourely together.

NAME:

CII\SS: StlBJECT TUTOR:

For Eremlncr's Usc Only


2 6 10

3 7

4 8 TOTAL

Thb papa' corsists of 5 printed pages.


Stclo A:Pure'Lthem 140 rksI


Find the roos of the equation z' = l, having your ansurers in exponcntiat form' I2l

z'=l
with the smallesl positive Ergument' By considering the
I*lr 0, b,lhe root of the equation
sum l+ar+..'+06,showtlntthesumofroosof thc equation z? =l is0' I2l

121
G HenCCIrovem )+ )+ ( )= :

ne m B with POSi
{XQ


lTllhe .


Nc iS e PSSiblC p
l m
=

)=#
3
that 1-

=+:

(0' l)

is
darSOndm J

given thal its tangent at
e ' d

paralicl to the line y=
I8l

Functions f and g are defined as follows:


f: in , >0
g: sln 0

sken tte nmcdOn t iidiCatingclea C intcrcepts


121

111
Slate tt range ofF

vefiry thar fg doos not exist and lind ttre maximum domain of g for fg to exist Dcfine fg in similar

form, and state the range of fg 16l


Fred worts as an isvestmcar bankcr d Brclsys Bank His 6rat salry fdr.hB frc year was $y. As a top
performer in dre bank he is orfetd 2 employment scllxnes.

Scheme l: An aanuat salary incrcrnent cquiyalent to 40l.o of his lim year total salary.
Sclrerne 2: An annual increnrcnt of r 7o.
_


r he

W thathL tod
sche aLr tt years
la
gy' rc f ra amsscli:mc ! mdhc wisk io caltr 386.400 io bis 4i ycar and a total of gl8,4oo fmo z
his 2d to h$ 4t ycsr (irrclusivc) of work, Iind the natue of a / '\ I : 16l


nCe dC rlllinc d Icast nmmber oF yus Fred shou:d wott such dlat Fred Il eanl more in
ll

Scheme 2 as mparod to Scheme l 121

VR 6C

Sec6on B:Stattrics 160 MarksI

__6 DeFective sPOtS are Follnd on iarge pieces of white clodL On aV" e,25 defective spots are Found Pcr 100

m2 c: th iFa picce oFcloth cOntalns One or more such sPots,it has tO be dlscarded

lt giVen ttt pi os oFcioth oFs12e l m2 are Cut Out lt Fequtt to esumate the peFCentage of
_ oF doth of Jze l m'w ch have to be dlscaFded COnsider the follo llg
etlt Sh
100 pi F clott oF Jze l m2 con 25 defcc ve spots,approxlmateiy 25% ::have to be
diSCarded Do yOu agrec or disagrec?ExplalL 121

1)_ State a tOndido under which a POisson dinibutlon wou:d be a suitable probability model and rlnd

epercentageofl rJ2 pleC ordOth witt deFectlve spo pl


[nd the probabi ty tta there
Using a suitable appro ma are tmmty pieces oF l m2 sized pieces ofcloth,
Out ori pieces,which have to be dinrded i31
RS
76 Dwing a woddtt bm 3 do
number of possiue nting a gments r
4Js med at a rolnd table Find the
are nmbed and e hc p seated
"
together. 121

Mr Tan ov o preSOcks,Four pink socL and hee e gFeen SOm Oe lost dle odler onc)
A hs soch arerando streWn h ssock
Every tllo ng.he tcs uplate nd in a FandO pu scts out oritt dm_to t

to rk
MrTOll 3 mChed ttrOfSOL I
ndttPLbl"

O
ng this 121


Kl

pr babl




ttl
such group will brry at least one 'special Discounf item is 0.997 . 121

Thc rardom vadablc S is the tcal number of crstmcrs out of 80 such groups who will buy a 'Spccial
< <
Dscount' item UsingpnEsl Linit Theores" fiDd to amroximrrc vslue for P(200 S 280)' l3l

groups that visited th shop


Using a suitable approximarioq frnd the pobatility lhet 7? out of the 80 such
have at most ? tourists who bought at least one 'speciat Discount' item l4l

obcervadom over a long period of time have sho$rn ll|ai the midday tempemture at a
particular pke during

the month otJunc has a sEan valuc of 23.rc- An ecotoSis ses up an expaimcnt lo colled dita for a

hypodles teslofwhetrthe dim ettgettnghmer Sheselectsmmdom20Juned SoVeFalVeyear


perid and records thc mid{ay temperaure Her rclls (ia "C) arc as follows :

20.1 26.2 233 23.9 30.4 284 17_3 22.7 251 242

154 263 193 240 19.9 30.3 321 267 276 23.1

1ll
Shre the nutl and alarnarirc hypothcises that the ccologis should ltse.

Given e atthd deM n is2 3 C,m out an a propnate tt atthe le/e stgnilCanCe leve:,

qumptbns IIlade.State your nclusion with regard to the mid day temperatu
stadng any 141
(110 By flnding the unbiasod ttmate oFdle pop o Standard dbM doL conduct anothertest at:00/o

significarrc lcvel Suttc your essumptiors and cooclusion cleady' 141

Calculare the sraadard deviation of the sacrple dd 8rd comment on iL 121


I 0 A study comparing ltc amouot of adwrtising linr o TV per wegtc for a prodru md ttre number of Sales per
rreck for ttc samc po&a was oonducred- The rslts orrq cight r*cks are girrcn bdouc

Advertising time 10 12 15 14 17 :6 Z2 20
(minutcs), r
Sales 23 2_3 k 31 3.2 29 50 40
(thousads), /
G) Find Orc ordiaacs of the point.though *titt thc regrwion liney oo.r and that ofx ony kior pass.
Give your msnrer in tenus of t 121

Ci) Givtatlrarhcrgrcsioaliacofyonrisy=0.I97.x+0.IE4,findt Hence liod 0p limar producr


firornart drdrlioo coefficieni r betutcn adrrrlising timc ard salcs pcr ureek I3l
Cro Plot I scatls dirgram ofy agaiust r. tJl
(!v) St lc with r rcGoo, thc cffccr on r if ttc adrtrtising time was in hours im&ad of minutcs I! |
(v) Givc two rcasoc wfty it is reasonable o us 6c regression linc of/ oo J to estimal lte rolue of.r .
when Y=t.{ 121

I I A soft drink dispeser dclivers lernonade into a cup when a coin is inserted of.-
in0o rhe machine- The anount
lemonade delivered is normally disrriburcd with mean 260 nrl ard stantard dwiarion t0 ml. The nominal \
amount of lernonade ln a cup is 250 ml and the capacity of the cup is 275 ml.

What is the probetility ahst the cup overrlows? l2l


On an occasion, live sudr cupa of lemonade were purchased. Find the probabitity that not more than
one cup oontains less dlan 250 rnl. lll
CiD Somc q.EtorDcrs have complained that thc{r is a high proportion ofcups with less than the nominal
amoual of lemoaade. The standard deviatioo of the amount of lemonade delivercd per cup is fixed,
but the mean can bc altered. What would be a suitable value of the mean so lhat not more lhan 5olo
oflhe cup will contain less than 250 ml oflemonade? t4l
John was helping his friends get a cup of lernonade.each. There are a number of friends. Given that
n is large, find the probability that the toht amourt of lemonade disperued excceds 280a
l3l ml.


END OF PAPER

,('i

NYJC 2007 Preliminary Examination Page 5 9740102



2o


(4-

=111+(
2
)+
x
2 4

2!

+: + +
]
RuCOfValu Ofr-4 4 -
+ l)(1+: +II
: )

Thlls nsm tem=: =5


Hcnce =72.

2 2-7 I-3
,+4 2
-3x +o-2(2-7)
0
,+4)
,2_3 +l

+4

( -2x 1)
0
+4
=

-4or kx 2

Lct y= -1.Thus
2 7
3
+4 2
-4or 2

1 -4(RcJcctd) t<G-tsz
4 9.

3( a)

=
0:

=
+

=i

2_2 +10)
=in( 2

2_2x+10)+:tan :( 1)+C
=:n(

Pagc l of7

= (h 2
fi
) '

= ' lill
:(h


21_
F:(h :in{+liti(:)

=

I [

=:
2_D hoWnb

y=ht00S
Sh
(

= tan

4l= sec2

=-2sccFsCC un

=-2scc2 xtan
+2=22=-2scc'x tan +2scc2
tan =OCShOWn).


When r = 0,
,:0,*=0,#:-,, #:0. #:-
The Maclaurin's expansion tt , = -! - f+ --.

Whenr= 1,

in cos(1)=
:( J (1)+




!n2 i(1+::) hOWn)


Pagc 2 of7




5 di cOs0
O sin0


=_3sin 0
cos0


r-1
Equadon ofttcnt
y-3in = ,_ln:)


In2+inl

= X+in2

No
F m hcGC, ctty h On OfttC mo curves Occu 0=:.

d d , .9999991

6
(1)V01ume gerrcratcd

-ar
ftl2 ----:-dr
= (12),_ Jo I + cos .r
I
=
f
l+2cos'I-l
.,

=
f:Sec2

taniI
=

= _ u ,

Page 3 of7

iD eXactarea

=: f

=
=:-1 fSec,

=:-1 +Lmf

[inl

=: +
+
=: fC J+
7
= =: =:
Coniecture
c771_:=:CJ2 :=:;tJD ;
:=::IJ4-:= :=t
uS
:=

1 + l


"

="+`

m mcnt that =br CZ+


"
W n L = =7
us P Is true.

Assulne me brsOme l c Z+,le. =


I
J( 1- )

= [ )

= )

'+41+3
2(1+lXt+2)
+D
=(1+3
2(1+lX +2)

=

ifi

ThuS ,l me.
Sin P truc and 4 PI is true, therofore by mathematical induction, P, is tru
for all"cZ+.
Page 4 of7

m(H2
yTc 2007 0
/( +1)=:
/(

2
2- 2_2r l
l
22

2-2r-1

2
I

-1)1-2
=(

=_
1 2
2

)=
(
=(I)
(/Cr)_/C7 lpl

trc2) /c3)
+ 3) (4)
:

rca+l)
= l) /( +1)
+1)2
=1_(
2 2
:


( =:L

=:



=1_(
2
+1)2
2"+:
:


1-(

:DE:;=: :
=:
)"

=4(:
t J )

=4(:

t Jr)

=40-

)

=6-

2+4"+6
=6-
2jl

Pagc 5 of7
=

=Q/(D+ '=2CD

+E=J
X=:( : E)

111= ,brm hm m ng ttnt C Q

0"I J
Sitt E= , ca1 0mptotc x=:

H
,:=:D
Thefo

Obliquc asttptotc is y=: +l

,dlus thc point o,4,o its On 71 and 2

VeCtOr ofll l_
X

Nomal veCtOr Of =lilXI:il


111= :

Thus Cartcsian equation of r, is 9x-2y + 5z = 4O -

Page 6 of 7

Ttre.systcm oflinea equations rcpresats 0rc inbrscrtioir bctwccn ,r,ri attd nr.
Sincc I is a cornrnon
line
to r,,r, md tr, lhus the system hss m irfnitc numbcr of
solutions.
Since z. is parallel to ar, the planes rr,r, ard, z. will.form an infinite triangular
prism.
ll



,/1/



Z,qCO=Z,atO=a
6
,{CBE is a rhombus.

Atq, or ADBE = z. !.
2-"0. t 23
L?z= J3 _ I unirs,.

Page 7 of 7
Sincc a a of Liangle/ is`,



Page l of5





l+2R )+2R 2)+2R o3)=0



















2= 5; 4,aluS







RcO+ 2)+R 3)=T:

)+COS( )+00S(

= =0







Sin = `;

COS(

2
l

3

1( =I

+: =Jl 1- )

=+:

7+C=

At(0,1) : :+C C=0


=f17
`:
hb4 -2Jl x+

CO,:)hi -2 D D=2
y=. 2F+2

T=0

(1) =(-_@,@)
iO Rg = [0, U is not a subset ofDl = (0, ),th( :refore/g does not exist.

ln` x& = (0, 'l l, max D, = (0, r)

:r) =l9in x)
= e*'ln(sin*), D6 =Dg=0,

J: .r r-+ e*' ln(sin x), Do = (0 )

r=(-,0I
5 (0t be Frcd's sdtt aner years for Schc l.us an AP with common
difFe nce O,

d sdary&r2 ycars= (2 +(2 -lxO.4")

= I.6+08 )

=
(b)Lct Q bc Frcd's sdtt aft 4 for Schcmc 2_Lct n be thC mmon ra O Oftte
GP Thus c"= :

Page 2 of 5
For C4=86400 and C2+G3+G4=2184 ,thtls

/1'=864
G2+G3 132000
1+ )=13201111
71+732=B2000
1' 86400

=
55rt2_36rl-36=0
1-6xl lrl+0 0

1=:,
lecietdS ,>o
Thustt mud mm =2 l Heno 0.2.

oN
" ),dlllsT> 1lQ +0.
Using GC, =12

6 o Disagrcc Onc c oFdodl can have more than i deL


ve spots on a
(iD ThC OCCurrcn s oFthc dcFec happen r"domly The number of dc


ll 2stt dm
Thus Po(1)
Thus P( 1)=1-R =0)=0 221 11 pOiSSOnpd 0.25,0]
Thi221%ofi m2s cbdt wil have deFective spots

Lct r bc dle number ofi m2stt cioth out 6f 100 p os ulat havc tO be d`eded hus
y B(100,0221)Sin =22.1>5,4=779>5,thus
"=100 is iarge,4
r N(22117227)apprOX
prob rcq =P(y=20)
(19_5 20.5) nonlndcdt19.5205,221,V17227)]

00844

7 ( no oFwayF 6!x2'x10=576

3
2 5
( 18

(ii) P(getting a matched prh in 3 socb) -


=l -P(all 3 sodis are of diffcrcnt coloun)


=1-

(iii) PBd sock docs not givc matcly'll 2 not matdreo

Page 3 of5

+Cl p=

8 Lt be tt nulttber oftotint out of 10 who wili buy ati Ot Onc`Special Discount'


nus ,B(lo,D
b req =P( =0' IbinOmdfoO,1 3,7)1

S=4+ Stt ,V Ltt By Cenm Lh 1


S N )4
prob.roq =Pa S

2803=0.341
.
lnomalCdfo 28Q800 3, 600 9)]

Lt y be me numberOf most7 who boun ttlettone`S


unt'itcm Thus r Bc80,0.997D.
"out Of80 dlat

Sincc"=80 is iargc. =79.76>5,"9=0.24 5,80-r Poco.24) pprOXimateiy.

l p b requ=P( =7 = 80-y=3)=0 181


PoiSSOnpdf0 24,3)]

9 @ during the month of June at the particular


plt(x'.Ho:p=2!.9, Ht:P>23.9.
lrqr X & the mirlday tcrnpcraErc &ring the month of Junc at tlrc patticolar placc.
Level of significucc = 0. I 0.
Assume thet X follmrs r normrl distribution.
-239
Under ,Z= N(0,1)

R ect rfo if v uc 0.10


Perfom Z Test OtoOSe Datt =239, =23,L L Ll,
valuc=0."8
Since,valuc Q10,there sumcient e dencc at 100/O signinmce levet to rciect
and wc cOndude ute mean midday tcmperature during dle month of Junc at u
particular plg"l s .

(i11)Assume follc a nol dbHbution ith unown vttancc


-23,
Under ,7=
Rci rpVm 0. .

s 4.53529 11Var stats Ll]


PJorln TTct OoosC D =23,,List:Ll,Pvaluc=0.2
Sincc P vduc>a10,6 b insurldel cviden at 100/ gniflcance icvel to tted
b md We conde dlat he mem mid day tem ture d ing the mondl of Jullc at

the partida att haS nOt changed

Samplc standard devialion=4.42055 11 Var S'n,C Lll


Sin the sampic deViation`dmost"vi the nccumcd valuc of 2.3 for dle
standard dc ation of dtere is cviden For e :ogist to question that the mld day
tcmpcraturC in Junc has be ome more va b:c.

Page 4 of 5
NJtrc 2oo-r rreln Exom (Hz lrklhs &per 2 so{rl

o =
10

(il) =0.197x+0184
=0197 +0_184


=2994
=09287

(iV) because r is a measure of thc ofscaner and this is

(v) lt is reasonable because Jr = 3.4 lies within the range of the given data and

r = 0.9287 * I ,the regression linc y on .r is almost identicsl to .r on /.


Lct bc the amount oficmonade ddivcred N(2 L102)
(, prOb rcqu=P( >275)=00668 nomalCdf(275,"%260,l

00 Pc 25o=0.158655
Lct y bc the nulnbcr of cups out of Flve dlat oontam icss than 250 m:of iclnonade
r B(5,0.158655)
Prob.req'd= =0.819. [bhOmOdf15,0.158655,1)]
(iD Lct be tt v of c nlealL We nCOd ( 0 0.05.Thus
21 005.
(Z )

ShCe Z -1645)=005, 5l; / s -f .oas p >26.45 .Thus a suitabte vatrrc for


=
mean is 2 .45.
(iv) Let lilbc thc total amount oflemonade dispensed for z orps. Thus N(20",lIXl .

- p,(z >zJi)
Whcn z is large, the required probability is approximaely 0.

Pagc 5 of5


MttCL
2

An arittmetic progression C has flrsttcrm and a non zc common difFcrcnce a G en


that the sum oftllc flrst 10 tcnlls in G is" ce the 22 te....ofthe scries and the 6 term
is 37,flnd thc values of and a 3]
A new series 11 is formed by selecting every third term of G. Find the sum of the first 50
terms of 1L
[3]

Express f(r) = in partial fractions.


G+ +2)
Hence find the expansion of f(x), in ascending powers ofx, up to and including the term
in :r, where lxl < l. 16]

lS a

[2

A child was blowing bubbles and noticed that if he blows too hard, the bubble would
burst immediately but if he were to blow gently, the bubble increases in size till its
optimal volume and detaches itself offfte blowing stick.

:

F

d l


cm3, nd hc dmctakcn ra bu
o tach itserorthc bb

slck ]

3]
[VOlume Ofsphcre=:

t_

MJ2007 JC2 Preliminary Examina 740/01 ITurn OVer

rYl i iantc

,,..
4 16) Using the substitution x=secd,showthat


= h(
)
whereO<d<1. [4]
)

rind Jicis'x d,. :)flE 141


-tbY


n that y= e"F
L

r Sh w that
/ +3(::)2=2
:
t41

or otherwise, find the Maclaurin's series for y, up to and including the


Herrce,
term in x2 . trl

3 6 Deduce that, for small x, e* (z-e*)-i * I +! r. [3]

A curve is defined by the parametric equations


= =+
Skctch dle curve for-2 2_ [11

Show that the rcgion t bounded by the axes, the curve and the line I = ar; where a > 0

n"
of
[1]

find
Hence the exact area region S which is bounded by the yaxis, the curve and the

, = !r. Find also the volume of the solid of revolution formed when S is rotated

lirre '2
through 4 right angles about the y'axis, giving your answer correct lo 2 places of
decimals. [8]

MJCI2OOT lA Pceliminary Examinationlg? 40/01 [Turn over


_
0 Prove that = -7:
F

Hence show that,: [4]


Fl: F :

6
% The term of a sequence is given by forr=1,2,3,...
",=-:r-
(2r)'-l
WHtC dOWn thc rst fourtertt ofthe seq n ,and hence state the values of

r =1,2,3 and 4.
:
[2]


lo Make a conjecture for the formula for terms of n, and prove


:
formula by induction.



-1-^\\
nJttmr
-/ JD, Solve the inequality
d uSInga [7]


A food manufacturer invents a new flavour of meatball from several ingredients-
The main ingredients are beef, chicken and fish.

l00g of beefcontributes 20g ofproteins, 30g of fats and 0.5g of cholesterol.


' 100g of chicken contributes 45g of proteins, 30g of fats and 0.2g of cholesteror.
l00g offish contributes 75g ofproteins,20g of fas and 0.2g ofcholesterol.

The key nutritional requirements for each l00g of mixture of meatball are 609 of
proteins,20g of fats and 0.2g ofcholesterol. other ingredients do not contributi to
these three key nutritional requirements.

Find the amount of beef, chicken and fish the food manufacturer should include in
I kg of mixture to meet the nr.rtritional requirements.
[5]

M!C/2O07 lC2 Preliminary Examination/g740lDt


[Turn over

(a)

A sketch of the curve


(.r + a)'z
t,,_- x+b
where a and 6 are constants, is shown in the diagram. The curve has stationary

poins at (1,0) and P.


State the values ofa and b. [2]

Find thc equation ofthc obliquc asymptote [2]

[11

Copy the above sketch, and on the same diagram, draw a sketch of the
curve
,' +y' = 16,
showing clearly the axial intercepts. [2]

Hence show that the equation


2x.' -6x2 - 68x - 63 = 0
.

has exactly two real roots. [31

MIC12007 JC2 Preliminay Examination/9740/01 [Turn over


6

(b) The diagram shows the graph of y=f(x). It has a maximum point(4a, 3a) and
as)mptotes x:2a and y=a.

Sketch the graph of y = f (x) , where f is the gradient function of f, showing the
relevant features ofthe graph. t3I

, -'-. d'r hY rt r,.-' l vS


100 '' '
1r
Express (l +i)' in the form a+Di , where a,b eD . Hence find the roots of the

equation (z+l\'1 =-gi:


fAt
s
(b) On a single Argand diagram, sketch the loci given by

lz-t-il=t,
/ 9- =
One of the mO cOmplex numbes sa6s,Ing bOth o and(li)

[J(

' State the other complex number in similar form.

(c)

Find the roots of the equation (z -Z)t =SZ. Give your answers exa*ly

form re'' .

MIC|2OOT lC2 Preliminary Examioation/9j 4OlOl


1 Arithmetic Progrcssion
Sl =2r22
5(2 +9 )=2( +21 )

8 +3 =0 - (1)
r6=37
+5 =37 - (2)
Soiving(1)and(2),
=-3, =8
For scHcs 4
First tellll=13,conlmon difFcrcn =24
S50=
[2(13)+49(24)]
=30050

f( )= _1)2( +2)
= + +x.2
(
/( -lXX+2)+B(x+2)+C( -1)2

(
_1)2( +2)
C=I
Bv 'cover-uo' rule-'9

Comparing coefficients in x' , A+C =O

Comparing coefficients in x, A+ B-2C =0

=
(
_1)2( +2) -1)+3(x-1)2+9(x+2)

=;(1- )l+:(1- )2+i:(1+ )


:

+
2+x,+.
=:(1+ )+

2+4
+ )+
:(1+2x+3

(1-:+; + ) ( )

=:+:X+: 2+
3+

From (*) ,

Coefficient of tlre term in

MJCV24X17 JC2 Prehm Exam Paper i Suggcstcd Marking Schemc/H2Maths


740yMaths Dept Page l oF13
3 DirerenI Eoua ons

Vdulnc ofbubbL om,/=:

Considcr = X ,


c ttmt whe tt radi
rcm.
3)=4
2x

4
211= _ 3

dr=ldr
:
31=


+LIC
:inl
]

_
3=/e r

r3= _ e
When =0,/=0, =0=, =
3

Wllen/= 3=:,sub h
, (1

r)
e =:
:= (1-e
=0 178 min

= (
3)

Sincc/=: r' 3=:

= :-3
3)=f(



)

H
D( ) :
-3/1=: +C
:in14

-34=: +C
:hF
in14 -3/1= +C'
:
3

4 -3/=/c r,whcrc/= eC

_:Ll=Bc ::,whcrc B=
3

3= Be r

Mtotz@1 tcz Prelim Exam Paper I Suggesed Marking Scheme/H2 Maths (9740yMaths Dept Page 2 of t3
4 Integration bv prrts/Inteeration bv substitution
(a) x =s@e
dx =sec4tan4 d0

When x =.D,
t
J2 =sec9+cosd=--1-
J2
-rt
4

When x : 2,
I
2=*c0=cos4=-
2
- o-t
3

sec tan d

tan d

:an
)]E
4

=
Ll(
[h(2+V5)
'+1)]
= in(1

(b)
lxcos2xdr =:lX(1+COS2 )dr
`)
os2x)

s2x dF
fx
=:xsin 2x sin2 (L
:

=:Fsin 2x+::cOS2x+/


icos2

Mlcl2@'l lC2 Prelim Exam Paper I Suggested Marking Scheme/H2 Maths (9740/Maths Dept Page 3 of13

Alternative method

Jxcos'xdr =t,(#*)*
=,(i,.1,.,1_ t, (+,.i,inz,) a,
=,[1,*f .io z,)-!r, + Icos2, +c
\2 4 )4 I
=|(zr' +zrsinzx +cos2x) + c

3
4 =_2e2,

4
3
=2
4_4

] 2)

=8y3


4
(3


n =Qy=L = :,
d2


2 4
7

Thc Macla n's series is

y
: :

MJCV21X17 JC2 Prelim Exam Paper i Suggded Marking Schcme/H2Maths(9740yMaths Dept Page4ofll
L
(2-e2F) :(-2e2 )%_ :

:
:c2 (2-e2 )
: :

c2`(2-c2
) : 1+:X

l' :

e2
(2-c2x) :=e2ry 3

t2xt : )3

_ :
+:

1+:X CShOn

Area JR=f'y =f+32 = fttdr(ShOwn)


Point of interscction:

+=:
5+ 3
2=

r5+ 2=0
By obscrvation, =1
=1,y=:

MJC72CX17 JC2 Prclim Exam Paper l suggested Marking Scheme/H2Maths(9740yMaths Dcpt


Page 5 o :3
Area ofS=ly area oftriangL

= 1111= 3 2 dr
:(:)(1)

=31 ,dr :

=311- dr_:


=3[ tan : 1-:

=3
tan 1-0)
:
= : units 2

Vdmcofmld S= tt 2 +vOlmc Ofcor

2u
2


= 077-ts 3
AlterElatiVe Solution:

Volume ofsolid S=
(;-1:
( +: (1)2(:)=0 77 units3

Method Difference to evaluate resulUUse of Methematical


Induction to prove the

r! (r + l)!
+l-1

(
+1)!

=lr
lll =LHS

=
[ ]

=
1-
1 1
+
3! 4!
+

++

]

MJC/2CX17 JC2 Prelim Exaln Paper i Suggcsted Markitt SchcmC/H2Maths 19740yMaths Dept Page 6 of l3
l l
2( +1)!

>0)
: (Sin
0)0 For a , =:

For =2,
2=
L

For =3, 3=

For =4, 4=


=:,




2

4



5' :


= +

+l


=-+
2k+t (2k+3)(2k+t)
_ k(2k+3)+t
(2k+t[2k+3)
2k'z+3k+l
=@+n!k-,
_ (*+t)(z*+r)
(2k +t)(zk +3)
(t+t)
=:t ': :RHS
l2k +3)

Mlcl2oo? lCZ Prelim Exam Paper I suggested Marking Schemer'tD Maths (g74oyMarhs Dept page 7 of 13
Thus P* is true P.., is true
= .

Since I is tme, and P* is tme


= P**, is true , by Mathematical
Induction, P, is true for all n e Z' .

(x-3)( 2) 0


brttd C X"
AQ>0 Ci)
MJ 2KX17 JC2 Prelim Exaln Paper l Suggested Marklng Schemtt Maths(9740yMaths Dcpt Pagc 8 of13
r
2
H 3
H)
2
AND 3
X -2 o r F) 2 -3 3

.'.-3<x<-2 2<x<3
Altemative solution:



x2_8-l l+2
0
1xl-2
2_l
l_6 0 Ml
1xl-2

(l l-3)(l l+2) Ml
0
lxl-2


I MI

:.1*1.-2 or 2<l.rl<: Al
But lxl > 0 for all x, .'. Z < lxl < r
2<r<3.or 2<-:<3 Ml
(Ans.)2<.r<3 or -3<x<-2 M
(b) Lct dlc propo bn ofbcct ch ken and
bc x, and z.

For protcins,
20x+45y+75z=60

For fats,
30x+30 +20z=20

For ch lcstcrol,
05 +0_2 +02z=0_2

MJC/2KX17 JC2 Pre m Exam papcr i Suggestcd Marking Schcnle/H2Madls


740yM Dept Page 9 oF13







Using GC on rhe augmented ."-r- []l ;;




lo, ,-,



nef is given by

Thus,x=0.0659,y=0.132,z=0.703

Thereforc,6.59g ofbeel 13.2g ofchken and 70 3g offlsh


should be included in 100g ofnllx

crefore,65.9g ofbeet 132g ofchicken and 703g offlsh


should bc included in lkg oflruxuc

Graphing Techniques - to include graph of f(-r) given f(x)


(a)0
y=
b=2
=-1
(a)(1) (x - l)': xz +l
-2x 9
' x+2 x+2 =x-4+- x+2
Equation of oblique asymptote I is y = v -4
( )(liD Using GC, coordinates of P = (-5,-12)
(a)(iVb

x2 + y2 =1

(a)(V)

MJGQ 7 JC2 Prc m Exarn Papcr i Su 3csted Marking Scheme/H2Maths o740yMaths ttpt Page 10 oF13
1)2 interscc
Smce thc grap of y= the gtth of
x2+ 2_16at exactly"w points,dlerc are exactly two rcal
roots for thc given cquadOn

(b)

10 Complex Numbers
(a)
2=2
+
(z+3)2_&=(2)22
+
2
(z+3 =(2
(z+3)2=(_2 21)2

z+3=-2+2i Or z+3=2-2i
z=-5+2i Or z=-1-2i

Alterruttive solution:
(z+t)'=-gi
=(2if
=1r+i)u
z+l = t(r +.i)'
=x(-z+zi)
z=-5+2i
(b)

1=1
(1+i =l

arg(2i iz)= arg(-1)(Z-2)=:[

arg( )+arg(Z-2)=

,+arg(Z-2)=
arg(z2)=2

MJ2 7 JC2 Prclim Exalll Papcr:Suttcsted M ng Scheme/H2Maths


740yMaths Dept Pagc!1 F:3

Page 12 of 13
Detr
Schemy'H2 Marhs (974oyMaths
9 anti
*C(r-2\=t
IZ-1-1=1

Ma*ing
=
+

i ) ar1(-i)(z - 4

MJC/2007 JC2 Prelim Exam Papcr I Suggested


2i-i4=L

Half-line obtained by rotating








Alternative method:

*s(z; -a7 :

clockwise.











0
rrarmt"e"

z=2(1+ei:
)

=2 + s + h
)

=2(2 s2 +2Jn
S
)

=4

(
ttiSh

=4 s , =0,L2,3,4
THE END

MJCr2007 JC2 Prcmm Exaln Paper i Sugg ed Marking Schcme/H2Maths(9740yMaths Dcpt Page 15 ofi3
M5 C
2

Section A: Pure Mathematics [40 marksl

The furrction f is defined by


f :xt-; x2 +x-2, x>0.

Find the range of f. [11

Sketch the graphs of y=f( )and y=fi( )on thC Salne diagratn.Label clearly
dle coordinates ofany interscction point(s) [3

The fi:nction g is defined by


g:.rHln(r), 0<.rSe.
(lii) Show that the composite function fg does not exist. [1]

(iV) The function fg exists if the domain of g is restricted to e Find the


least value of ir.
[2]

(V) Find the range offg, where fg is defined on [i(, e].


[11

2 Samuel owes the bank $15000 for his university fees which he intends to pay by monthly
instalments. He intends to pay $300 in the middle of every month. Interesi ii aaaed to thl
amount owed at the end of every month at a fixed rate of o.s%o of the outstanding
amount, after making the payment in the middle of the month.

(D Show that the amount he owes at the end ofthe second month after the interest
has been added is$14545.87 .
tll
(iD show that, at the end of the zs month, after the interest has been added, he will
stiil owe the bank$[60300 - 4s300(t .00s"
)]. Vj
(iiD calculate how many more months he will take to repay his debt, compared to the
case when no interest is imposed.
t3l

MJC/2@7 JC2 Preliminary Examination/97 4OtO2


[Turn Over
3

A cu c dcined pram Jly by x=:,y=h(2 2),whe >0

Rnd ,expresing pur allswcr h ttms of .


[2]

(D The tangent and normal to the curve at the point where .r = 2 meet the r-axis at P
and p rcspectively. Find the distance between P and Q in exact form. I41

(10 Determine the rate of change of ry when 0 =L arrd, increases at a constant rate
J
ofO.l dt per second_ 14]

(i) Find the position vector of P, the foot ofthe perpendicular from the origin 0 to /'
t3l

(iD Find a cartesian equation ofthe plane Z, containing O and I .



[3]

I/, ke!

(iii) It is given that / also lies in a plane with equation , . Show

that k =!2 . [21

(10 Find the angle between the planes I/, and n2, giving your arswer in degrees. [3]

0) A ihi.d ptane I/, has cartesian equation x+2y+z=7 . Determlne the nature of
the intersection ofthe three planes I1,, II, and IIt- t3l

MICf:OOT lC2 Preliminary Examinarion&l 40102 [Turn Over


Section B:Stat 6 160 1111arksI

An unbiased six sided die is dhown until a`6'appears.

Find the p bability that

(1) it Wiil take exactly throws, [1

(li) l Wl take a lcast throws. 12]

Given that no '6' appeared in the first z throws, state the probability that a .6' will appear
6
in the (z + l) throw.
tU

comment on the validity of the following statement made by a gambler: "Since I have
not obtained a '6' in the last l0 thows, rhe next ttrow is likely to be a .6'.,,
t2t

A class committee corsists offive boys, three girls and two teachers. They have to take a
group photograph for the school magazine. Find the number of ways they can be
arranged

in a row without any restrictiors,


[1]

in a row such that there is a boy at each end ofthe row,


[2]

in two rows, with four people in the first row, such that the two teachers stand in
the middle of the first row,
2]


/.
llChthat tt M nchcrs adOgethcr and no m e next tO h
[3]

7 A cereal manufacturer produces thousands of packes of cereal each day. The mass of

cereal in a packet follows a normal distribution with mean 475 g and standard deviation
0.7 e.

Following a slight adjustment to the filling machine, the mass, x g, of the cereal in t0
randomly chosen packets are summarized by

Zx = 47 47.s, lr' = zzs+Zst.os .

(a) Assuming that the variance ofthe distribution is unaltered by the adjustment, test
at the 5%o significance level whether therc is a change in the mean mass of
cereal in a packet. 4]

(b) Assuming that the variance of the distribution may have been altered, find the
smallest level of significance at which the test will indicate a decrease in the mean
mass ofcereal in a packet. t5I
MlCl20O7 lC2 Prcliminary Examination/974OlO2 [Turn Over
5

E In a one-hour interval, the number of customers entering an antique shop has a Poisson
distribution with mean 2. Independently, in a five-hour interval, the number of customers
leaving the antique shop has a Poisson distribution with mean 8.

(D Find the probability that in a one-hour interval, no one will enter and no one will
leave the antique shop. I2l
(ii) Find the probability that in a one-hour interval, at most one customer will enter
the antique shop, given that the total number of customers entering and leaving
the antiqr.re shop is exactly 4. 121

(iii) By rsing a suitable approximatiorL find the smallest integer r swh that the
probability of a total of more than n customers entering and leaving the antique
shop in a five-hour interval is less than 0.3. t3l

(rO The antique shop is open daily. Find the probability that there are at least two
days in a week whereby no one will enter and no one will leave the antique shop
in a particular one-hour interval. l2l

In an investigation ofa shrub, research workers measured the average widthy (in cm) and
stem demity x (number of stems per m2) at ten sites with the following rcsults:

4 5 6 9 14 15 19 21 22
y 08 2 0.65 0.6 0 0.65 0.55 0.35 0.40 0.38

(D Calculate the correlation coefficient for the data. Ql

(ii) Give a sketch ofthe scatter diagram for the dat4 as shorvn on your calculator, and
comment on your answer in (i). t3]

(iiD ldenti$ the two data pairs which should be removed, and calculate the correlation
coefficient, and the rgression line ofy onx, for the revised data- t31

(iv) Use the regression line of y on x in (iii) to demonstrate that it is unwise to


extrapolate beyond the range of the data. - [2]

(v) A new data pair (a,6) is now obtained at a$ eleventh site, where 4 < o s22 .
Using the revised data in (iii) and the new data pair (a,b), the regression
equation ofy on x is calculated and found to be identical to that using only the
revised data. Find a possible d xa pair (a,b). I2l

MIC|Z007 lcz Preliminary ExaminarioolgT40loz [Turn Over


6

1
10 The exhaustion time of an alkaline battery is defined as the time it takes to discharge
completely under controlled experimental conditions. Research has shown that the
exhaustion time of AA-size alkaline batteries manufactured by the market leaders,
Durazer and Energi+ell, have the following population parameters:

Mean (in hours) Standard dwiation (in hours)


Durazer 19.87 0.24
Energi-cell 20.16 0.19

(e) () 50 Durazer and 80 Energi-cell bafteries are tested. Determine the


disribution ofthe sample mean exhaustion time for each brand. l2l
Hence find the probability that the difference in the mean exhaustion times
of the two brands of batteries is less than 0.2 hours. t3I

A new company, Batter@, intends to launch a new line of rechargeable lithium ion
AA-size batteries with per-charge life+imes comparable to the AA-size alkaline batteries
manufactured by Durazer and Energi-cell.

(b) (D The normal random variable Wis defined as the exhaustion time (in hours)
of a new Batter@ battery. Given that P(I/ <18.5)=P(W > 20.3) = O.tO2,
flnd the mean and standard deviation of 1
t3l
(ii) 100 independent experimental readings of W are taken. Using a suitable
approximation, find the probability that ar least 25 of these readings are
more than 20 hours. t5I

(C) Market surveys are carried out to determine the popularity of the new Batter@
batteries with users. The suwey is conducted by an independent company which
hires l0 surveyors, each assigned to interview 50 people. Each surveyor is to
interview 30 males and 20 females, in three different age ranges as follows: below
25,26to 35, and above 36.

(i) State the sampling method used. .tU


(ii) Give one advantage and one dlsadvantage of this sampling method. ll'l

I/.lCt2007 fCZ Preliminary Examination/97 qlLz


l

2007 2 THS 740 JC 2PRELIMINARY EXAM PAPER 2)


SUGGESTED SOLUT10NS

SECT10N A:PURE DIAT EMATICS

1 ' i ilt
] Functions
Rf=[-2, )

y=fl( )

( , )

intersection e.i"t (J7, '6 ) or (t .+ t, t .+ t) .


{, = (-"o, tl e Q = [0, co) . Therefore, fg does nol exist.
(iV) We need t=[Qll
In =0 =l

Thcreforc,a=[1,el, =l

(V) Rrs = [-2,0]



WC|2OOT lC2 Preliminary Examinarion/974olPaper 2 Solutions




Amount owcd atthe end Ofsecond month
=[(15000-3CXl)(1 005)-3CXl] 005)

=S14545.87

month Amt at the beginning Amt atthe end


1' 15000 (l5ooo-3oo)l.oo5
2nd 15000(1.005)- 300(1.0c r sooo(r .oosf - 3oo(l.oo5)'
-300(r.oo5)
3rKl
lsooo(1.00s)'2 - 300(1.0 r 5000(r .0os)r - 3oo(l .oos)i
- 300(r.005) -3oo(l.oos)'?
- 300(r.oos)
nul
l s000(r.00s)" - 300(t.005)"
- 3oo(l.oo5)"-l
+... - 300(r .005)

Amt of the end of zth month


= r 5000(r .005)" - 300(1.005)" - 300(l.o0s)*r +... - 300(1.00s)
= I s000(1.005)' - 300[1.00s + 1.005'? +... + r.00s" ]
'oos" - r)'l
= 1s000(1.00s)'- roofr'oos(r
L r.00s-r l
= 1 s000(1.00s)" - 60300(1.005' - l)
=60300-45300(1.005)' (shown)

60300-45300(1.005)" 0

L00
51

i
in1 005
57.3

MlU2007 ICZ Preliminary Examination/9740/Paper 2 Solutions




Assunting no interest no.Ofrrontb required= =50
No.ofcxtra inonths requircd 8 months

3 Parametric

(1)
When.r=2, e=4, :.+=1, y=tn3z
d-r
The equation of tangent is y-1n32:x-2
.'. Wheny = Q, x=2-1n32
Coordinates ofP = (2 - ln 32,0)

The equation of normal is y - ln32 = -(x-2\


Wheny=Q, x = 2+1n32
.'.
Coordinates ofQ = (Z + tn fZ,O)

Iength of PQ = 21n32
2)
=:in(2

=:ho
=
==2x01=0_2

=0.0/.96 (to 3 s.f)

MJC/20/J7 tC2 Preliminary Examinarior/gT40/paper 2 Solutions


4

Vectors


09007 JC2 Pre:iminary Examination/9740/Papcr 2 Solutions


Examinario.y'97,+0/Paper 2 Solutions







M|CIZNT lC2 Preliminary


-8+6 -2 +5+ =-3
-3+(7-21) =-3

7-21=0

(liD
(10

Let bethe angle beneen planes tt and

cosd =
V 17.25
=6.8 '

0 The system of equations is


lOx+l9y+82=0
2r+!r+r=-3
2'
x+2y+z=7

flo 19 8lo\
Hence the augmented matrix is | 2 + , I -:

Ir z rlt)
I

fr o -3 lo\
UsingGC,ttreRRenislo t, lol
[o o o I'J
Thelastrow
= 0=l impossible!

Therefore, there is no solution. The system is inconsistent.


Herrce the tfuee planes 2,, fI, and ff , do not intersect.

MIC/2O07 lA Preliminary ExaminatioV9T40/Paper 2 Solutions


SECT10N B:STATISTICS

lt J
F i
5 Probability
(D
Reqdrcd prob =(:)H(:)

(iD Required Probability


l(:)+(:)
=(:)
(:)+(:)"+1(:)+
1(:)

_(:)

1-f
6
:

=(:)

Alternatively,
Required Probability=P(no``6''in flrst( -1)thrOvvs)
1

=(:)

Rcquircd Probability
I

The statement is not valid because the probability ofobtaining a O in


the ll throw remJns at tt and is independcnt ofthe rcsul in thc
6
flrst 10 throws

MlCl2007 lcz Prcliminary Examination/974olpaper 2 Solutions






6 Permutation & Combination
No. ofways = l0!= 3628800

________
587654321
No. of ways : 5x8!x4= 806200

T: T2

No. ofways = 2 k 8!= 80640

(iV)

No of ways=6! 2!x7x6 5=302400



MI MI Al

MJC12KX17 JC2 Prcliminary Examination/9740/Papcr 2 Solutions


i= 4747'9 =q74.t9.o:0.7
z:10.'10

Ho:. y=
475
p
Hr: *475

Test Statistic: Z =I+


/J;
Level of signific ance: 5%o
Critical Region: Reject H6 if p-value < 0.05
Assuming He is trtre, from the GC,p-value = 0.343

Since p-value :
0.343 > 0.05, we do not reject Ha and conclude that
there is no significant evidence, at 5oZ level, that there has been a
change in the mean ofthe distribution.

s2= 22-(1ll;2)2]=029433
1221li;

HO: =475
Hl: 475

r=
Since =10 is snlan and o2is uttnOwn,

ddd k
RcJtt Ho r
vJ
i::
Assuming Ho is tnle,from the GC,
valuc=0_12 03

For mttivc hypothesis to be acceptd,ic.H rei d,

Pvalue
10

=>0126003 11
100
The snlallest level ofslgniflcance 12.7%

MIC/2@7 lC2. heliminary Examination/9740/paper 2 Solutions


1

10

8 Binomial and Poisson Distribution


h

bcthe no ofcustomers entcnng an antique shop in lh X Poc2)


t y bethc no_of customers leaving an antique shop in lh
(1.6)
P(nO One wi enter and no one icave h antttue shop)
=Pcr ).P(r cl)
=e-2 e 16
.0273(3.s.o

Alternative Solution:

:1:: il
3
=PK3 6)
+y
P(m One will ellter and m one will avc the an que shop)
=P(rtt )
=00273(3s0

(1) Req prob


=P( 11 + =4)

0_0074612+0.037306
01912223392
00447672
0.1912223392
=0_234(3s/)
(liD G I/b" th" n- .f entering and leaving an antique shop in
5h_/ L(18)
"*to*ers
Since =18>10,/ (18,18)apprOX

P(/> +05) 03
>1972(from GC)

=,lcast is 20

(1 Gt I/be th" no. of days whereby no one will enter and leave the
antique shop in a one-hour interval out of7 days. W -B(7,0'0273)
P(w >2)
=1- P(w <t)
= I - 0.985716
=0_0143(3s/)

MJC/2007 JC2 Pre:iminary Exarnination/9740/Paper 2 So:utions


Correlation Coefficient and Linear
=-0.542

'iftie scatter diagram indicates a strong negative linear


correlation
between x and y. This is not consistent with the value of r in (i) due to

The data pairs whlch shOuld be rcmoved are(5,2)and(H,0).

For the revised data, r=-0.916


regression line ofy on x: y :0.83992
:0.840
- 0.021267x
- 0.0213-r (3 s.f.

Substitute a large value of; (-r > 39) and obtain a negative value ofy,
which is impossible.

Method 2
0.83992 - 0.021267 x < 0 x > 39.494
> =
For -x 40, y < 0, which is impossible.

Any pair (a,6) that lies on the regression line ofy on x, with working
shown clearly where 4 < a <22 .

a b
14 0_542
15 0521
16 0_500
17 0478
18 0_457
19 0436
20 0_415
21 0393
22 0.372

MlCl2OO7 lcz Prcliminary Examination/97,10/paper 2 Solutions





12

10 Normal Distribution and Samolins


(aXID t and/be the exhaustion time(in hOurs) fa Durarrand an
Energi eli battery respectively.

Sin :=50 is large,by the Central Limit Theorem,


Nl19.",
)
P

SinCe 2=80 is iarge,by thc Central Limit Thoorem,

7 N
ttlapp

(aXli) - P) = e{7 )- sF I : rg.tt - zo.rc -- -o.zs
Var( -7)=Var( )+Var(7)=121+ =0.0 160325
80
N( .29,0 160325)

r(17 -71 . o.z) = t(-t.z .T -I .o.z)


=0_012297 (using GC)
=0.0123 (to 3 s
(b)(1)

N ,
2)

P( 185)=P( >203)=0102
symmetrlcJ about 18 5+203=194
Hencc, =194 -

Stt
d
Z ) m2 Z


) m2 P

49
FromGC, P(Z <-1.27(Il4)=0.102 = =-l-2702+
o

M!C|2N7 lCZ keliminary Examinatiory'9T'l0/Paper 2 Solutions


13

9
=
-1.27024
=0_70853
0.709 (to 3 s.1)

(bXl) bt/be the nulnber ofttdings out of l thatare more h 20 hours.


/ B(1 , )
where =P(7>20)=019855

Since =iCXl large, =llXp(O19855)=19.855>5 and

=1 (0 80145)=80.145>5,

/ N(19.855,15913)apprOX

P(/ 25)=P(/>245)uJng COndndty corrcc on


=012210
0.122 (to 3 s l)

(c)(D Quota sampling.


(0(ii) Advantaee: (any one)
. Easy to administer
. Fast in gathering data

Disadvantaee: (any one)


. Quota might not be a good representation of the population as a
whole
. Sample obtained is non-random, thus it may be impossible to
estimate the sampling error

MlCl20O7 rC2 Preliminary Examinatiory'9 yPaper 2 Solutions




CATHOLIC JUN10R COLLEGE
PRELIM!NARY EXAM:NAT!ONS 2007

MATHEMATiCS 9740 01
Higher 2
Paper l lo september 2007
3 hours
Add ona:matedals: A eF Paper
mph Paper
of Fomuiae(MF15)

READ THESE INSTRUCNONS FIRST


Question Marks
1
Write your name and HT group on all the work you hand in.
Write in dark blue or black pen on both sides of the paper.
You may use a sofl pencil br any diagrams or graphs. 2
Do not use staples, paper dips, highlighters, glue or coreclion
fluid. 3

An$,\rer all the queslions. 4


Give non-exact numerical ansrers conect to 3 significant
llgures, or 1 decimal place in lie case of angles in degrees,
unless a different level of mrracy is specified in the question. 5
You are expected to use a graphic calculator.
Unsupported answers from a grzrphic calculator are allowed 6
unless a question specifically shles otherwise.
Where unsupported answers frorn a graphic calculator are not 7
allowed in a question, you are required to presenl the
malhemati(=l steps using malhemalical notations and not
calculator commands. 8
You are reminded of the need for clear presentation in your
answers.. 9
The number of marks is given h brackets I I at the end of eacfl
question or part question. 10
Al the end of the examinatirn, fasden all your work securely
together.

At the end of the paper, detach the cover page and atlach
it to tfie answer scdpt witfi the cover page on top. 12

Name: Tota:
HT:
Calculator Model:

This document consisls of 5 printed pages-

[Turn over
l A cubic crve passes hrough J POints(2,4)and 1, _Find the equation oft curvc ifit has a
stationtt pOint at(1, 1) 141 -

2.o
nd tt VJuesof
7 =1,2,3,4.
r P]
(m)Make a mdabri h
t ra ntls of
ll _
(lii)PKIVC your cOttecme using mathcmaticalinductbn [31

3. A seqwrre {Jr, [Jz, Ut,... is defined AV U,=


In
(D show that IJn- u*r = =2'*l . [1]
n2 (n + l\2

O HCnCe flnd
[3]

(lio Dedu that the sum to the frst telllls ofthe sedes

9 15 21
1222+2232+3242+
is less than 3. [2]

4. (a) Write down the difference between a permutation and a combination. [11 _

(b) A team of 14 sodents goes on an excursion.


(i) In how many ways can tlry travel ifthere are two vehicles, a van that can sit l0 and a car that can
sit 5? l1l

(ii) At an Indian restaurant where they stop for lunch, there are only two round tables available, a table

for 6 and a table for t. Of the 14 students, 3 ofthem are good friends. Find the number of ways in
which the 3 good friends are seated together at the same table.

131

[Turn over
5 Find tte cxact sOlution ofthe fb o ing inequalities:

(1)il [3]
;>

(li) 3-. l>0 [41

6_

0 pand h aSndhg pwers Jx o md mh [21


( )Statc the range Ofvalues of for ich ttc above exPansion is valid

(ili)Deduce the cquation oftangentto tt curve


2
l+
y='Vl-4x
at the point where r = 0. [31

7.Lety= ln(l + sinx).

(i) ' -
dxz=t-",1(q\'*tl
-'-.-",d,'!


Showthat 'j


L(a,/
(ii) Find the Maclaurin's series for /, up to and including the term in :2.

(iii) Verifi the correctness ofthe series found in (ii) by using the standard series expansion for sim and
ln(1+ )
2]

8. The firnctions f, g and h are defined as follows:


f:x <o
-+ x2 - x,x
g:x-->e'-2,xe9A
i::-+lnln(.r),r>l
(&) State the largest possible value ofa for which f-r is defined.

-r
Hence find f in similar form.

(b) (D Find the range ofg.


(ii) State why the composite furrction hg does not exis.

(iii) By restricting the domain of g to(a,o1, where aeR, find rhe smallest value of a in

exact form srrch that the composite function hg exists. Define hg.
3
9( The frst four posit c tenns l,72, ,V4,Ofa geomcdc progrssion are such that
V4 V2=15 and v3=4 l

Find hc vJue of [31


( A man deposited tt atthc mrtOfevery ycarin a mtt Thc bank gave anintcrd of4%per

O ShOW thatttthe end of ycar,the IIlan n have s2 1.04"-1>. [21

(li)HOW much(to the nearcst donarD mustthe man depodtin tt bank every yearifhe to havc

$ l0 000 at the end of I 0 yean.


ll

(c) The numbers r, satis$ the relation x,., =- for =1,2,3,

ASZ-+co,ro-+t
(D Find the exact value of/<.
3

(ii) Show that ifx" <,L then xn < x.,+r


3

By uSing


41


(b)Find
2+2x+3 di



2=


2)%
(C)O PrOve m
2'and ind _


(iD Hence ind 2
vl_

ll (a) Express in terms ofx andy for yE = q65y [3

[Turn ovcr

4

(b) A cylinder of height d and radius r is irscribed in a fixed cone of height i and base radius a;

the two figures having a common axis of symmetry. Show that the expression for the height of

the cylinder is r ={r-:) Prove that the greatest volume ofthe cylinder inscribed in the cone

.A
is lofthe volume of the cone. t6l
9

[Proving ttnt th volume of tlre cylinder is a maximum is not required]

[Volume ofacone is given by


I , bur" * x height]

(*=-?)'
12. The curve C has equation f(t)=, (r-JXr+J)^. -

(i) Express f(:) in the fo* P+ Or+ 4- t3l


x-3 x+3
(ii) State the equations ofall the asymptotes ofC and the coordinates of the points where C cuts the axes.

t3l

(ili)SkCtCh On separate diagrams,


( y=/( ),


1 0 =1 ,

(Oy= /(2 1)
making clcar the rnain relevant fcatures ofeach curvc

End of Paper




k(k+2)+t




H2 maths Marking Scheme for Prelim _
-(k+lxk+2)




Paper I


k2+2k+l


(k+ lxk + 2) =


y=ax3 +bx| +cx+d


k+ I
=ffi




t' dY
= nHs
= 3ax2 +2bx+c
dx [B21
n
.. Pr true
= Pr+t true also,
Pr*r true, by
tMrl Since Pr is truc, and Pl true
01) We COnJecture that
1
=
Math Induction, Pn is true for all n e Z*
r(r+1)
r=l
(8)+ (4)+c(2)+ =4 n [fno proper statements, deduct I mark
(-1)+b(1)+C(-1)+ =7 n+1 [BI



from whole questionl
[All
(1) b(l)+`(1)+ =-1
Statem = 3(D
(3)(1)+b(2)(1)+C=0
1 I I (n+l)2 -n2 = 2n+l
u' - f, = - -i

n
@ @. tf- fu +T

=

'*r 7

1 0 0


n+1

tBll


0 1 0

When n=l:LHS=ul=:


Jl

..


2n+t

0 0 1

1t@=


Qn

RHS=:=LHS Pl is truc [BI]





0 0 0

c ie

Assume that Pkis truc for a


LU, -U *, =Ur-Uz +U, -U, + "'+U N-t -U N +U N -U N

Thus a = l, b = l, c = -5,d=2
Jl lMll
Prove that Pk+l is also truc i c
I


_r _,ry+,

Equation ofcurve: y= x3+x2- 5x+2 (/{+l),

lAll tA2l



LHS = [BI]
(k+1)(k+2)

2(1)



=
k
+ 1)(k+2)



00 + + Total number of ways = 29937600 +
3991 6800 = 69854400

lAlI
+3[
]
(u)(i)
2lxl -3,7
lxl
={ + + t +
2lxl-sEl:r]
lxl
tMll ,' -' 2 '>0
lxl-l;
"
=r[,- t-..l . r
(1/+l)'l
...
.,o
| ---l-..-
(1/+l)'? - ---ili1-[-"
4lxl- - lxl(xl- .
6 r)
^
[Ml,Shape OFcurve and
Ml,intersections with axes&asto s
lBll
>0
4 (a) A permutation considers the order in i 1lF -1 x 1,-3 -2,2 x 3
which a number of objects are chosen
[MI] [Al]
whereas a combination does not consider
-
lxl'? 5lxl + e
the order in which objects are chosen. #<0
2(lxi- t) [Ml] (li)
lBll
(bxi) dxl-:trlxl-zt [Ml]
Case I: l0 students in the first car and 4 in
[Al] x-3--2->0
2(lxl - t) -1
the second = laCl 2_4x+1>0
Case II: 9 students in the first car and 5 in [Ml]
lxlct, z< lxl<:
the second = IaCs X-1
total number of ways in which 8 students
raCr -l<;< 1, -3 < x <-2, 2< x <3 ( T(2+ 6)) T(2 V5))>0
can travel = * raCs = 3003 -1
lMll + [Al]
(bXiD
Case-l: 3 friends at table for 6: I IC3 x 3l x
Alternatively,
from GC
3! xtCs
x 7l=2993?600 IMll
Case_ll: 3 friends at table for 8: llCs x 5! i
3! x 6c6 x 5! = 39916800 tMtl

I
: 1

1MI]

J+:
m
2+
[Al]
+
(iiD y = sh _On
2
=1+
2- 5 1 2+ l 3 9
[Ml]

[Ml]
Vhen x=0, =:
=:(

) +.

[Ml
[Ml]
EquatiOn oftangent is given by i (verifled)
x_
z-Ji.x<1, x >z+J1 =:'+1 0r3y=4x 3 [Al]

7(1)` =1+Sin x
IAlternatively, 8(a)a= 1/2 [BI]
lf use GC and graph drawn, [B I ] for
=cos


[MI]
shape of graphl
Let y=x2_ =( [MI]
:)2_:
I

6,(i) - er;-; = Sin [Ml]



1t +;r'?;11 `
( )2+` ::
' =1- [Bl]
:
:

=1- ( )2+1
SQ
: :
[Ml] [BI]

=0+ 2) +:x+
2+.
)
7hen x=Qy=Q =L =J lAll
ll/2 mark deducted for not expressing fl
in similar forml
1+3 x+ 2
[All
[MI]
9 (b) (i) range of g = (2, o)
By Maclaurin's series, y= 2
lBll
:
0
: (ii) Since domain of
[Al] h = (1, co) , rangeof
[BI] g c domain of h. [Bl]
(iii) For hg to exist, range of g = (1, o). Sincer=2,ut=2.5
=2-V4-4(5)(-1),as k o [Ml]
lBll [BI]
e'-2=l+e'=3=x=ln3
tMll = [Al]
(b)(1) At the end of n'h year 5
Least a = ln3

lx+ .+104
hg(x) = 1r1"x - r, = ln(ln(e'- 2)) =S[104 +1.04 t- J6
'-J-
]

',
5 ;-2x"-1>0
[BI
hg : x -+ ln(ln(e' - 2)),x > ln 3
[Bl]
lAll


= $[ L04x(l + I .04 +... + I .04'-r) ] 5xl>2x,+l [Bll


Note: hg(x) = In(ln(e*-2)), x> ln 3 is
acceptable
L [Bl]

9(a) Since A=2r and =4


= 04xlttH [Ml]

ul ,1 +1
'

,'.r2 =4 * =2
= $ 26(l .04" - l)x
10(al X=2 sin0
[Bl] (ii) '$ 2611.0+'o- l)x= $10 000
From uo =15+u, x= $800 [Al] ar=2 cos

r'u, = 15 1-1r, IMl

(c)()As n ,x" and


+
2sin +1 2cos
V4-4sh2
3u, =15 ) ur=J [MI
= [BI]
[BI]
2cos

512_2 -1=0
12sh +D
:


=-2cos + +C
[All
= yG = cosy
JElny = 1n1s6r; [ ]


[Mll
=-2 Sin 1(:)+C
0
+(f) =m l

= V4-
2 +sln l(:)+13


2)%=:
:ll
, % 2xl=

II
7

ltan
= :)

[Al
dy ylny
0
(11)


dx ,lzxlytany-J?:) IarJ


(b)

=l h-d
ra =L
) tBrl
2)(vl_
= (_
2)
a =n(t-L\
[Mll \ a)
Volume of Cylinder,
=_ 2v x2_ vl_X2(_2x) [Ml]
2_ti)
4= 2 = [

(

7_: 7 Volume ofCone,/2=: 2

[MI

= 1) [ ]
hP+2x+1-
( )+C 7_ "
(2

-1:i)=0 [ l]
[A2 (2
2
= [ l]
3


I:ll ;

=::
:4
]
,q

/. - o\2 g0- lgx


12. (i) Using long division, we have /(x) = =t +GiX;,
Gffi
624
-'-(r-3)-G+, [A3]

(ii) Vertical asymptotes ar x = 3 and


x : -3. Horizontal asymptotes: y = I

The curve intersects x - axis at ( 9,0) and y-axis at (0, -e1. [A31

(iil)

(a)

J
=

[2 marks for correct shape; I mark for indicating minimum(9,0)and maximum point(1,-8)
clearly]

(li)

I mark for conect shape, I mark for correct intercepts with axes, I mark for correct maximum
point and asymptotesl

(iii)

i2

Qx-\\'z ('-)'
v=-sgvs-1=
' Qx-4)8x+4 = (x-}(r+D
n mark for correct asymptotes; I mark for correct shape and I mark for correct stationary
poinsl



CATHOLIC JUN10R COLLEGE
PREL!MINARY EXAM!NAT10NS 2007

MATHEMATiCS 9740 02
Higher 2
Paper 2 29 AUGuS 2007
3 hours
Addttonai matttals:Answer Paper
Graph Paper
ust of Foonulae F15)

READ THESE INSTRUCTIONS FTRST


Question Marks
1
Write your name and HT group on all the work you hand in.
Write in dark blue or black pen on both sides of the paper.
You may use a sofr pencil for any diagrams or graphs. 2
Do not use staples, paper clips, highlighters, glue or corectbn
fluid. 3
Anslrer all the questions.
4
Give non-enct numerir=l answers correct to 3 significant
figures, or 1 decimal place in the case ofangles in degrees,
unless a different level of accuracy is specified in the question. 5
You are expected to use a graphic clculator.
Unsupported answers from a graphic calculator are allowed 6
unless a question specifically states otherwise-
Where unsupported answers from a graphic calculator are not
allowed in a question, you are required lo present the
7
mathematical steps using mathematical notations and not
calculator commands. 8
You are reminded of the need for cbar presenlation in youa
answefs. 9
The number of marks is given in bractets I I at the end of edt
question or part question.
At the end of the examination, faslen all your work securely
10
together.
Totai
At the end of the paper, detach the coyer page and attach
it to the answer script with lte coyer page on top.

Name: Calculator Model:


HT:

This document crnsists of 6 p{inted pages.

[Turn over
Section A: Pure Mathematics [40 marksl

The variables.r and z are rclated by

,2" 4as" =ra rn


&.

By means of tlre substitution y= xe' , obtz'in a diffelential equation relatingy and x. 121

Hence show that the general rclution of(f) is

= r"(z' * t) , r is an arbitrarv constant' 121


'
(iii) Sketch the solution curve for t = - l, stating clearly any asymptotes and axial intercpts.
21

2.(a)
(D Find the fourth roots ofthe complex number -l +.f3r , giving your answers exactly in the form
re'dwherr>0and-r <0 3n. t31

(ii) Deduce the solutions of the equationz8 +2za +4 = 0, giving youranswers exactly inthe form
re'Pwherer>0and-n<0 3 n. t31

(b) w is a complex number satisfling the condition lw +2+2i l<2. Shade on an Argand
diagram the locus of r+. Fird the maximum and tlre minimum values of lw - 6 - 4il.
151

3. The equation ofthe plane n; isgivenby-r+2y The position vectors ofthe points,4 and D
-22:lE.
+ +
are given by - 2i i and 2i 5j - 3k respectively. The foot of the perpendicular from ,,4 to the plane
n1 is B. C is the point on 8,{ prodtrced such that Bl : lC = 2:1. Find

(i) the position vector ofB. t3I


(ii) the perpendicular distance from,4 to the plane n1 . tr1
(iii) the position vector of the point C. t2l
(i") the equation ofthe plane DBC in the form r.n = p . t31

(v) the length of projection ofthe line 8D on tr 131

[Turn over
4(
(D In the diagram on the right, the shaded region ,{ is
bounded by the circle with equation tz + y2 =19,

the curvey = a, .r-axis and the line.x = 2.


x
Find the area ofthe shaded region,{,
giving your answer conect to 3 significant figures.
t4l

In the diagram on the right, the shadd region X is


bounded by the same circle, the line r = -JiI- and
the liney = .,fi[. Find the volume of the solid
formed when R is rotated through 360'about
the line x = -JD , giving your answer correct to
3 significant figures t3l

(i)The graph of y=2 ,for O l,is


shown in the diagram Rectangles,each Ofwidthl,arc

+[ ,show that thc tOtal area/
drawn undcr thc curve.Given thatl+ + 2+x3+ + "=l F

all n rectangles is given by I

t3

(iD State the limit of ,{ as n H co .


[2]
Section B: Statistics [60 marksl

5. A soccer tean of30 players has l8 Singaporeans and 12 foreigners. Ofthe Singaporears, l0 are
midfielders and 8 are defenders. Ofthe foreigners, 5 are goalkeepers and 7 are strikers. Forn players are
chosen at random-

Find tlre probability that all four players chosen are Singaporeans. r2l
,,(i|
Show drat tfre probability ofexactly one striker being chosen is 0.452. Iz',l
1iD
,(r'li) Ifexactly one striker is choserq find the probability thai tfuee Singaporeans are chosen. t21
**rO the probability that there is exactly one goalkeeper or exactly one defender (or both). t2l
@t,nd

fi, At a partic\lar petrol statiorq the number of car tanks topped up with diesel is recorded.
State a condition under which a Poisson distribution would be a suitable probability model.

The average number of car tanks topped up with diesel in a randomly chosen hour is 3.

,6'r Find the probability that exactly 2 car tanks are topped up with diesel in a randomly chosen
one-hour period. 12

/1o on hour peHods are chown tt random Find dlc p ba that CXactly 2 car tanks a"
topped up with diesclin atlcast 4 of dlc onc hour pcriods. 13]

y() By using a suitable approximation, find the probability that not more than 9 car tanks are topped
'up with diesel in a randomly chosen four-hour period- [4]

.X=' In country zrl, 3 out of l0 people have heights exceeding 1.70 m and l. out of.20 people have
heights below t.50 m. Assuming a normal distribution of heighs, show that the distribution has
>r
. .*iS' ,.0. mean I .65165 m and standard deviation 0.09220 m, correct to 5 decimal places. t41

X -.no--\
' i'.? O50 people from country I are chosen randomly. Using a pilqgjplloxim{q, find the
probability that 6 people have heights exceeding l-5 m. t31

In country B, the heights of the population follow a normal distribution with mean 1.55 m and standard
deviation o' 15 m' [Turn over

4
Find tle probability that the total hcight of 3 randomly chosen people from country ,{ is
trt
more than 3 times the height of a rardomly chosen person from country B.
t41

I
m ttein h

d
.

y7f Tlte popularion of Singaporc (population 4 492 150) are divided into the following races: Chinese
76.7%,Malay 14.00/0,Indian 7.9 1.4%.A sarnple of200 Singaporealls is tO bc chOsen for

a survey


l
wmT
O blayS,50 1ndi

Statc one disadvantage ofdl method

Describe how you would carry out stratified sampling.

State one disadvantage of this method.


[5]

The lifespan,l, in years, ofa randomly chosen mobile phone produced by a certain manufacturer
may be taken to have a normal distribution with mean p and standard deviation 0.5. A random

sample of z such phones are purchased. The probability that the sample mean lexceeds p by at
most 0.1 is more than 0.9. Find the least value of z.
4]

./i e particular machine has been calibrared to produc golf balls of 5 cm diameters.
A random sample of6 golfballs prodrred by that machine were measued and the diameters in.
cm were found to be

5.00, 5.02, 5.05, 5.05, 5.t0, 5.il.

Find unbiased estimates for the population mean and variance.


Perform an appmpriate test at the 2olo level of significanc to check if the machine is not
producing golfballs of the correct size. State one assumption that is used in the test.

[5
,!s\
*
{rrt\'$$On
-r\
NSA A random sample of 50 packets of the snack'Yummi Cnnch' is weighed and the mass.x in grams
' is recorded. The results are summarised as follows:

o-150)= 0,XX-150)2=25CXl.

A tt ws carnCd outatthe 5%slgnincallce level with the following


"hes:
t'-t
\rl0 ,1b1lb / Hr : the population mean mass of lhe snack is < p6
: T ' 4,,;+)
K
Given that Ho is rejected in favour of H1, find the set of possible values of 15. Assume that the

distribution of the mass of the snrck is normal. t61

Iil.
It is believed lhat ttre probabilityp of a randomly chosen pregnant woman giving birth to a Down
Syndrome child is related to the woman's age x, in years, by the relation p = a6' , 25 ( r 3 45, where
a and 6 arc constants. The table gives observed values ofp for 5 differcnt values ofr.
25 30 35 40 45

000067 0.00125 000333 0 0 111Cltl 003330

nd fi"a linear regression lines and their product moment correlation coefficients of lnp on x and
lnp on ln x. I41


Which regression line is better in establishing a model for the data? Give a reason for your

2
n
Plot on the same graph the-dari ana tne regression line ofyour choice in part (ii) [2]

By using your choice of the rcgression line in (ii), estimate the expected number of Down
Syndrome children (to the ne$est child) that will be bom to 50q) randomly chosen pregnant

women of age 53. Explain why this is not a good estimate. [21

^4


,



'

E Papar

6

(b)Circle centre(-2,-2),rad 2
Prellm 07 H2 Maths Peper 2 Marking Vertical asymptotes are x=O and Shading the circle
Sch eme PC= J64+36 10



x=
[Al] Max dist=PA=12
Min dist=PB

8
l = + [MI
2(a)



=4x [Al]
024_1+
=2 [MI]
3o
=(
)b=(
+(

=2
`121
1)

= 4
2`
l =2
2+c =(ti)
Ml
=2 [Mll
l'31):,k=0,_1,1,
'=2x2+c z=2: 2 [A2] 0-2)+2(2 +1)-2(-2o 18 [Ml]
9 =18 =2
=2x+`

" "+:)a_n

(ii)24_
z8+ 2z4+4=0
-2 V4-16
Al]


= l V3,[Ml] 6)1 =2 1+4+4=6
arbitrtt constam 2


[Al]



[Bl




0:
I



Therefore z =2i e


l
l
k=0, 1,1, 2[Al]
J 0:
I

i
and z =2tr e
k=0, 1,1, 2[Al]

[l mark for correct shape]


Curve cuts x-axis at (l/2,0) and (1,0).


4( )USing enher Cc OrnOn CC method,


2and l intersectat
the graphs
10-

(b)(D TOtal area of n rectangles





(1,3)
= +1(2:)+ (2:)+. + (2 )


Bl]


"



=ll+2:+2:+. +2

=31088+2 0794 1MI]



[Mll
x
= 519 Al]

(l)x3(1

(i We rOtate the curve


Therefore equation of plane is 2+( _
)2=10 abom ey
instead.
[Al] Th ,x=V75-lo_
2 V75
Liml ofA= 2X =1.44 or



Bll

(i
0

1
Volume required= [Ml,Al
in 2
,2 = (V
V10-
2)2


0 0

[Mll

5(i) P(all 4 players arc Singaporcans)

= =
=952
[Al] IMl,Al]

proJection ofline BD on the plane=

V22+ IBl,Al]
=norlnalcdf( E99,9.5,
Events occdr uniformly
12, )
(ii) P(exactly I striker is chosen) Events occurring in one time =0.235 lAll
interval are independent Of
='ciiic,
La
=o.tsz [Mr,Ar] those occuring in
any other mutua y exclusive
7)Let X=height ofpeople in
country A
tlme intervall
(iii) P(3 Singaporeans lexactly I striker)
X N( ,o2)
_ P(3 Singaporeans 11 | sti ker)
P(lstri ker) Let X number ofcartanks P(X>17)=03
topped up whh Diesel
'tc, x'c, RZ>
30^
tn in one hour,X P (3) IBll
"
= = 0.46 r
P(X=2)=poiSSOnpdf z 1:=L
0.452
lMl,Arl (3,2)
lAll
17
" li
= 0224 =invNom(0.7)

(iv) P(exactly I goalkeeper or exactly Let Y=no orone_hour
I
(l)
periods with exactly 2 car
defendcr (or both))
tanks topped up with Diesel
- P( I goalkccper) + P( I defendcr) - P(X 15) 005
P(l goalkceper n ldefender)
Y B(10,0224)
z
> 1


:Bll = 5

_tcr^"c,
toco _'cr*"c, _
0671
'oco
5c,x!c,r'' c,
'oc o
P(Y 4)=1-P(Y
=l binomcdf
(10,0224,3)
3)
lMll
Noml

[Ml,Al]
=0.167 IAll (l)-OV hvNom(0 11

hour period,X invNorrn(005)


Either one ofthe following: lBl I
For 4 P (12)

- Evcnts occur randomly in a Since 12> 10, Then c=009220 and =


IBll
continuous given time interval. X N(12,12) 165165
- Evcnts have a low probability approximately lBll
of occunence at any givcn
instant.
P(X 9)=P(X 95)using Comnent: [MlJ marla
tlsrll 'awarded oen Commcnb For ld.ndrying
CC. for ' cor.ci mil
(i) inconect st and ard iza t i on :
d used instead ofo
(ii) incorrect use of invNorm = 0.738 :All (b) X-N(tt0.5'?)
function: did not change the - 0.52
x-N(p,T)
sign to '<' but penalize by not Comment: Accept 0.739
awarding [Al] nark. X and W are independent lBll
variables. P(X-ps0.l)>0.9 Bl]

P(i<0.1 + rr) > 0.9
Let Y - no. ofpeople with
heights excceding 1.5 m Sai) Quota Sampling etz<ffil,o.e [Ml]
Y - B(50,0.9s) tBll
np - 4?.5 > 5, nq=2,5 < 5 Disadvantage: \;
It is not a good representation ofthc
Let Y' - no. ofpeople with population as a whole compared to othcr P(zs0.A6)>0,9
heights below 1.5 m lBr I sampling methods. [Bl] 0.2r[ > InvNorm(0.g) - l.2E155 41]
Y'-B(50,0.05)
np=2.5<5,soY-P"(2.5) lBl I rfi > o.+ozzs
approximstely (iD Draw random samples from the >
n 41.06
different races with a sample size in lhe . .least value ofn - 42, [Al
sanre DroDortion as the size ofeach race.
P(Y=6)=P1Y'=a;
Rnce Samole Slze
= poissonpd(2.5,4)
:Al: Chinese 7670/o 200=1534 153
= 0. 134
Malay 1400/o 200 28

Let W = height ofpeople in Indian 790/0 200 158 16


country B Others 14%x200 28 3
w- N(r.55, 0. rs1
lBt I l^21
E(Xr +X2+X3-3W)= Dlsadvant0ge (Any one ofthe following):
3(r.5sr65)-3(r.ss)= lBr I
o Time consuming
0.30495 . More diflicult to conduct and analyse
Va(X, + 1, + X, - 3YY; = lBl I results than simple random samplinE.
3(0.09220) + 310. 15'?) = ,,
0.2280

P(Xr +X2+X3-3W>0)=
normalcd(0, E99, 0.30495,
Jo.22ro )
There is insu{Iicient evidcncc at the 2olo
r 0994 [Bl,31]
level to say that the machine is not
producing golfballs ofthe correct size.
lnP=-2r.0*r.*,n*,
lBll r=0.980 [Bl,Bll
Regression line ln p on x is
o x +150 14&4 1]
befter because its r is closcr to I,
s2_ (2500- )=48408 or from p-ab*, we get

x 5.055 All
[Al] lnp="**,nO [Al,AI]
s2_0043242=000187 All

H : =5

Bl]
h StatlSt C Z

Hl: 5

Ho eCted
:est
:Data " Fu

p Va
:b
ll:
:5.055
5x:.0432434966.
n:6

KlnvNorcm5 [Bl] inp on x

c:
:est
5
11 : 3 Ew
[MI]
[Al,All

t=3.115426512 1484- -1 61845


P= 0263883098 1tO>150 [Al]
=5.055
5x= 0432434966
n=6

Since g-value = 0.026 > 0.02, we do not 10(i) ln p=-125+0:98x,


reject He. [Al] lnponlnx [All
Using ln p = - 12.5 + 0.198x, when

x=53, p=0.13453 [Ml]

Thcrcfore out of 5000 pregnant


women, there will be an expected

0.13a53 (5000) = 673 children with


down syndrome. [Al]
The estimate is no good because it

is an extrapolated value.

tAlI
OR
Using ln p = - 12.5 + 0.198 ln x,

whenx=53, p = 8.1794x10{
Therefore out of 5000 pregnant
women, there will be an expected

8.1794x10'6 (5000) =0 children

with down syndrome. [M I ]


The estimatc is no good because it
is extrapolation. [Al
ANGLOCH:NESE JUN:OR COLLEGE
MATHEMATiCS DEPARTMENT
JC2 Pre:iminary Exanlinauon 2007

MATHEMATICS 9740
Higher 2
Paperl

A cub curve passcs through ulc pOin (l, 3),12,12)and O,5",and i` inttrcept
-4.Fhd dle cquatbn oftt curve 141

Find thc generat solution ofthc differential equation 0<x<f,


fr="u*^*,wtretc
exprcssingy in terms ofx. t3l

Provc by induction that

=

,


for all posidve integers


State prccisely a sequencc of trarsformations which maps the circle in diagram I onto th
ellipsi in diagam II. State lhe equation of the ellipse. l4l

Diagram
Diagram I i Diagraln II


"T can bc exprcssed as V
Show thatthc wmplcx numttr z=1+

C m ex nmttr tth mdd 5 mdargumm: C COIn numb w

in cxact polar form. 151

Find the set ofvalues ofx for which


lr+
' '>|r-2
I3l

Hencc, solve
lz.r- tl
l-----l > -x t3l
r
I I l+2x
.

- [Turn Over
,laglo.Crit*tc Jot k* C-olcg.
H2 Md.rl.b r1o: 2m7 rc 2 Ptttmhrv Erlhilrlb PrF I
Patc , 0{6


l
, or otherwise, ind the valuc of for which
the subsdtution =

:
i61

The funaion f is defirBd b, f ,r*l#t, xeR, x*-21, tr>O, and the graPh is

shown below. The lirps t = -2tr' and y = | are asymptotes of the curve, and the
r -intercept is given bY .r = 2.
v

y=l

The function f has an inverse itits domain is restricted to a < x <0 . Find, in terms of 2,
(D the minimum value of a, I l

(ii) the definition of f-r. 141

The cuwe C has equation ,, = sin-t (2x+2) .

00 Hndtt I
(l11)It iS g cn that h linc y= +b is tangendal to ulc curvO at = ,WhCrc
,b c R Find tt in tenlls of Hcn indmerhgeOfpossibicvalucsof 14

10 (a) The sum of thc firstz terms s"=6-+


of a series is ^s'' where Prove that the

series is a geomctsic progrcssion, and find its sum to infinity. 14l,


(b) By considering U" + U,., , where

u"=;i'
J

Fhd i(-1 --l- in terms of n. t4l


4r'-l' -

Hence, write down the sum to infinity of the series in which the rs term is
IU

.lrglolfrrct Ja or AA.gc
H2 M{n alii! y/{q 2007 JC 2 Paliminry Erminlrio.l P.F I
Pog. a q6


The diagram below shows a sketch of the graph of ), = (l - x) widr a minimum point I
at (1,2) and a maximum point I at (2,0) .

'A(!.1)

Sketch, on sep&ate clearly labelted diagrams, the graphs of


(i) (l-r),


y: =
(ii) .y = (r),
statin& in each case, the coordinates of the points corresponding to I and g.

12 (a) The parametric equations of a curve are


x=t(r+2), y=2(r+t).
.Find, in terms of p, the equation of the normal to the curve at
elo(t +z).2(n +r)f. r4l
lf this normal meets the r-axis at G and Ar is the foot of the perpendicular from p to
flG.
the x-axis, find ttre length of
I3l
( The equarion of a curvc C is 2+siny=x2-ry.fina!.
&. l3l
Show_that any tangent to C cannot be parallel lo the r-axis.
It is given instcad that c has a tangeni which is parallel to the.paxis. Show that the
l2l
fcoordinarc of the point of contact of the ungent with C must satis$
sin2 y+sin y = ycos y-1. 0l

[Turn Over
lrgto-C'ltt,.r..tunb, A .N
H2 Mrdrcauis 91O 20O7 rC 2 lrdioiarry Errninrixr prer I
Pap. 5 of6

13 The p:anes and 2 haVC equatiO r=1111+


l:+ l l and rl:11=6
reS iVCly,where , R.
( )ShOW that 4 and tt are parallel. 121
( The lnc 4 pSSes tt ugh the point ,with Position vector, L and paral l
3 +7k mec atB.Find
(1)the posilion vemr ofB: :41
(1) lenga10fprOidion Of in the pialle r2 131

14 Exp"ss f )=P_5 . 151


+& 4 h partid'actb
Expand f( )in ascendhg powers oftt up to and including dle tenn in 141
State the sct of yalues ofr for which the cxpansion is valid. 111

ls (D Find Jcos(lnx)dt. 141

,
(ii) Sketch the curve .v = cos(ln r)
for x> e- 1 , stating the coordinates of the tuming
point and of the intersections with the axs. The region R is bounded by the r'axis
and the ofthe curve ,'=cos(ln:) betwcen r=3 and x=5' Find the area of
Part
151

End of Prper

,nab4{n 4.rttbtdt g.
H2 Mdr:ari.! 9a{} Hinirry ErEinrin Pr!" I
AXf, rc 2
Pag`` ar6
AngioChinese Junior Coi1090
H2,` JLa" s'7ra
21X17 JC 2 PRELII S PAPER
" I Solutions

L"t th" iquution of the cubic curve be y=o,+bxt +a'+d '


intercept is 4 =-4

-3= +b+`-4
12=8a+4b+2-4
59=27 +9b+ -4

=3, b=-2, 0=0= =JxJ 2r

2 =SCCX tan


=SCCX+C
=ras + + +D

3 Lct P(")be thC Statcmcnt


5r-4+J5 +l=:(J5
+1-1). cZ

" L = X
Hcncc P(1)iS truc
=
RHS=:(J -1)=LHS

AssuFnC PCtl me,: +1-1)Forsomc l c Z`


5 -4+ 5 +l=:(V5

mttC

=:( ( +1)+1-

LHS ofP( 1)=


HN
=:
1

( +7
= +7 X



) (
Thereforc P(+1)iS me
Sin l)is mcand R mc P( 1 is me for al:acZ
Page l of8

4 I : A scaling parallel toy- uis with scalc factor { units
ll : A scaling parallel to thex- o<is with scale faaor I uniS
III: A ramlation of I unit in direction of the negative.x - axis
OR
I : A scal ing parallel to y- axis with scale factor I units
II : A rarsluion of2 unit in dircction of the negative x - o<is
III: A scaling parallel to the .r - axis with scale factor { units
OR
I : A trarslarion of 2 unit in direction of the negative.r - o<is
ll : A scaling parallcl to dre.r- uis with scale factor + units
lll: A scaling parallel toy- uis with scale factor + units
OR
I : A tsarslation of 2 mit in dircction of the negative x - o<is
ll : A scaling parallel to y - uis with scale factor * units
III: A scaling parallel to thcr- uis with scalc factor { units
OR
I : A scaling parallcl o drex- axis with scalc factor I unis
ll : A translation of I unit in dircction of the negative x - axis
III: A scaling parallel toy- axis with scale factor I units
OR
I : A scaling parallel to the r - axis with scale factor I units
II : A scoling paratlel toy- axis with scale factor I units
lll: A translation of I unit in direction of the negative.r - axis

Equation ofcllipse: (x+ t)' + (2y)' = I



n(:)=:

5 z=1+ :=l+ s(:)

brc, F ,ag(2)
c

IJ
i
, arg(W)=: :)=
(

W=

F`

6 McmOd l:Graphical

with equation(s) stated

State intersection poinls and/or equation of vertical asymptote.

x< 2 or *.,tj p-zu1


Page 2 of8

Qn6 ntinucd)
Medlod 2:Algebraic

x+2> _ (x+2)>
x-2

2 x> Js @236)




>



Re ace,by-1,

-!.2
T

x<--I) 0 >

11

7

'=

=4
1

2_:
6

=:

rT


[Sil 6


[Sin l

=2

= 22

0=1 , S

=
+2
+2 = +
+x= -2
,0+1)= (1-2 )
=
(1-2 )
1+

, =
= Page 3 of 8

9o

liD = =1

1-4 + 2=

4( +1)2=1_

( +1)=

At,= ,
r

7

Forthc valucs of tt to bc Valid,

tO haVe real valucs


lr : fOrI_1
12 4
2 or t -2( ected Sin 1>O for posidve grad nt oftangent
to any point on the curvc)

10 = .

=6-
FT (6- ;)
6

mmmon Q

=
6

=
L=
6 2

Hen , e se estagcome gession

s_=6
Page 4 of 8

ln ( 2 hlhs

10b
+ + =
:

i)
Or
=:


=: [
+ ]

=:[(1+:)
(:+:)+(:+:) (:+:)

(7JLT+7 [T)]

=;[f

1 7]

$,Jr)


I Bl(2,0)

Page 5 of8
4=z(r+r\ *=*
12o) *=,
Equation of normal to the curve at P:
y -zQ+ t)=-(o+ t)(x- p(p+z))

t - 2( p + r) = - (p + t\ x + p (p + t)( p + 2)
y + (p + t) x = (p + \(p2 + z p + z)

AtG, y =O= x= P2 +2P+2


y, y= p(p+Z)= p2 +2p
:. NG=2

12b 2+siny = 12 - vy = *, tfr= z,-(.*. r)


dY 2x- Y
= A= t*t- -
When y =2x, the tangent to C is parallel to thc;r-axis'

...Z+sii2x= x2 -x(Zx) = 5i,r2r= -(Z+r') has no solution,

sin"e -(z+12)s-2 vxeR.


Thus, any taogent to C cannot be paratlel to the,-axis' (shown)

For C to have a tangent parallel to the}raxis' x=-@s! '


.'. 2 + sin y = 6s52 y + y@s y = 2+ sin y =l -sin2 y + ycosy

= sin2y+siny = YcosY- I (shown)

138 NormaiVCCirizl=(:)X
il)=-3(1)

nOrlnal t 2=(
)
Sincc the nomals ofthe two p:anes are paralicl,thc plancs arc paral!C!

b)i)

POS:tiOn vcCtOr of =(
'4,)=(1)

Page 6 of 8
Method i:

Let tte foot ofperpendicular hm/to plane be F

EquatiOn oFlinc/Fr=(1)+/(1)

PointFis ule intcrsectionofline Fand Plane

(17:;)(1)=6 =,/=:
POSitiOn veCtOr ofF= :(1:)

Lengh Ofp"jection of/ onto 2=BF= =7.86




Method 2:

Length ofprojection oflE onto =7.86

14 f( )=x3_5,2+8
-4=
9,2_25x+20 / c
+ +(.-2)2
(
-1)( -2)2 = 11

2_25 +20= ( -2)2+3( _2)( -1)+C( -1)
!
9

Solving,_ =4, 3=5,C=6


f(r)=1+ J+ 6
x- I x- 2 (x- 2),
= a(r- l)-' + s(.t-2)-' +6(x-2)-'?
= -a(t - r)-' - s(z- x)-' + 6(2- x)-'z

= -a(r-,)'' -,[,(,-;)]',. r[,(,-;[


= -4(r+r+.r'. )-J(r.1. "...).-f(,.,.i,....)

x- 3l x' +...
=-5-3342

The range ofvalues of; for which the above expansion is valid for

{xeR.'-1 <r<1}

Page 7 of 8
COS(ln,)di=(00S(1 ,))( ) )((

= S(h )+ Sh(inX)
= COS(ln X)+(Sin(ln ))( ) (


= 00S(1 )+ Sin(bl ) S(

(h ='als(h )+,Sh(h +C
2

COS(h
= (" )+; ( +




lo P10tt '11t2 Pl t3

:A7q
1',(X))(X
rYz=I
rV, =



rVt=
rVs= Zaro
rV6= [=1.810t771 Y=0

turning point: (1' l)



/-r,


I ,o
le )=(o.2os,o)

j S(h )a+ (h J


Rquired arca=

= 03871 00037
= 0391

Pagc 8 of8
ANGLO CH:NESE JUN10R COLLEGE
mATHEMATS DEPARTMENT
JC2 Preliminary FY mina on 2007

MATHEMATiCS 9740
H:gher 2
Paper 2

Sectio A:Pure Mathemadcs 140 marksI

Th dia s be:ow show tt tth of f and its derivttve r.sttL cl


ons,which pphoffand moff` "with
i21

Dhml

A cup is obtained by ong the paraboia 2=4 about thc x axis.If:t bolds
CXacuy 32 cm3,ind dlc dialncter of m ofthe cup, ven mat the radius ofthc
base is i cm pl

A famcr ra cs chickens in a fam Taking into accOunt vanous oondidOns,such as


sale of poul deathS etc_that afFd tt population of chickens in his fam,the
numbcr ofchickcns atthe cid Ofa montt is modclied by 4,whcrc

r"=0875"_:+50.
Express t, 2 and in ten ls Of%.
Show =o.875" -4)+41Xland deduce thc vduc Of r.when bc mes
Yery luge.
l

civen that ,=;r,show drar find Maclaurin,s series


#=r*Hencc
fory, up to anil including the tcrm in ./.
I5l
using the standard series expansion for ln(l+.r) and Macraurin's series for y,
find the scries cxpansion of ln(l+tan.t), in ascending powers ofx up !o and
including the term in y'. pl
Hcnce show lhat the first three non-zrro terms in the cxpansion of ,t*'a,
l+tan2:'
arc l-2x+txz. tJI

[Turn Over
1

2
PcrJ
`
m dtti
:
1".l.ll-2.

O StatCtt minimum Vduc of J. :
modulus and thC argumcnt ofw fOr which
oD Find ulc extt valucs oFJ
ardW+1-

oontairu two substanccs whose masses are ': kg


and
sunnose lhat a clrcrnical mixtrc
;Tf#;ffi;ffiil;;;i;-i k& At anv timc the ratc at which x is
'' el frat time. Obtain a
'l'ffiJ;; is proportionat to il- proauc if t'" tio masses
rtifrercntial equation retating.r andt. IU
il;lh" ilc.*J *f,ft", tf O" differentiat cquation may be expressed in the
f.rnr hfil) = Ir , whcrc C and t 8rc Positive constants'
l4l
\l-rl
andwhen t= l, x=i'expressx in terms
of t3l
Givcnthat initially ,= ''
*(o. IU
Show that r -) I as , -)

Scction B: Statistics [60 merksl


'ls with mean p kg and variance
Thc mass of sugar sold in bagr normally distributcd
have masses which differ from kg
p
d tg'. tt is knovvn lhat 3(P/o oi rtt" uugt oisugar ohosen bag ofsugar has a mas-s
bv at least c kg Find the ptoonUiiity tf,"t a ruidomly Isl
'.i#ffi;";;rr.Glvr*'our2cks'
a
When pond water is examined under
Ponds are home to a huge vuiety of microbes' t
microscope, the ut"*" *ntti'iaiio;;ilttbe; iti p"r mr' If there were at least
find 1.he value of &' l2l
rnicrobes in 95% of all l-ml s#ptiip""J*'t"' "ttti"
luvur J{"raY
Find the probability thaf a random sample v' 3 ml of pond water contains at most 3
of - ---'
l2l
microbes.
study'
are collected for a Pafiicula' research
Ten samples caclr of3 ml ofpond watcr contain at
(i) Find 6re probability --- of tlre ten samplcs will each
dtat;;;t;o
e^*' lrrv out l2l
most 3 mic[obcs. . . ... .. samples will
probability that the ten
(il) Using a suitable approximaioq find rhe I3l
contain more than E3 microbes'

L P F2


9 The heart rate (r) and diastolic blood pressurc (y), boft in suitable unis, were
measured for each of ten hospiul patients after bcing given a certain drug
Unfortunately, the nurse who was in charge of recording the measurements did rot
label thc data sets, and thc rcsuls, hencc labelled 'Dda Sct ,r" and 'Data Set n", are
given in the tablc below.

Data Sct 51 75 54 58 63 78 49 70 64 68
Data Sct 88 70 58 91 82 71 90 77 85 76

With ttrc aid of a suitable diagrarr1 @rnmcnt on tlrc conelation bctwecn tlre 2
data sets, pointing out any ine7ulrities, ifappticabla I3l
A docror, on closer look, discovcrcd that not rtl data poins arc rcliablc and thaf
the value of the linear prcdmt firomcnt comldion coefficicnt is only
approximdcly -0.4. Suggcst which pair of data points tlrc doctor should ignorc
so as to obtain a betrcr valrrc of the lirrcar pmdrct momcnt corrctation
coefficicnt, and calolatc tlre new tincar pmduc moment correlation
cocfficient.
t3l
(iii) With the pair of dara poinrs dropped in part G), tlre cstimated least squares
regression line ofy on r is giveit as ,, = 128.5-0.?4r.
(a) Shte what data set .8 reprGcnts. tU
(b) Estimate the heart ratc when the diastolic blood pressure is g0, conectto
the nearest whole number.
(iv) l2l
Give a rcason why it might bc unwise to use eithcr of thc rcgression lines to
establish diastolic blood pressure when the heart ratc is 90.
Ul

10 In the 2006 graduation cercmony of a particular university, the proportions of


graduates from the 4 faculties were c follows:

Engineering Accountancy Vetcrinary Science Mass Communication


42.1% 21.9% 8.6% 274%
Tte management office invited the graduates to participalc in a survey on the amount
of time they spnt at thc central library during thc month of their final year
examination.
(a) Describ how a sratified sarnple of 50. graduatB may bc.takq to reprcsent the
4 facuhies. Write down one advantage and onc disaivantage of this method of
sampling I4l
(b) Frcm a suney of the 50 gradusts, r rpreseob thc number of hours the
graduates spent at the central lihary. The surnnariscd daa are
as follows:

f,.r=1564 I(x-rO)2=zgO
1i; Catorlatc the unbiased cstimucs of the populdion mean and variancc. [31
(ii) Estimate rhc probability rhar a random iample of 50 graduates spcnt;
average ofat least 32 houn at the ccntral library. 121

[Turn Ovcr

l12 Mathanncs, tlm plF 2


r mas oF a mndomly choon shidte mttshroom a no l dLttbution w


mean S.C12g and standard deviatioo O.833 hile the mttC of a rando ly chostn
abalone mushom has a nomal di bution with mean 7.12g and standard deviation
l.053
(1)F prObabJity that a randomly chosen shitake mushroom has a mass of morc
mwg 08.Find w. 121
(lD Shitake mtlshomls are sold in packets ofsix.A customer bought eiJt aba10ne
mtlshroom md a padcet ofshLk mus nts.Fid the prObabnity that twicc
the mtt of tt packet of shitakc mush more han dltt Of tt el t

one mlshmon.
ab
141
oll)A mdOm piC Ofeuty abalonc mus
Cthan 7g.
iS taken.Find the probability
121
lCan alH isi

12 At Fun-GFms, a class docidcs on a gamc which rcquircs a pl1ryr to pick somc


6 rc( 4
colourcd car& wtich arc indistinguishablc cxccpt for their colour. Ttrerc arc
blue and 2 Pllow crds.
(e) Find the nunbcr of waYs in which
(i) any 3 cards are selected wilhout replscnrcnt; t3l
ilil oniy I I crds are arranged in a line- t3l
4 wi
' ' i; class decides rhat rie game is pJqcd by
(b) picking cards tout
replaccment and the total point! are addcd from the 4 cards with each
colourcd
card assigrcd a different score as follows:

Redi l pont Ycllow: 3 points Blue: no Point

Fhd tL pmbability dlat a player


TLcards ven
%l
el S

13 . -A lootrvear gompany wishes to check


out ttre ayerys9 lenAt of,a,9a!]1,:l1EHi
-initat n ue ietectea randomly, and their left feet measured tn centlmeEes' Inc
measurellTcnts are:
9.5 8.9 86 104 87 1 102 89 96

Thcse measuruncnts arc uscd to trt, at 5 % signi$cancc


la'el, the hypothesis tha dle
mcan foot-lcngth of toddle'rs is l0 crn.
'Ei;lrii;rd 5% significancc lcvel' in this context. lu
.a
td ..-,t
l, uv o. pr,o.e
l,our .*d;-;;h"; o nottni t6t (z-tcst) or d should
(b)
-' stdc, gving -be
,*d,'rfu -ny out ftc tcsL
'-tcst 16l

(c) s#;;;prt*mdeforthetestin(b)tobcvalid' UI

End of Prpcr

' bebe2*-,5,W@'hg'
H2 Mrrrrcoder 9?161 2rxII JC 2 Prclinrinerv Errindtor PtFr 2
Pos.6 of6
Anglo-Chinese Junlor College

2 7Jd2 Soludons

Diagraln 2:Graph oFf


l
`
Diagram i:Graph of f

c minimum pdnta =O h diagrall1 2(graph Of f) rrcsponds to


the r intercept in diag-1( of r)

2 "li f* = *!i ax a = rlxzl =2"(*2 -t)=n r

- ,=Ji or -Ji7 lrepcccl


:. y2 =4Ji - y=z(ln) or -2({i7)
Hencc diameter of rim of cup = +(Vi7)+. f Z cnr
3 ur =0.875rro+50
ur=0.875ur+50
= 0.875(0,E75rro + 50) + 50
l

= 0.875r,r0 +50(l + 0.875)

ur = 0.875'uo + 5o(l + o.szs + o.tzs')

u, = 0.87 5'uo + 50 (l + 0.E75 +0.875, +... +0.875"{)

= o.87s'u^ +so(l
-0'875')
" ( t-0.E7s /
= 0.875"uo + 400(t -0.875")
= 0.87s" (u" - ao0) + aOO

As n 1o, 0.E75' -r 0and hence ui = 400

4i)

=tall,
=Sec2 =1+
2

=2 hOwn)

+ =2(
)


When=Qy=Q =: =Q =2

=tan I=0+o)1+(0) +(2) + + = +

41i)

Page I of6

ln(1+tan )=( +i;)
:( ;)2+:(,+i;

= +:
:

DifFerendatlng = +22_

2_..6hOW0
. scc2 2 =1_2x+


5a)
=` , =QL
(
z-4i=2 =2
.z= +41.
Whcn l=Q4= +
When l=1,2.=6j
whcn t=2,z3= +3j

centrc 1,1
radius=2

minimum lz J=15
neod tO ind modulus and arttmcnt OFthe mplex numbcr w
reprcsentcd by point B:

=2- ,argco

(1- )
,=
L=1 - !L*f*={a


F
= hl.{-lnll-.{+lnC=t
tl*l=o(shown)

.'.lnc=o
9 = c=9
:.yr9=* = t=ln1

Page 2 of 6


Jnl h3 h =

9,=3 (1-x) =

= l as _) hoWn)
TF=
=

Pagc 3 of 6
'

7 Lct be tt mtt of our. , ).


P( J al=0.3
P(4 3
,=0
P(Z -1))=0 15 or P(Z 85

,=1.036 =)==0


P( J 2)=P(4 =PC-2.l172 Z 2.072y=Os%2

8 trtffimnfrabte Xbe no. of microtcs in I ml of pond wdcr.
0)
1
=0 95 2


FII 1 1 .

From G.C., 0=0 05 =1

0

`04

Lct random variabic rbc n .Ofmictobcsin 3 mi ofpond water.


y
0
P( 3)=0.0212

Lctrandom vanable S be nO.ofsamples with at most 3 micfobcs.


S-30Q00212)
6= =00170

Lct mndom variabie rbe ncl.ofmicrobes in 30 ml ofpond wacr


r (")
Sin =90>10,. T ( , )apprOX


)
P(r>83) lldndtycorI n


9
`[F

There 8"gatiVC lation bemeen 2 data s Wid1 0ne dear


Oudi wLcn =54,3=58.

iD The doctor shoild ignore tt patr of pointS When/=54 md

lli

ilib)

in
Page 4 of6
fl,..U. z*Z ttclq tson (HZ l|apo f"f* Z)
10o To obtrain a strdiEcd surple of 50 gradu4cs based on lhcir
faali wc d ndom ftom thi agc-group with
s2,in dlc r
tlrc sizc ofeach faculty
Enginecring Accountancy Veterinary MaOC Comm
Science
42% 5021 219% 50 11 860/ 50 4 27.4% 50 14

Advantage: gives good rcpresentativc sample of the populatioq or


a
givs more accuratc estimatcs than random sampling
Disadvantage: samplc not evenly sprcad, or timc consuming or
MorB difficult to conduct and analyse resulb dran
random sampling

; = E'
bi) I
Unbiascd csrimue for popukition mean, = = 3 I.Zf
#
Unbiased cstimate for the populuioo variancc:

bli) By CLT, (31_28, )


( >32)=0.0896

Lct and y be mass Ofa shltakc and an abalone mushFOOm


rcspectively.
(5.02o.832)r (7.12,1052)

f J 0 8 a
=o2
: (6x502,6(083)2)
:


2
[ 11 GX
22x4B34+&
I(1,:
lltt
P(2r> =P(2r S> =0742
'F

=80,7- 7.:2, )

P( 7)=0153

Case I : all 3 distinct colours: No. efwap =1


Case 2: 2 samc & I different No. of ways = tcrr2=6
Case 3: all 3 cards same colour: No. of wsys = l+l
Total = 9

I l!
Case I : without I of the red .ard = 5l4t2t
Case 2 : witfiout I of the blue card - Il!
3t612l

Page 5 of6
3: d10ut b Card=

l R2BIP
=

1+


:= =0176 :
0R
+ = +
fi=Q
6


bli)

2 l bu sOOre O=

= =

0 1

llL
0 1758=0.827
It means that ulm is a o.
foot-lcngth is not l0crn whcn it ac$ally is l0 cm'

The t-test should be used.


[*ontt.rpt" size is small, and population varianc is not known

Ho: P=19
Hr: p*10 at 5% level of significance
Under Ho, 7 follows ttre t-distribution & degree of freedom
d
rest statisric t =i JP = -2.678
7.t;

Since p =0.0316 < 5 %,


at 57o
W" r"i.t" IIo and conclude that there is significant evidence
is not l0cm'
level thar the average length of a toddler's feet

Assumption is 0ra the length of toddler's feet


follows a normal

Page 6 of6
_
lnnovo. JC
2

=ar+ c PaSSeS tt ugh


The curvc with equation tlle points(1,7),(-3, )and


(5,99)FInd the equatlon oF tt curve 41

Gven that =0 4 10"and =5 when = l, furd an exprcssion for y in


telllls of
t61

Let y=0-e3 )]
(1) ShOW that
y2 _ +9=0 t2l
(il) By futter difFercnthiOn oF thls resuL OtttMse,ind Maclaurin's series
for up to and including the term in 2
t4l
I
Deduce the equation of rhe tangenl to th curve y=(g- e3\1 at the point
x=0. lrl

Sketch, on an Argand diagram, the locus of p representing the complex number z


wtrcrelz-2+2tl=2. el
The point O representing complex number w is the point on the locus of p such that
l: + 4il is maximum. Find
(D the exact value of lw+4i1, [2]

(iD the value ofw in the fom r + iy, giving the exact values of-x and y.
3]

The posliOn v tOrs oFthe points ,B,and C are given by i+j+`4i+3j+2 k and

-7i- 2j - k respectively.

(i) Prove that the points,{,8 and C are nor collinear.


(iD Find a vector which is perpendicular to the plane,{rC-


(iiD Deduce the exact length of projection o f FQon te plane ABC, given thar
OP =Zi + qi + 7k and OA =ai + 4i + 6k.

974071 2007
ffum ovel

lnn
3

Let xt,x2, 13,... be a sequenc of positive integers, where \=2 and

xn+t=x,2 -xn+l forallz= 1,2,3, ...-

s,=i I


rf for alt rr 1,2,3, ..., prove by the method of mathematical
i=l ''


induction thar =I-
.S,,
--!-- .


ra+l - I



Civen that -r, > z for atl n +, state tlle value of Iim .S" .

Show that

t43 f-5r
[2]
r r+l r+3 r(r + t)(r+3)'

Herrce find
n <- 1
\--'-
lor(r +t)(r +3)'
giving lour answer in terms of z-
4]

Find the exact value ofthe series


6 2l 36 291


o)ox4)(2)(3)(5)(3)(4x6) (20x2tx23)

-+-+-+...+=-

The functions fand g are defined by

:x-- e2' ,
f r . ,
g:r--t2-6x+L, re',r <3,where l, is aconstant.

(i) Find an cxpression for f-t(.x). tz'l


Find ttr range ofg in terms ofL.


(iD t?l
(iii) -rg exiss. By
Find t, the le6t value of 1", such that the comPosite function f

putting tr = f, find f-rg(r), stating its domain and range. t5I

9740 1 7 ffurn over


4

The equation of a closed curve is (x + 2.y)'? + 3(r - !)2 = 2l .

(D Show, by differentiation, that the grad ient at the point (r, y) on the curve may
4x
- Y-
dY
be exoressed in the form
' dx 7y-x
- t31

(ii) Find the equafions ofthe tangent to the curve that are parallel to
(e) the:-axis,
(b) the y-axis. [61

10

The plane fl has equation The line / passes through the point P with

r,)

r,)
I a I and is parallel to | 3 l.
l"J
position vector

l-, j
(D State the peryendicular distance from the origin to the plane fl . tl]
(iD Find the acute angle between the line I and the plane f[. t4]
(ii| Find the position yector of the foot of perpendicular from the point P to the
plane fI . Hence find the pos ition vector o f the reflection of the point P in fJ .

t61

consider the curve ,-x'-3<zx+6az , x+a,whereais a positive constant.


x_a
(i) Find the equations oithe asymptotes- t3I
(iD Find the coordioates ofthe stationary points. t4I
(aio Draw, on separate diagrams, a skerch of the curve

.
(a)
x2 -3ax +6at
x-a

(b) ,-lx'-3ax+6a1

making clear the main features ofeach curve. [5]

9740/1 2007 flum over


D

2=L icatltt d quttb Of tte



" O Sketh tt Curve oF

Heme ind thc area ofthe"gbn e bsed by e cu e _D2_1,the


(

ne =2 and the r axls,giving pur allswer rrectto 3 d imal places_13]

(b) Find the exact values ofthe r-coordinates ofthe points of intersection of the

line y =3-2, and the curve y = 8-1. sterctr, on the same diagra^, ih" lin"

y=8-Zx andthecuwe.y=8-x3. t41

The region bounded bythe line y=g-2v and the curve !=8-x3 is rotated

ttrough four right angles about th r-axis. Find the volume of the solid of
revolution formed. t3I

F`

974"1 007

ark scheme
Prelin1 2 1H2 Pl
1 Obtalning
+ +c=7
9 -3b+ =ll
25 +5b+ =99

Augmented IIlatr

:5 :3 1 ::

Alt: -3

y=3
2
oO =0_4 y)

=f04
equivalent

Mthd r:l tl!-12a,=


. JO.+d, or using partial fraction.
ilto_y)
+
O n =i4
Mthd l--i-
2: Jz5-(y-5\2 a"=
' lo aa"
-10

-
hl=
+C
"tllvalem
, =
l
nu1/=5 /= 4,

c =


"

=
3(1) l

y= _e3 ),
3=9_e3,

2
3 =_3e3 =:(9-23 )-2/3(_3a3 )

y2 =_c31= -9

2
_ 3+9=0

3(li)
2
+2 (1::)2_3y2dy=0

Whn =0,y=2
dy l
d 4
d2 _ 13


2 16

=2+( , ) +
(

y=2-:
,2+

3citD I
Equation of tangent b y = )- '-
v
4
4 -2+zil=z
lz

+41=PQ=PC+CQ=V22+22+2=2 +
Rccognising 45
ed and attcmptto fmd opp side Or adJ side
Obtaining =2+V5 o =_2+V5
=2+V5+1(-2+V5)

5(1)




Find any 2 ofthc 3 vectOrS: =} =::ll, =:

Sincc/ not paral:eltO BC(or equivalcnt), / &C nOtco!:incar



5(11)

IXI:l IXI:10requiValCnt veCtOrtttOplanc/BC=::


5(lil)






2 ri
O lx1 2
-1, 1-7 7

MILd l: Lngtt ofpmJn=
= 0
2


nof P2 0ntO 9
Mthd 2:length oFPr =

Using Pythagoras'ThQ length Ofprttn of P2 0ntO plane=

6
hL hk me


,
C

W n 1,LHS= =:,RHS=l = = =LHS



(,12 -,1 t)- + t
=1-

12_2+ J 2


Therefore,Pl ls truc

Assume P iS true,le,: 1-
eJ

J + J
(= )J ( )=MS
Hcnce P +l ls true
Since Pl true and truc implies L+l true,hence by MI P"L true fOr all c'

Since > , as ,I =!
(1- )=1
7(i) 3-5
To show l +3
+l +3 r(r+l)(r+3)'
lthdl:

1 4 3 (r + l)(r +3)-4r(r + 3) + lr(r + l) (r' + 4r +3)-(4r' +tzr) + (3rz +3r)


- + _
+l
+3 r(r + l)(r +3) r(r+ l)(r +3)

Mthd2:
3-5 B C
Lt = +7.l+r.3'
r(r+ l)(r + 3)
E R ngCo hL = ,3= =4,C=I

OR 5 -3=/( +lXr-3)+Br +3)+Cr( +1)
Sub =0,get A=1
Sub =-1,get B=4
Sub r= 3,get C=3
3-5 1 74 3
7( +1)( +3)= l+r.3


7(lED


3)=:]( :+
: )

=
( : )+ ( :)

=3

/
=3[:+: -1]
]+[
3 1 3 3
= +
2 +1
+2 +3

6 21 29 .
36
5 3 _2rl_

(2x4)'(2x3x5)'(3x4x`)'1(20x21x23)
+1)( +3) k'T T 5
(

= emaluep
-3) ,sR=
Alt rthd= use scquence
= 3(5
;:: GC command:sum(u(1,20)
8(1)
f: e-21, c. Lcty=e 2 , x=_:in y

f :( )=
:in X, X>0
8(11) g )= _G+ =( -3)2+ _9, r . ,r<3,
Rg=( -9, )
8(lil) f lg exists c)Rg Df l So( -9, ) (0,)= 9,ie t=9
=9, g( )=( -3)2

f lg( )= -3)2) =_inl -31 or in(3- )


:Ll((
Donlah=( ,3) Rangc=( ) r
9(1) ,+2y)2+3(
)2=27
+2 +2dyb+ X -2)=0

(x+2 +3 -3 )+(2 +4 3 +3"dy=0

(4x )+( +7 )1::=0

"_7

Tangent parallel to r-axis: Tangent parallel to y-axis:
y -4x =0 7y-x=0
y=4x x=7y
(x +Er)z +3(x -4x)'1 =27 (9y)'z +3(6y17 =27

8lx2 +27x2 =27 189 y2 = 27


,rll I
x'=-; x=l- 'J7
Eqnsoftangent:y=t2 Eqns of tangent: -r = t.,,/7
10(1)
nettm hm tte Lhb = =

Mthd 2:
Mthd l: Let O be the angie be veen and ` 10 bcthc ang betw n and _

coso = =28 5in0 =

0=348 0=55_2
Angle ben ecn and =90 -348 =552

l,et /V and P .be the foot of perpendicular of P on I and the rcncctlon oF P


respectively.
kt /r, be the line through N and P .

'- '{i].^[],J -=[i].^[-i]''*'" ^

snce,v sap,.n, + 33+,4'=5, 2

l[iJ.^[_i]][i]=,

x' -lc + 6a2 -2 x, +4


= =( = _2 +

Eqns ofasymptotes: x = a, y=x-za

J
Let dy=

=1

=2
= or3
2)2_3 (a)+6 2=_5
When x= a,y=(

When =3 ,y= =3 The rd s arc ,5 and c 3


l_ _1)2=l Eqns ofasymptotcs: 1= =, = +l


: :
VeFtes at(2,I)and Centre at(0,1)
RecognISIng thatthe eqn IS y=1-[ IFlandgetting,=J when y=0



8-l =g-2, = x1x2-21=0, x=0,+Jl

u", :"j'Jr(8-2r)2-(8-13f a,*n[o (8-rr)2


-(8-2x)2 dr
=32 or100_53 cublc unis
lnm 3
2

Section A: Pure Mathematics [40 marksl

Find the 4tt roots of the complex number .6+ i . Cive your answers exactly, in the

for- reio. t4l


Herrce solve the eqrrtion z8 - (2.,6)24 +4= 0. Give 5rour amwers exacrly, in the
form reio
14]

The diagram shows the giraph of y: (x) with stationary points at (_3, 3) and
(1.5, -4). The curve cuts thc axes at (-4,0), (_1,0), (3, O) ard (0, _2).
On separate diagrams, sketch the graphs of


(D y =2f(x-3),
(iD y2 = -f(x),
(rlll I
-Y = f(-r)
-

9740 2oo7
ffum over
3 .

) h ageomart p ttsbL e ntt terln 2 7 md L commn


:
(1)Find,correct to 2 decir al places,the valuc oF the sum oF an the
negative terms ofthe progrcssioo
31

(i0 Fird the least vatue n such that lU,l.*, where IJ, denotes the zth

term ofthe pmgrssion. 14

( An arithmetic progression has first term 2 and mmmon dif[ercnce d, \Ylcre d


is non-zero. The seoond, fifth ard tendl terms of the progtession are
consecutive terms of a geometric progression. Find ttre sum of the first 15
terns of the aritfunetic progression. [5]

Prove that
d (tan'x)=-.
-r I
dr l+x'
R M 14]

m m exact v F12



O m thccxtt d J12 ,2 -ly dr.

9740 2007 ffurn over





S tion B:StatL cs 160 nlarksl

,6 A kindergarten has nine Year One classes and six Year Two classes with different
class sizcs. An education ministryofficial wishes to call on th kindergarten to visit
five of the classes- Describe how he could make a random selection of the classes
using


JY simple random sampling,


stntified sampling.
y/
The Prircipal of the kindergarten wishes to take e sample of 30 children to meet up
with the ministry official for a short conversation. She clrooses 2 children at random
from each ofthe 15 classs. State, with a reasoq wlrether this gives a random sample
of30 children from the kindergarten. [11

(, Fird the number ofways in which 9 people can be divided into 1 I . -


` /
(i) two groups consisting of5 and 4 people,

9i, three groups with each group consisting of3 people. pl

Four men and five women sit at a round table. The seats around the table are
numbered I to 9. Fhd the number of ways of arranging the nine people if all
the men must sit togcther.
,t2l
,) \,
// For any married cOuplc whO arc membcrs OF a tcnnis club,the probability that the

husbmd h au ve y degree`:md me prob ihy ttt wlfc h au ver

dcg e bab y e husband has a uttvesly deg e, ven thtt he


:The p

H h au vetty degree t

A married couple is chosen at random. Show tlat rhe probability that both of
them have university degrees is !.
24
lll
married couples are chosen at random.
Two Find the probability that only one
of tlrc two husbands and only one of the two wives have university degrees- [4]

t I ia I t:' ', t rz._, rc

t'

, 1 `

974 007 ffurn over






OR

(a) In a large company, it was found that th proportion of people who own a car
isp, where 0.5<pcl.

A random sample of 15 people from ttre company is taken and the


random variable X is the number of people in the sample who own a
car. Civen that P(,Y = 8) = 0.081 13, fird tlre value ofp, giving lour
answer oorrect to I decimal place. [21

Assume that p =0.945. Using a suitable approximation, find the


probability thal in a group of 60 people randomly chosen from the
company, thcre are more than 55 people who own a car- I4I

On average, a particular information cour cr receives 2 enquiries in a l0


miru.rte period. Using a suitable approximat'torl find the pmbability that the
number of enquiriB the information counter receives in a 2 hour period lies
between 20 and 25, inclusive. [3]



Water in a reservoir undergoes a puriflcation process before it can be co umed Thc
efFectivcness( %)ofthe process for various ow rates(Xm s )t shoWn below

l 2 4 6 8 10 20 30 40
80 _45 40 30 25 18 15 10

The variables x and , are thought to be related by the equation e/ = q1b , wlterc a

s
and 6 are constants. 1

Sketch a scatter d iagram of y against ln x . \--- ;'i

2]

(n y lnr. tll


Fird the teast squares regrcssion lineof on


W Describe how 1ou could use pars (i) and (ii) to assess how well the data fitted
the calculated equation- [l]

of a atd b .
Estimate the values

12]

ftedict the effectiveness of the Plocess $'hen water flows at 49


1

m3s

,/Comment on th reliability of pur prediction.


2

/) Comment on the vatidity ofthe above model for large values of x. [!]

9740M007 ffurn over


6

10 In order to diagnose a certain disease in adults, the hormone levels of these adults are
measured. Past studies have shown that it can be assumed that the hormone level ofan
adult is normally distributed with mean 35 units and standard deviation 7 units. An
adult is classified as healthy if his hormonc level is in the range (22, 48). Otherwise
he is classified as unhealthy.

Find the probability that a randomly chosen adult is healthy. tll



A sample of 90 adults is examined. c
One of the adults is Wong. Find the probability rhat the differencc
between the hormone level of Wong ard the average hormone level of

the rest is at least S uits


14l

Using a suitable approximatioq find the probability that fewer than g0


aduls in the sample of90 are classified as trcalthy.
t4I

It is decided thar the range (22,48) should be replaced by (35-a,35+a) so


thal not more than 3oZ ofdll adults will be classified as unhealthy. Find the
range ofvalues of a.
Vl

9740722007
ffurn over
7

11 1) The mean height ofa mustard plant was found b be 5.1 mm after 4 days of
growth. A random sample of l0 muswd plans had the following heighs, in
mm, after 4 days of growth.

))-. 5.0,4.5, 4.8, 5.2,4.3,5.r, s2,4.9,5-l, s.O

--/. Using a 6% significnce lvel, test whether the mean height ofthese planfs is
less tlnn 5.1 mrn. State any assumption made. [5]

Explain the meaning of"6olo signiftcance level" in the context of this question.
tr I

(b) In genera! the marks obtained by students in their Mathematics examimtion


nray be assuned to be a random variable with mean 65 and variance 32.

After the intoduction of a new program of revislon classes, it is found that


there are improvements to the marks ofsome ofthe students.

A random sample of the marks of 50 students is taken and the mean is found
to be 66.4, A test is conducted and it shows that there is insuflicient eviderrce
that the new program of revision classes is effective. Find an inequality
satisfied by the sigoificance level ofthe test. t3l

State, with a reason, whether it is necessary to assume thal the Mathematics


.orn1*rion.".kt follow a normal distribution. ttl

G) A sample ofn observations is taken.from a population with mean 5' The


sample mean is found to be 5.t and samPle standard deviation 0 7' 2-tailed
-A
normal tes 1z - test) is conducted and the null hypothesis is rejected at the 59lo
significance level. Find the least value ofn. t4l

d Ent of Qdper<,

974072007
2007 prclim2 maths solution tD pryer ?

1
La z bea46r t of +i.
1
:

Then z4= +i=2e


+2
=2c( ), =Ql,23

z=2: +:
),1= ,12,3
z4=N i

+:h),
z=2:e( =Q12,3
2i

5,-8)


2ii

(l

(15,2)

2iii

pg i of6
2007 prelim2 maths solution H2 paper 2

3ai The negative tenlls ofthe series L U2,U4,U6,

2007( 7(
:),2 :)3,24X17( :)5
New GP ith frstterln 2 7(
:)=-1561
Common mtb(
:I=1:
SO Sum tO ininity eX and S =i:ilill= 395128

3ai


:
7

J:

2007

)H

(
-1)ln(:) in(
5:7,)

-1)>
(

>6152
Least n-62
ALerllajve!v Keyinto GC:Mi n)=1
Un=20t17(-7/9)
l

Umin=2007

:
Least r 2
3b 2+4 2+9
2+ 2+4
2+16 2+20
16 =9

=1
7



pg2 of 6
2057 prelin2 matls solution Il2 pqer 2

4i Let y=tan l .

Then tan = .

D ga s
=1
"ndate
2

= =+
1
2

2 2_

4ii

sec0 tan0 d0

15
I

=1
1


FSeCOtal10
d0

=lirSeco d0

+tan:
=in, :
sec O+tan 0

=h(2+ )

4ili

,2_ =[ tan

tan

5
(ii) Radomly setect 3 classes fromitrelTEl classes, and randomly select 2
classes from the 6 Year 2 classes.
Not a rardom smpt.- sin""@
equal chance ofbeing selected.
6ai "c, = 126
6aii
l'C, x "C,l/3!= 280
6b
(4) (5!)19)
=25920
7i bOtt LVe degrees)
=2(W:fe has JQgreC)X Lusband llas degree ifwife hasJpgrcC)

pg 3 of6
2Co7 $elirnz maths solution 112 paper 2

1 11 11
2 12 24
7ii

5 24 2 24
:7 1

120 24
1_1_ L=
5 24 120


d prObttity ls
+ = )

8ai

8aii O B(60,0055)
y Po(33)
P( 5)=P( 4)

=0763
8b Po(24)
N(24,24)

P(20 25) P(19 5 255)=0441


9i

..
in

91i y =73.3-l8.4lnx
9111 Draw the regression line in(ii) n the SCatter diag in(1).Ifthe data points are
close to the ight hne, n ttd flts the calculated equation
9iv
a=e
= 6.E9x lOil (3sf)
D = -18.4 (3s0
9v 1.840/0(3so
Since x=49 is out ofthe given data range of l 40, prediction

unrellable
9vi n- trrg" *t,res of .r , the model gives y < 0. So the model is rrot valid for large
values of r -
10i P(22 48)=0937(3s0
'

pg 4 of6
2QO7 prelim2 matls solution II2 pryet 2

101ia
(

)
N(0,49+:;)

P(l

'1>5)
=1-P(-5 5)=0478
`-57
101ib BK90,0937)
(84_30,5_34)
P(y 80) P( 79.5)
=0 0188 osf)
10m P(( 35- ) ( >35+ )) 0 03
P( r 35- ) 0.015
35- 19.8
15.2 OSf)




pg 5 of6
2007 !/.eltrn2 maths solution 112 pqet 2

1la Let X be the r v forthe height ofa mustard plant ater 4 days ofgro vth.
To test= : =5.1
AgJn Jf: 51
Perform a l taiied test)test at 6%sig M.
Enter into CC: =5.1,LLt:L:,Freq:l.Choose
We obtaln p=00380(t= 2004)
Since p 006,we reJect lL and cOnclude at the 60/O sig ivl.,that there is sufrlcient
cvidence that the frcan height ofthese plants is IRC than 5 1 mm
Assump on: helghts ofthe mustards plants after 4 days oFgrowth is mm ally
distributed.
`%,6 sig.lvl"Means there is a 0 06 plobabilly thatthe test shows 1 mean
height ofthe m rd PiantS after 4 days of o 4h is less than 5.lnlm when in fact

itis 5 1mm
1lb To test .:=65
Against 4: >65
Pcrform a l tailed( tes at %Slg 1

Urder H`
(65,::)by CLT
SInce tt notreJected,P vahe=P( >664)> %
004 6> %
o% 401%
OR
Enter into CC: 65,c= 52,I=664, =50

Ch se >
We obtain p=004 6 Since Ho is not reJectcd,
004 6> %
oO/0 41010/0
No. Sirr.. the sample size 50 is large, the mean marks of 50 students follow a
normal distribution approximately (according to CLT).
1lc To test : =5
Against [: 5
Perforrn a 2 ied(2 teSt)at 50/O Jg lvL
Since
`reJ
> 96
Z>1 96 or Z L96(na)

>1_96

Least n 190
">1892384

pg 6 of6
Admission No.

Candidate Name:

M:d Year Exarllinations 2007


Pre-university 3

MATHEMATICS 9233
PAPER 1 9233t1

Monday 1O September 2007 3 hours

Additional materials:
Answer paper
List of Foimulae (MF'l 1)

INSTRUCTIONS TO CANDIDATES

write your name, class and admission number in the spaces at the top of this page and on
all the
work you hand in.
Write in dark blue or black pen on both sides of the paper_
You may use a soft pencil for any diagrarirs or graphs.
Do not use staples, pape( clips, highlighters, glue or corection fluid.

Answer all the questions.

Give non-exact numerical answers corred to 3 significant figures, or 1 decimal place in


the cse of
angles in degrees, unless a different level of accurary is specified in the question.
At the end of the examination, fasten all your work securely together.

iNFORHATION FOR CANDIDATES

The number of marks is given in brackets I lat the end of each question or part question.
The use of an electronic calculator is expected, where appropriate.
You are reminded of the need for clear presentation in your answers.
This question paper consasts of S printeA pages.

ffurn over
h" w

n"
15]

2 Express i(2 3+5r)h mSd _


i51
=2

3 Express X2+5 m pa J ttd10n,

Hence or othettserxPand(1_ upto he tem m x3


2+2)h ascendlng powett of
)(

State the values of fbr which the expansloo is va:id 171

4. Prove
1x4 4x7 ^: t 1)=-\
by induction tnat' ;!+;!+...+;-" (3n-2)(3n
3n+
, for all 1
posrtrve

-
integers n. Hence, deduce the vatue of . Vl
=trir-hrE
5. (a) The complex numbers z and w are such that
w=1+ai, z --2i -b,
. where a and b are real and positive. Given that wz -= -7-i, find lhe exact values of a
and b. l3l
(b) The complex numbers p and q are such that

lpl=z a's(P)=;, lql=z' a6li6;r=-;'


(i) Express p in the form a+,b. l2l .

,.5
(ii) Find the modulus ana argument ot I I|. t3l
-l.pj
6. Given that x, 6, x+9, are the third, fourth ard fifth terms of a geometric progression and
that the sum to infinity of the series exits, find
(i) x, l4l
(ii) the sum to inlinity. t3l

-2-
7_ O The graph J y=flxl t Shom ",me o, and 10a
OdnaLs d
points A and B respeciveiy_Sketch,on separate diagrams,the graphs of


(i) j/2= (x),

y= (
showing in each case, the asymptotes and the coordinates of the points
conesponding to A atd B.

( SketCh the curve of y=J+(x_2)2'Stalng the equatlons of any asym otes,

ooordinates of any stationary points and coordinates of any intersections wlth the
axes
15]

On a single Argand diagram, sketch the following loci.


(i) lz+4-3,1 <5,
(ii) lz -til<lz -zil.
Hence, state the minimum value of lzl.

9 (a) ln hou, many ways can 3 retters ftom the word 'ATTENDANCE be ananged in a row
if at least one of the lefters is E.
141
(b) 5 manied couples bought 10 seats in a ror for a movie. rn how many ways can they
seat themselves if
(i) each couple is to sit together,
12

(ii) all the men sit in a group to the reft of afl women who are arso seated in a group,
given that no couples can sit together.
131

-3-

10. Shorv that t(i=!-Qxz
x- -l) has exactly one root, a. a2l

Using linear interpolation once on the interval [0,11, find an approximation, xs, of a,
correci to 2 decimal places. 121

Explain whether xo is an underestimate or overestimate of al l2l


Taking the value of obtained ea.lier as the fitst approximation, use the Neu,ton Raphson
process to find a, conec{ to 3 decimal places. t3l
Explain why the Nerrvton Raphson method, using an initial value of xo = -0.5, cannot be
applied to obtain a better approximation of a. 121

11. By means of the substitutisn z:x+y , shol that the differential equation
dy +2y +l ro
dl . Hence, or otherwise, find y in terms of
-2x may be reduced
' dx=3'-.1
dx x+y-z z-z
x. Vl

12. Find
- l-4x
(i),-- 141 -
{l-x'
,t
t2^
(ii) i sinz 3xdx , leaving your answers in terms of r. 141
0

13. Answer only one of the following two altematives:


EITHER

It is given that t(0)=zc/,sz e-4sinz e - cos(u-1).


'2'
(i) Express f(q in the tom, a@sze + bsinze + c , where a, D, and c are constants'

(ii) Show that f(4 can also be e)Qressed as .6cos(2d + 63.4") - I .

Hence, or otherwase, find


(iii) the general solution of the equation f(d) = 0.

(iv) the maximlm and minimum values of j; ana determine the smallest positive
f(0)
values of , at which these occur. 141

-4
OR
The diagram shows a cuboid with a square base of side 4 units and the height is of
length 2 units.

It is given that M is the mad-point of AB.


(i) Calculate the angle btween the skew lines HM and AD.
(ii) Calculate the angle between the planes MGD and MBCD.
(iii) Show that angle EDM is 36.9'.
(iv) Hence, or otherwise, find the perpendicular distance of E from MD.

ggg ENooFpApER \t &, &

-5-
l :



0 0
R.!bh0o.tNo. .- --
N

Cl _- P.p.. No.-, --.--.-
illennia Su --
institute c 38 D!t.
Pv^ < e'

h ) t'f,2 '11'2
"u
.!-
lx a)(x-:-) -.6-
oa) *,2 *o, tt,t lt

++
t-.2 cjry o4X <2



<J-
',-lrl


r `
1-2
`
( '1,) lrl 11) _ ___ 4)
(
r

_ J
3'
dmlDI

C rO ``
`

P 1

(2


lKI`
)

i
`
an3 t`2

oz
-41,tzl



0


J Name R IIBt701 1n No

Sublect
illen nia _ Paper No
institute Cl 38 Dal
(4t
, 2,

( 4)(
), 1
A
,
`
_
LI`l =

,
D4
4 ,31
= 3

i 'Ittl


FFTT


'3
l 413=

= 1)` -3)

:
'
'

l l
l



I Pr4`rt


`14
1 * ntZ+





1
' = 1

t?
1 1



l l l

Ma

`

R l NO _

N


__ _ P _
111ennia Su "N
institute clos _____ to
_


xi 1

( i ;: 1

,3

),4
(

= 8

8
= 1,
r

3D


0 ,t I M..r! i
J NBm6
Sublscr
R.g'rlreuon No, . ---- i
tt I

illennia ..__._. __-_P.pu No

institute Cla!s...- .. . .. - O.i.



.

'(
t' 1

, 11)11`
12 (
121,i 12-

AM-ll l

21J
2

1os


=0

1 1`-38'
o

1 `11 y

FT N
ra (D:`, :do l

' `'
: l,=

(

1=tJ`^

[
:i
o
:0 { 1
0
0
mm muon No_III I

_ PBper No_________
' ).r,s'i' 43ri.lp -
inennia Subled -
o) '= 1pa (
institute c1831 __ _ Dat8 _ _ ;fi;t ;i'i: t;ire -.oDOLra
=3` `'

2
l

=,f l =`34
( =
/Lf
l!

3.
: fr +l futo"*\=-- /3 I

7 = 1
P.Y':...+t-:..Y...
: , + g .471

F ``
3
=
4:

0=
4



= x , "

2 =J 11_1:
= IK l tr
4

7.( )) "
1 2 ll )fllx_21 l
`

lpil p
r_J
0 '
9
10 (y ")] A^t . 'T
r t 1) C(K 2)

( I!`' 1

=

.r.
l'' g;'t 4s*
l:1li1ll x31 3 y 9
t =


( h(
:

1
'[ (X')
Ili:I
1
T
,
=

9


J Nemo _ _ R0013n On No

Su OCl _ _ _ ___POper No
illennia
institute
Clu6! _..._ -. _ _. O.t

AX

(= )L) o ( {)(l
(`a ) .

= )
`x) ` ` `

64
:
`

x

Jo - !,,A>X - t.C>l l.ptt-
48 `
, (lk _) ( l- rejr.)!

::F:[ =:
F
l
53 1
Fl
- 11

&t c* L y'hu F6 D*
^<

"fnrl , + M\Z
;< '

,{}1 =
}'tY2+y?) -lr/ V lf?i",<l#.fa
25' 4.4
5 .`
:HTz=76

lnJ '(A

T"

I`tt
f Y2
Y

Ji8 .14`
3

: : : :

lndex No

Candidate Name:

millennia
institute
Preliminary Examinations 2OO7
Pre-university 3

MATHEMATICS 9233
PAPER 2 923312

Thursday 13 September 2007 3 hours

Additional materials;
Answer paper
List of Formulae (MF11)

INSTRUCTIONS TO CANOIDATES

write your name, class and index number in the spaces at the top of this page and on all the work
you hand in.
Write in dark blue or black pen on both sides of the paper-
You may use a soft pencil for any diagrams or graphs.
Do not use staples, paper clips, highlighters' glue or correclion fluid'

Answer all the questions in both Sedions A and B.


place in the case of
Give non-exac{ numerical answers conecl to 3 spnificant figures, or 1 decimal
angles in degrees, unless a different level of accuracy is specffied in the question-
At the end of the examination, fasten all your work securely together'

INFORIIIATION FOR CANDIDATES


question.
The number of marks is given in brackets I I at the end of eacfi question or part
The use of an elect onic calculator is expected, where appropriate'
You are reminded of the need for clear presentation in your answers-

This qu;s6on paper consists of 5 printed pages-


ffurn over
-2-
Section A: Pure llathematiqs


1. By sketching two appropriate graphs, show that the equation x =4-x3 has only
real root o.

Show that the iteration formula *n*, = .E a may be used to solve for o.
Yxn
Apply the iteration formula, with inilial value 1.35, to find q. Give your answer to
decimal places.

2 (a) The lines l, and I , have equations

l=[iJ.^[ti] 2n6 r71=z+3 -2


'Y

respectively, where L is a parameter.


Determine whether the lines intersect.

31

lbl Gi n ha!al=3 and lbl=2,and a heangettmen e and 2 b 1lnd he

vJue d(2-9)
:G e youranttrcOmd b hree sb F 19u

A curve has equation y= nd the x 0 rdinates ofthe oointS On the Curve such

hatthe normJ b the cu e at each pdnt parJ itO he ne y=+


G en hatk= Fnd the equalons ofthe norrnJ

4 The region A is bounded by the curve y = 2sinx , the y-axis and the line y
= J3.
(D Find the exad area ofA.
(iD Find the volume of the sorid formed when the region A is rotated 2rr radia",
the x-axis. "JtJ
I4l

5 The functions f and g-r are deftned by


l: x-+ex +1, x>0,
g-1 : x-+Jil7, x>3.
(i) Find g(X)
[31
( ) Exp:ain whythe composlte fundOn f g l exists and ind the function
141
(ili) Find the range of f g 1
[21
-3-
Section D: Probability and Statistics

6 The random variable X has a normal distribution with mean100 and standard deviation 5.
50 independent observations of X are taken. Find the probability that the sum of the lirst
30 observations exceeds the sum of the remaining 20 observations by more than 1050.
t4t

7 ln a lae company, 1 in every 12 employees is a smoker.


(D Find the pobability that, of 6 employees chosen at random, exactly two of them are
smokers. l2l
(iD N people are cfiosen at randorn. Find the least value of N so that the probability
that at least one will be a smoker exceeds O'9. I4l

8 A manufacturer claims that 6506 of hb products are of Grade A A sample of 1240


products was taken. 776 of them were found to be of Grade A. Test, at 5% level of
significance whether he has overstated his claim. 16l

State what you understand by the elgression'al 5o/o level of significance' in the context
of this question. I1l

9 (a)



_____----___-__
c,

The tree diagram shows hovrr three events A, B and C are related to each other and
the various probabilities associated with them.
0 .
srate P(B I A) tl l
(ii) Find P(A'|B). 121

(i.i) Fino e (Auc)']. t3l

(b) (Monte Hall Problem)


in the popular game show "Let's Make A Deal" which was hosted by Monte Hall,
the stage was set with lhree doors. Behind each door was a prize. One prize was
very d&irable and valuable. The rernaining two prizes were undesirable. After the
contestant selected a door, another door was opened to show an undesirable prize
(the host knew where the desirable prize was) and the contestant was given the
choice between the already selected door or lhe other door that had not been
opened. Should the contestant switch doors? Why? l2l
-4-

10 The demand for orp cakes in a cafeteria may be taken to follow a Poisson distribution
with mean 0.6 per hour. The cafeteria opens for 5 days in a week and operates for 10
hours per day.
(D Find the pIobatility that the demand for cup cakes is at least 5 in a day.
(ii) Find the probaulity that there is at most 1 day with zero demand for cup cakes in13]a
week. (Give your answer conecl to 4 decimal places.) I3l
(iii) Given that the demand for cup cakes is exactly 6 on a particular day, find the
probability that this occuned within the first half ofthe day.
t3l

The (1fi) - cr)% confidence interval for a population mean p is (ab). Explain, in terms of
probability, what q96 means.
I1l
A teacher suspects that the students in her school are not having enough steep and
hence face the dffiorlty of staying awake du,irg lessons. She surveyed 6g of them ard
found out the number of sleeping hours each of them has on an average school day.
The data gathered was as follows:

No. of sleeping hours No. of students


3 0
4 4
5 22
6 30
7 8
8 4

Find the unbiased estimates of the population mean p and variance d of


number of sleeping hours the students have.


Find a 90% conlidence interval for p.

A symmetric (100 - G)o/o confidence interval is found to be (5.Sg , 6.00). Find o.

12. ln a country, 80% of the population weigh more than.-Sokg. Assuming a normal
distribution, show that the mean p and the standard deviation o of the disiribution are
related by p=50+0.&42o. pl
Given also that the probability of a person from the population weighing less than Tokg is
0.65, find the values of U and o.
let
'A random sample of 76 people is taken from the population. Find the probability that
more than haff of them weigh more than 70kg. l5l
-5-
13. Ansnter only one of the following two aftematives.

EITHER

The continuous random variable X is uniformly distributed on the interval [ -1 , 5 t. Find


the cumulative distribution function of X. 121

Two independent. observations, Xr and Xz, are made of X and the smaller of the two is
denoted by Y.
(i) Use the fact that Y > x if and oniy if X1 > x and X2 > x to show that the qrmulative
distribution function of Yfor -1<x<5 is given by


Flx1 4-

Hence nnd thatthe probability density function of Y



nd Var(Y)_


State the mode of Y

The cumulative distribution function of aa discrete random variable is


X 0 2 5

P(X X)

a b 1

Giventhat Eq x D E(X)=;and PIXl=2 and X2=5)= ,lnd a and b l

(b) The discrete random varlable Y has unfo distribution over the set of values(4,6,
a an+ }shtt hd RY
31

Given that n=199,lnd E(Y2) "=


31

END OF PAPER


l l

7
2` . _J 2
1)


q _A )

P]
:

)


FX _
li
1
:lli4J:
f )

s ,2 0:
`6

o'1 i"
"dJ,-' 1= y'-x:
32
5

IS 'Jo o.` =l-2


":.i S= 12` lr
"S
U.I II . l t,
`
c

)
,
)17 t .L7 ` f21 r2_ ) ` =
:)ta-- Lt - )( `
'(' S
x'. 3'l IL


-1t-i,., 1- `2'(3)`


x= f+.rr
1
c,l"*- )
I
:`
15 ss`)


. 5g r

=

X,= `86l k

) 11= X )(

F :

` 3. , (
)Y K
1 Z Jl
,
Fl
l

,r J `
._ 83 t 1[:
= 2
g
4[
,11



r 3 l 31
=

U 3 f//

Li ][
SJL 2
2)


6 : x^ (laD


2[ J
,

(X3 4X
( X )

` +2
d . _
N(10
X

)
l
() x

=
+2

' "6 Re

tttJi Jx
r ril >

=-7(
, .1213
)

0187

l,


6)


ry: (t) t"t. q. J Y+z P]


tJ
lsl !-'-x+2


o1(t) -- a-- 2

1 g//

3//

6)

1


2]


t )


f`, i
s)
ili l J 1 1 _l



5 n= l?.?o )

,
6_
:


q.: p= 0.6< Xr`


H,: i'ri<0.6( `

.. `

(c-,"1 rr 5?i = L 12`ISILlS


4



`

ra1 l.t. f( 2
o( `:
-ii*1 stjr;1';, l 0
, Z ` `2S


,4) c) A)'1 //
= 1? 6 P(5

. , rt'l, . ra G
r
'

(,G
i1un, PI A
/-l\
( <*.c h^tiz* :, s i, e.,'111,., it


j( 1, ^4)i
:fI(
k ^
J :J 1


PDll

)

6) )0) )r )0
i

lot F )

( )
(

lt' J


714,01{ //

a
]:I; , 1'lif
r10 1las212)
: [,
itt,t I (4

7(
O 2 1801117 2, l

` .1117
:

]


T


I
^

x ) (22) 6(b) 1 8) 8( X ),
.

=31 ( )

2

+6 (1'(1)+8 ? 4 ),08
,23 2

F` 0't'2
` =
/ 711
s2,

D3
Pi]
.8,23
//
0 882// F(
)'06
0,) fi- tuA c.i , =
?5
IA
lr^'{.1 -
` Ff =70-O t
`__C)
\.b07 <t< s1g? C) C): ( 4 =Y23 0 O , 3
// 8=, 2 i,1
)
'0

l;i3//

-,? lX.. t.s+r _431 // _ _ _


7

.
.',(01511
' 7


.
'ty' o.oizb V t(3F;] ;

.'. / =
(p.otzexz)*1sq
4


Ff'(7':


1

L.^ P( ,,3)_ C(Y> g )


-
- b \t /./.
'PO>2862)
o4
11
=00021//


I I I I :



.
1 ) t


Ett'49V

lF tt


x -uF,tl F(Y) (


,]
lG)= t, -t<.r<( 4- ]
:> r&)-- !-1t;*,


f ] +





(x,f

F

` (` E(
)(,


P(Xl)

(* ls x<s

[2( 'X'72 ]
= r

[
CX`) 12
2)

L


)=C( f )2

= )2,
`2//
. l-%


P[Y=,r) = t 1//


,61".-)
rl =


l r(\ fto')a --*)
)
("


(
J

`

0
//
cf) y(Y=1)'k G' a_ )

0) tn d Tn=1
2]



( 1)


1 )
E(



1:1 ,
lY: )
`

= //.
_ .

=
ar
=]
6
Pr<`2
"

J, `]
2
(b )'



: (
2r l



: t?Fll ` rJ 11)
9)r-9 111]

i
`1`)(an
00
1:

l+LD"- t-gob+11'o

('

.

You might also like